Direct Tax by CA. Akash Sir - Only Questions
Direct Tax by CA. Akash Sir - Only Questions
Direct Tax by CA. Akash Sir - Only Questions
HW Question: 1
Calculate tax liability assuming that the assessee has opted out of default tax
regime in following cases-
(i) Mr. Ajit has total income of ₹8,00,000
(ii) Mr. Anmol has total income of ₹10,00,005
(iii) Mr. Ajay has total income of ₹12,00,000
(iv) Mr. Rohan has total income of ₹14,00,004
(v) Mr. Soham has total income of ₹23,00,000
(vi) Miss Anamika has total income of ₹54,00,000
(vii) Mrs. Manavi has total income of ₹1,30,00,008
(viii) Mr. Akash has total income of ₹2,23,00,000
(ix) Miss Kalpana has total income of ₹5,40,00,000
It is assumed that the age of above individuals is below 60 years during P.Y. 2023-24.
HW Question: 2
Calculate tax liability assuming that the assessee has opted out of default tax
regime in following cases -
(i) Mr. Ajit, a resident senior citizen has total income of ₹8,00,000
(ii) Mr. Anmol, a resident of age 45 years has total income of ₹10,00,005
(iii) Mr. Ajay, a non-resident senior citizen has total income of ₹12,00,000
(iv) Mr. Rohan, a resident of age 61 years has total income of ₹14,00,004
(v) Mr. Soham, a resident senior citizen has total income of ₹23,00,000
(vi) Miss Anamika, a non-resident senior citizen has total income of ₹54,00,000
(vii) Mrs. Manavi, a resident of age 25 years has total income of ₹1,30,00,008
(viii) Mr. Akash, a resident of age 16 years has total income of ₹2,23,00,000
(ix) Miss Kalpana, a resident senior citizen has total income of ₹5,40,00,000
HW Question: 3
Calculate tax liability assuming that the assessee has opted out of default tax
regime in following cases -
(i) Mr. Ajit, a resident super senior citizen has total income of ₹8,00,000
(ii) Mr. Anmol, a resident of age 95 years has total income of ₹1,20,00,000
(iii) Mr. Ajay, a non-resident super senior citizen has total income of ₹12,00,000
HW Question: 4
HW Question: 5
HW Question: 6
Mr. Bablu has total income of ₹2,05,00,000 comprising of Income from house property
and bank interest. Calculate tax liability of Mr. Bablu assuming that:
a) His age is 45 years and he is resident
b) His age is 65 years and he is non-resident
c) His age is 85 years and he is resident
Assuming that Mr. Babu has not opted for the provisions of section 115BAC.
HW Question: 7
Calculate tax liability for the following for A.Y. 2024-25 in following cases:
a) Mr. Rajkumar, resident individual, aged 46 years having total income of
₹4,99,000
b) Mr. Shyam, a non-resident individual, aged 59 years having total income of
₹3,00,000
Illustration: 1
Solution:
As per section 6(1), an individual is said to be resident in India in any previous year, if
he satisfies any one of the following two conditions:
(i) He stays in India for 182 days or more during the relevant previous year;
(ii) He stays in India for 60 days or more during relevant previous year and also for
365 days or more during 4 years preceding the relevant previous year.
If the individual does not satisfy any one of the above two conditions, he is a non-
resident.
HW Question: 1
Determine residential status of individual in following cases. Write your answer along
with relevant provision and explanation (for practice purpose):
3. Meenu 165 65
4. Sheenu 182 64
5. Monu 125 370
6. Kanu 25 400
HW Question: 2
Mr. Mohan has a flat in Delhi where he normally resides while he visits India. On
02.07.2023, he visited India and stays in Hotel in Mumbai for some personal purpose.
Determine his residential status for A.Y. 2024-25. He claims that he shall be treated
as non-resident since he has not stayed at usual place of residence. Comment.
Illustration: 2
Miss Monika, an Indian citizen, leaves India on 22.09.2023 for the first time, to work
as an officer of a company in France. Determine her residential status for the A.Y.
2024-25.
Solution:
As per section 6(1), an individual is said to be resident in India in any previous year, if
he satisfies any one of the following two conditions:
(i) He stays in India for 182 days or more during the relevant previous year;
(ii) He stays in India for 60 days or more during relevant previous year and also for
365 days or more during 4 years preceding the relevant previous year.
If the individual does not satisfy any one of the above two conditions, he is a non-
resident.
Also, as per explanation to section 6(1), any individual who is a citizen of India and has
left India for taking up any business or profession or employment outside India will be
considered to be resident only if they stay in India for 182 days or more during the
relevant previous year.
During the previous year 2023-24, Miss Monika, an Indian citizen, stayed in India for
175 days (i.e. 30 + 31+ 30 + 31 + 31 + 22 days). Therefore, she does not satisfy the
minimum criteria of 182 days. Also, since she is an Indian citizen leaving India for the
purposes of employment, the second condition under section 6(1) is not applicable to
her. Therefore, Miss Monika is a non-resident in India for the A.Y. 2024-25.
HW Question: 3
Mr. Hansraj, an Indian Citizen, has business in India and has left India on 29.06.2023
to USA for a business deal. He came back to India on 05.04.2024. His total stay in
India during past 4 PYs is 380 days. Determine his residential status for A.Y. 2024-25.
Illustration: 3
Mr. Alok and Mrs. Alok are settled outside India for the purpose of employment and
they came to India on 15.10.2023 on a visit for 7 months. Both of them are Indian
citizens. In the earlier years they were in India as follows:
Year Mr. Alok Mrs. Alok
2022 – 2023 235 Days 365 Days
2021 – 2022 330 Days 30 Days
2020 – 2021 Nil 28 Days
2019 – 2020 118 Days 120 Days
Find out the residential status of Mr. Alok and Mrs. Alok for the assessment year
2024-25.
Solution:
As per section 6(1), an individual is said to be resident in India in any previous year, if
he satisfies any one of the following two conditions:
(i) He stays in India for 182 days or more during the relevant previous year;
(ii) He stays in India for 60 days or more during relevant previous year and also for
365 days or more during 4 years preceding the relevant previous year.
If the individual does not satisfy any one of the above two conditions, he is a non-
resident.
Also, as per explanation to section 6(1), any individual who is a citizen of India or is a
person of Indian origin and is having business or profession or employment outside
India comes on a visit to India during relevant previous year will be considered to be
resident only if they stay in India for 182 days or more during the relevant previous
year.
Stay of Mr. Alok in India during previous year 2023-24 is 169 Days {17 + 30 + 31 + 31 +
29 + 31} and stay of Mrs. Alok in India during previous year 2023-24 is 169 Days {17 +
30 + 31 + 31 + 29 + 31}. Since they are employed outside Indian and has come to visit
India during P.Y. 2023-24, they will be resident only if their stay in India in relevant
previous year is 182 days or more, hence they are non–resident.
In the year P.Y. 2023-2024, a sailor has remained on ship as a crew member of an
Indian Ship as follows:
(1) Outside the territorial waters of India for 184 days.
(2) Inside the territorial waters of India for 182 days.
Is he considered to be resident or not for the Assessment Year 2024-25. Comment.
Solution:
As per section 6(1), an individual is said to be resident in India in any previous year, if
he satisfies any one of the following two conditions:
(i) He stays in India for 182 days or more during the relevant previous year;
(ii) He stays in India for 60 days or more during relevant previous year and also for
365 days or more during 4 years preceding the relevant previous year.
Also, as per explanation to section 6(1), any individual who is a citizen of India and has
left India as a member of crew of an Indian ship will be considered to be resident only
if they stay in India for 182 days or more during the relevant previous year.
Illustration: 5
Mr. Ramesh is an Indian citizen and a member of the crew of a Singapore bound Indian
ship engaged in international traffic departing from Mumbai port on 6th June, 2023.
From the following details for the P.Y. 2023-24, determine the residential status of
Mr. Ramesh for A.Y. 2024-25, assuming that his stay in India in the last 4 previous
years (preceding to P.Y. 2023-24) is 400 days and last seven previous years (preceding
to P.Y. 2023-24) is 750 days:
Particulars Date
Date entered into the Continuous Discharge Certificate in
respect of joining the ship by Mr. Ramesh 6th June, 2023
Date entered into the Continuous Discharge Certificate in
respect of signing off the ship by Mr. Ramesh 9th December, 2023
Solution:
As per section 6(1), an individual is said to be resident in India in any previous year, if
he satisfies any one of the following two conditions:
(i) He stays in India for 182 days or more during the relevant previous year;
(ii) He stays in India for 60 days or more during relevant previous year and also for
365 days or more during 4 years preceding the relevant previous year.
Also, as per explanation to section 6(1), any individual who is a citizen of India and has
left India as a member of crew of an Indian ship will be considered to be resident only
if they stay in India for 182 days or more during the relevant previous year. Moreover,
the time period mentioned in Continuous Discharge Certificate (CDC) shall be
considered to be the period of stay outside India and remaining time period shall be
considered to be stay in India.
Hence, the period beginning from 6th June, 2023 and ending on 9th December, 2023,
being the dates entered into the Continuous Discharge Certificate in respect of joining
the ship and signing off from the ship by Mr. Ramesh, has to be excluded for computing
the period of his stay in India. Accordingly, 187 days [25+31+31+30+31+30+9] have to
be excluded from the period of his stay in India. Consequently, Mr. Ramesh’s period of
stay in India during the P.Y. 2023-24 would be 179 days [i.e., 366 days – 187 days].
Since his period of stay in India during the P.Y. 2023-24 is less than 182 days, he is a
non-resident for A.Y. 2024-25.
Note - Since the residential status of Mr. Ramesh is “non-resident” for A.Y. 2024-25
consequent to his number of days of stay in P.Y. 2023-24 being less than 182 days, his
period of stay in the earlier previous years become irrelevant.
You are required to determine the residential status of Mr. Kamal, a citizen of India,
for the previous year 2023-24.
Mr. Kamal is a member of crew of a Singapore bound Indian ship, carrying passengers in
the international waters, which left Kochi port in Kerala, on 16th August, 2023.
Following details are made available to you for the previous year 2023-24:
Particulars Date
Date entered into the Continuous Discharge Certificate in
respect of joining the ship by Mr. Kamal 16th August, 2023
Date entered into the Continuous Discharge Certificate in
respect of signing off the ship by Mr. Kamal 21st January, 2024
In June, 2023, he had gone out of India to Dubai on a private tour for a continuous
period of 27 days. During the last four years preceding the previous year 2023-24, he
was present in India for 425 days. During the last seven previous years preceding the
previous year 2023-24, he was present in India for 830 days.
Illustration: 6
Mr. Alok and Mrs. Alok are settled outside India for the purpose of employment and
they came to India on 15.10.2023 on a visit for 7 months. Both of them are Indian
citizens. In the earlier years they were in India as follows:
Year Mr. Alok Mrs. Alok
2022 – 2023 235 Days 365 Days
2021 – 2022 330 Days 30 Days
2020 – 2021 Nil 28 Days
2019 – 2020 118 Days 120 Days
Find out the residential status of Mr. Alok and Mrs. Alok for the assessment year
2024-25 assuming that Mr. Alok and Mrs. Alok has total income from India of
₹16,00,000 and 14,00,000 respectively during P.Y. 2023-24.
Solution:
As per section 6(1), an individual is said to be resident in India in any previous year, if
he satisfies any one of the following two conditions:
(i) He stays in India for 182 days or more during the relevant previous year;
(ii) He stays in India for 60 days or more during relevant previous year and also for
365 days or more during 4 years preceding the relevant previous year.
If the individual does not satisfy any one of the above two conditions, he is a non-
resident.
Also, as per explanation to section 6(1), any individual who is a citizen of India or is a
person of Indian origin and is having business or profession or employment outside
India comes on a visit to India during relevant previous year will be considered to be
resident only if they stay in India for 182 days or more during the relevant previous
year.
Also, in case of an individual who is “a citizen of India or is a person of Indian origin and
is having business or profession or employment outside India comes on a visit India
during relevant previous year” having income other than income from foreign sources
exceeding ₹15 lakhs during the previous year will be treated as resident in India if:
(i) He stays in India for 182 days or more during the relevant previous year; or
(ii) He stays in India for 120 days or more during relevant previous year and also for
365 days or more during 4 years preceding the relevant previous year.
Further as per section 6(6)(c), he will be NOR if his stay is less than 182 days during
relevant previous year
Stay of Mr. Alok in India during previous year 2023-24 is 169 Days {17 + 30 + 31 + 31 +
29 + 31} and stay of Mrs. Alok in India during previous year 2023-24 is 169 Days {17 +
30 + 31 + 31 + 29 + 31}.
Therefore, Mr. Alok having Indian income of ₹16,00,000 would be treated as NOR
during A.Y. 2024-25 and Mrs. Alok is non–resident.
HW Question: 5
Mr. Kushal, an Indian Citizen is employed outside India and he visited India on
01.11.2023 and left India on 30.05.2024. In the earlier years he was in India as follows:
Year Days
2022 – 2023 235 Days
2021 – 2022 330 Days
2020 – 2021 Nil
2019 – 2020 118 Days
Find out the residential status of Mr. Kushal for the assessment year 2024-25
assuming that:
a) His total income from India is ₹12,00,000 for P.Y. 2023-24
b) His total income from India is ₹17,00,000 for P.Y. 2023-24
Illustration: 7
Mr. James Bond an American citizen has come to India for the first time on
10.07.2019, as an employee of a multinational company. The particulars of his arrival
and departure are as given below:
Date of Arrival Date of Departure
10.07.2019 07.08.2020
07.10.2020 27.11.2021
01.03.2022 01.02.2023
10.05.2023 30.03.2024
Not yet returned
Determine his residential status for previous year 2019-20 to 2023-24 assuming that
the provisions for A.Y. 2024-25 were applicable during prior years as well.
10 | P a g e www.hakkseca.com/ 783-892-5588
11
2. Residential Status and Scope of Total Income
Solution:
As per section 6(1), an individual is said to be resident in India in any previous year, if
he satisfies any one of the following two conditions:
(i) He stays in India for 182 days or more during the relevant previous year;
(ii) He stays in India for 60 days or more during relevant previous year and also for
365 days or more during 4 years preceding the relevant previous year.
If the individual does not satisfy any one of the above two conditions, he is a non-
resident.
As per section 6(6)(a) of the Act, an individual who is resident in India shall be
considered to be NOR if he has complied with at least one of the conditions given
below:
(i) If such individual has during the 7 previous years preceding the relevant previous
year been in India for a period of 729 days or less, or;
(ii) If such individual has been non-resident in India in atleast 9 years out of 10
previous years preceding the relevant previous year.
11 | P a g e www.hakkseca.com/ 783-892-5588
12
2. Residential Status and Scope of Total Income
HW Question: 6
Mr. Shiva an American citizen has come to India for the first time on 01.07.2020 as an
executive of a multinational company. His employer has allowed him to visit USA every
year and for this purpose he will be leaving India every year on 1st November and shall
come back on 31st December, besides that he has visited Hong Kong on several
occasions in connection with the official work, because he is looking after the
employer’s operations in Hong Kong also, with details as under:
HW Question: 7
Mr. Dey, a non-resident, residing in US since 1990, came back to India on 1.4.2022 for
permanent settlement. What will be his residential status for assessment year 2024-
25?
Illustration: 8
One HUF is controlled and managed from India. The Karta of the Hindu Undivided
Family comes to India every year for minimum 60 days and maximum 91 days.
Determine residential status of the Hindu Undivided Family and also that of the Karta
for the assessment year 2024-25.
12 | P a g e www.hakkseca.com/ 783-892-5588
13
2. Residential Status and Scope of Total Income
Solution:
Hindu Undivided Family is resident since it is controlled and managed from India. Since,
the total stay of the Karta during seven years can be maximum 637 days, hence, Hindu
Undivided Family shall be considered to be resident but not ordinarily resident. Total
stay of Karta during PY is maximum 91 days and during 4 preceding PYs is 364 days
therefore, in his individual capacity is non-resident because he cannot comply with even
one of the two conditions given under section 6(1).
HW Question: 8
One Hindu Undivided Family is being managed partly from Mumbai and partly from
Nepal. Mr. Sam (a foreign citizen), Karta of Hindu Undivided Family, comes on a visit to
India every year since 1982 in month of April for 105 days. Determine residential
status of the Hindu Undivided Family and also that of the Karta in his individual
capacity for the assessment year 2024-25.
State with reason, whether the following statements are True or False:
Mr. Ram, Karta of HUF, claims that the HUF is non-resident as the business of HUF is
transacted from UK and all the policy decisions are taken there.
Illustration: 9
Solution:
As per Section 6(3), a company would be resident in India in any previous year, if-
(i) it is an Indian company; or
13 | P a g e www.hakkseca.com/ 783-892-5588
14
2. Residential Status and Scope of Total Income
In the case of POPULAR Inc., its place of effective management for P.Y. 2023-24 is
not in India, since the significant management and commercial decisions are, in
substance, made by the Board of Directors outside India in Sweden. POPULAR Inc. has
only a liaison office in India through which it looks after its routine day to day business
operations in India. The place where decisions relating to day-to-day routine operations
are taken and support functions that are preparatory or auxiliary in nature are
performed are not relevant in determining the place of effective management.
Hence, POPULAR Inc., being a foreign company is a non-resident for A.Y. 2024-25,
since its place of effective management is outside India in the P.Y. 2023-24.
HW Question: 10
Adani Ltd. an Indian company has most of its business outside India. Determine its
residential status.
HW Question: 11
HW Question: 12
Best Ltd., a foreign company and it carries on majority of its operations and decision
making activities from Calcutta and Assam but some part of operational activities and
few decisions are being taken from the place at which registered office of Best Ltd. is
located, i.e. Dhaka. Determine its residential status for the assessment year 2024-25.
14 | P a g e www.hakkseca.com/ 783-892-5588
15
2. Residential Status and Scope of Total Income
Illustration: 10
Mr. Ashish, an American Citizen has following incomes during the P.Y. 2023-24:
Solution:
Illustration: 11
Mr. Manmohan, an Indian citizen aged 45 years, a government employee serving in the
Ministry of External Affairs, left India for the first time on 31.03.2023 due to his
transfer to High Commission of Singapore. He did not visit India any time during the
previous year 2023-24. He has received the following income for the Financial Year
2023-24:
S.No. Particulars ₹
(i) Salary (Computed) 5,00,000
(ii) Foreign Allowance [not included in (i) above] 4,00,000
(iii) Interest on fixed deposit from bank in India 1,00,000
(iv) Income from agriculture in Nepal 2,00,000
(v) Income from house property in Nepal 2,50,000
Compute his Gross Total Income for Assessment Year 2024-25.
Solution:
15 | P a g e www.hakkseca.com/ 783-892-5588
16
2. Residential Status and Scope of Total Income
As per section 6(1), Mr. Manmohan is a non-resident for the A.Y. 2024-25, since he was
not present in India at any time during the previous year 2023-24.
In view of the above provisions, income from agriculture in Nepal and income from
house property in Nepal would not be chargeable to tax in the hands of Manmohan,
assuming that the same were received in Nepal. Income from ‘Salaries’ payable by the
Government to a citizen of India for services rendered outside India is deemed to
accrue or arise in India as per section 9(1)(iii). Hence, such income is taxable in the
hands of Mr. Manmohan, even though he is a non-resident.
Illustration: 12
Calculate her gross total income from A.Y. 2024-25 assuming that she is (i) ROR (ii)
NOR (iii) NR.
Solution:
16 | P a g e www.hakkseca.com/ 783-892-5588
17
2. Residential Status and Scope of Total Income
Mr. Ram, left for USA on 01.05.2023. He has not visited India thereafter. Mr. Ram
borrows money from his friend Mr. Laxman, who left India one week before Mr. Ram's
departure, to the extent of ₹10 lakhs and buys shares in Ram Ltd., an Indian company.
Discuss the taxability of the interest charged @ 10% in Mr. Laxman's hands where the
same has been received in New York.
Solution:
Stay of Mr. Ram and Mr. Laxman during the previous year 2023-24 is less than 60 days
hence both of them are non-residents as per section 6(1).
As per section 9, if any non-resident has taken loan from outside India and the loan was
utilized in India in any source other than business or profession, interest received by
the person who has given the loan shall not be considered to be accruing/arising in
India and is not taxable in India. In the given case, loan amount was invested in the
shares of an Indian company hence interest received by Mr. Laxman shall not be
considered to be income accruing/arising in India.
Solution:
17 | P a g e www.hakkseca.com/ 783-892-5588
18
2. Residential Status and Scope of Total Income
As per section 9, if any non-resident has provided any patent right or any managerial,
technical services and such patent right etc was used in India, in such cases any royalty
or fee received by non-resident shall be considered to be income accruing/arising in
India and shall be taxable and it do not matter that the non-resident do not have
residence or place of business or business connection in India i.e. there is no territorial
nexus or non-resident has not rendered services in India. In the instant case, since the
services were utilized in India, the payment received by Mr. Kulasekhara, a non-
resident, in Colombo is chargeable to tax in his hands in India, as it is deemed to accrue
or arise in India.
Mr. Mark, a non-resident and citizen of Japan entered into following transactions
during the previous year ended 31.03.2024. Examine the tax implications in the hands
of Mr. Mark for the Assessment Year 2024 - 25 as per Income Tax Act, 1961. (Give
brief reasoning)
(1) Interest received from Mr. John, a non-resident, outside India (The borrowed fund
is used by Mr. John for investing in Indian company's debt fund for earning interest)
(2) Received ₹10 lakhs in Japan from a business enterprise in India for granting license
for computer software (not hardware specific).
(3) He is also engaged in the business of running news agency and earned income of ₹10
lakhs from collection of news and views in India for transmission outside India.
(4) He entered into an agreement with SKK & Co., a partnership firm for transfer of
technical documents and design and for providing services relating thereto, to set up a
Denim Jeans manufacturing plant, in Surat (India). He charged ₹10 lakhs for these
services from SKK & Co.
Solution:
(1) As per section 9, if loan has been taken by a non-resident, interest income shall be
accruing/ arising in India only if loan amount has been utilised in India in business/
profession but if loan amount is utilised in any other source in India or it has been used
outside India, interest income shall be accruing / arising abroad. In the given case, loan
amount is used for investing in Indian company debt fund for earning interest and not
for business purpose hence interest income shall not be considered to be accruing
arising from India and shall not be taxable in India.
(2) As per section 9, if any income is accruing and arising in India relating to royalty or
technical fees etc., it will be taxable in India even if the person receiving income is non-
resident and even if such non-resident does not have any Territorial Nexus with India
i.e., such non-resident does not have a residence or place of business or business
18 | P a g e www.hakkseca.com/ 783-892-5588
19
2. Residential Status and Scope of Total Income
connection in India and also the non-resident has not rendered services in India. In the
given case, income received for granting licence for computer software shall be deemed
to be income accruing arising in India and shall be taxable in India.
(3) As per section 9, if any non-resident has the business of running a news agency or
of publishing newspapers, magazines or journals etc. outside India, no income shall be
deemed to accrue or arise in India to him from activities which are confined to the
collection of news and views in India for transmission out of India. In the given case,
income is from transmission outside India hence income shall not be deemed to accrue
arise in India and shall not be taxable in India.
(4) As per section 9, income by way of fees for technical services payable by a person
who is a non-resident, where the fees are payable in respect of services utilised in a
business or profession carried on by such person in India or for the purposes of making
or earning any income from any source in India. In the given case, services utilized in a
business in India hence income shall be accruing arising from India and same shall be
taxable in India.
Determine the taxability of income of US based company Mr. Been Ltd., in India on
entering following transactions during the financial year 2023-24:
(i) ₹5 lacs received from an Indian domestic company for providing technical know-how
in India.
(ii) ₹6 lacs from an Indian firm for conducting the feasibility study for the new project
in Finland.
(iii) ₹4 lacs from a non-resident for use of patent for a business in India.
(iv) ₹8 lacs from a non-resident Indian for use of know-how for a business in Singapore.
(v) ₹10 lacs for supply of manuals and designs for the business to be established in
Singapore.
Explain the rate of tax applicable on taxable income for US based company, Mr. Been
Ltd., in India.
19 | P a g e www.hakkseca.com/ 783-892-5588
20
2. Residential Status and Scope of Total Income
Comprehensive question
Brett Lee, an Australian cricket player visits India for 100 days in every financial year.
This has been his practice for the past 10 financial years.
(a) Find out his residential status for the assessment year 2024-25.
(b) Would your answer change if the above facts relate to Srinath, an Indian citizen
who resides in Australia and represents the Australian cricket team?
(c) What would be your answer if Srinath had visited India for 120 days instead of 100
days every year, including P.Y. 2023-24?
20 | P a g e www.hakkseca.com/ 783-892-5588
21
2. Residential Status and Scope of Total Income
4. He has one house property in Ghaziabad and income of ₹5,00,000 was received in UK.
5. He has received salary income of ₹5,00,000 (computed) in India and half of the
services were rendered in UK and half in India.
(Presume all the above incomes are computed incomes)
Compute his income presuming that he is NOR, NR and ROR.
Mr. Amir had following income during the previous year ended 31st March, 2024:
(1) Salary received in India for three months (being computed income) 25,000
(2) Income from house property in India 18,000
(3) Interest on savings bank deposit in SBI, in India 4,000
(4) Amount brought into India out of the past-untaxed profits earned
in Germany 20,500
(5) Income from business in Bangladesh, being controlled from India 12,542
You are required to compute his gross total income for the assessment year 2024-25,
if he is a
(a) resident and ordinarily resident;
(b) not ordinarily resident; and
(c) non-resident.
Presume all the above income is computed income.
Mr. Rahman earns the following income during the financial year 2023-24:
(1) Income from house property in London, received in India 60,000
(2) Profits from business in Japan and managed from there
(received in Japan) 9,00,000
(3) Profits from business in Kenya, controlled from India, Profits
received in Kenya 3,00,000
(4) Profits from business in Delhi, managed from Japan 7,00,000
(5) Capital gains on transfer of shares of Indian companies, sold in
USA and gains were received there 2,00,000
(6) Pension from former employer in India, received in Japan 50,000
(7) Profits from business in Pakistan, deposited in bank there 20,000
(8) Profit on sale of asset in India but received in London 8,000
(9) Past untaxed profits of UK business of 2020-21 brought into India
in 2023-24 90,000
(10) Interest on Government securities accrued in India but received in
Paris 80,000
(11) Interest on USA Government securities, received in India 20,000
(12) Salary earned in Bombay, but received in UK 60,000
21 | P a g e www.hakkseca.com/ 783-892-5588
22
2. Residential Status and Scope of Total Income
Ramrahim Pvt. Ltd., an Indian company has an income of ₹30 lakhs from a business in
India. This company has a business income of ₹12 lakhs from outside India. Out of
which 7 lakhs were received in India and balance outside India.
Compute total income of the Indian company for the assessment year 2024-25.
Mr. John, a foreign citizen (not being a person of Indian origin) came to India for the
first time on 2nd December, 2023 for a visit of 210 days. Mr. John had the following
income during the previous year ended 31st March, 2024:
(1) Salary (computed) received in India for three months 1,00,000
(2) Income from house property in London (received there) 2,75,200
(3) Amount brought into India out of the past-untaxed profits earned
in Germany 80,000
(4) Income from agriculture in Sri Lanka, received and invested there 12,300
(5) Income from business in Nepal, being controlled from India 35,000
(6) Income from house property in USA received in USA (₹76,000 is used in
Canada for meeting the educational expenses of Mr. John’s daughter and
₹10,000 is later on remitted in India) 86,000
You are required to compute his total income for the assessment year 2024-25.
Mr. Rohan earns the following incomes during the financial year 2023-24.
(1) Profits from a business in Japan, controlled from India,
(half of the profits received in India) 40,000
(2) Income from property in Bombay, received in UK 70,000
(3) Income from a property in USA, received there but subsequently
remitted to India 2,00,000
(4) Income from property in USA, received there (₹50,000 remitted
in India) 80,000
(5) Salary received in India for services rendered in USA 50,000
(6) Income from profession in Paris, which was set up in India, received
in Paris 80,000
22 | P a g e www.hakkseca.com/ 783-892-5588
23
2. Residential Status and Scope of Total Income
(7) Interest from deposit with an Indian company, received in Japan 9,000
(8) Income from profession in Bombay received in Paris 30,000
(9) Profits of business in Iran, deposited in a bank there, business controlled
from India (out of ₹4,00,000, ₹1,00,000 is remitted in India) 4,00,000
(10) Interest on German development bonds, half of which is received in
India 10,000
(11) Income from property in Canada, one-fifth is received in India 50,000
(Presume all the above incomes are computed income i.e. all the exemptions and
deductions have already been allowed)
Determine the gross total income of Mr. Rohan if he is (i) resident and ordinarily
resident, (ii) resident but not ordinarily resident, (iii) non-resident in India during the
financial year 2023-24.
Question: 9 [Residential status, scope of total income, salary, gift, Agri Income]
Mr. Mohan is a citizen of India and is employed in Sarla Limited and getting salary
₹1,00,000 p.m. and he was transferred out of India on 01.09.2023 and he left India for
first time from 01.09.2023 and he visited India from 26.01.2024 to 15.02.2024 and
salary for January 2024 was received in India and at the time of departure he received
3 gifts ₹20,000 from 3 friends each and also a phone of ₹70,000. He has agricultural
income in India ₹4,00,000.
Compute his tax liability for assessment year 2024-25.
Mrs. Mehta is a citizen of India and is employed in Raj Ltd. in India and is getting
salary of ₹60,000 p.m. and she was transferred out of India w.e.f. 01.09.2023 and for
this purpose she left India on 01.09.2023 for the first time and she visited India from
27.12.2023 to 07.01.2024 and her salary for the month of Dec’ 2023 was received in
India. Employer and employee both have contributed @ 13% (each) of salary to the
recognized provident fund and during the year interest of ₹50,000 was credited to the
recognized provident fund @ 10% p.a.
Compute her total income and tax liability in India for assessment year 2024-25.
(b) Presume she was transferred w.e.f. 01.11.2023 and she left India on 01.11.2023 for
the first time.
Determine the scope of the following incomes in the hands of a resident and ordinarily
resident, resident but not ordinarily resident, and non-resident for the A. Y. 2024-25 –
23 | P a g e www.hakkseca.com/ 783-892-5588
24
2. Residential Status and Scope of Total Income
Calculate taxable income of an individual on the basis of the following information, for
the assessment year 2024-25, if he is:
(a) Ordinarily Resident
(b) Not Ordinarily Resident; and
(c) Non-Resident
24 | P a g e www.hakkseca.com/ 783-892-5588
25
2. Residential Status and Scope of Total Income
Mr. Ram earns the following income during the previous year 2023-24.
Compute his gross total income for assessment year 2024-25 if he is
(i) resident and ordinarily resident.
(ii) resident but not ordinarily resident.
(iii) non-resident.
(1) Income from agricultural land in Bhutan received there and
remitted to India later on 40,000
(2) Pension for service rendered in India, but received in Paris 15,000
(3) Past untaxed profits of 2022-23 brought into India in 2023-24 50,000
(4) Profits from business in Paris, deposited in bank there 1,00,000
(5) Profits from business in Canada, controlled from India, profits received
there 1,75,000
(6) Interest on saving bank deposit in Punjab National Bank, in India 20,000
(7) Capital gain on sale of a house in Delhi, amount received in Paris 2,00,000
Mr. Rahim earns the following income during the previous year 2023-24. Compute his
Gross total income for assessment year 2024-25 if he is
(i) resident and ordinarily resident.
(ii) resident but not ordinarily resident.
(iii) non-resident.
(1) Profit on sale of machinery in India, but received in Japan 1,20,000
(2) Profits from business in Bombay, managed from Japan 2,25,000
(3) Profits from business in Japan, managed from there, received there 1,45,000
(4) Income from house property in India 1,50,000
(5) Income from property in Japan and received there 1,50,000
(6) Income from agriculture in Japan being invested there 75,000
(7) Fees for technical services rendered in India but received in Japan 65,000
(8) Interest on Government securities accrued in India but received in
Japan 80,000
(9) Interest on Japan Government securities, received in India 40,000
(Presume that all the incomes are computed incomes)
25 | P a g e www.hakkseca.com/ 783-892-5588
26
2. Residential Status and Scope of Total Income
Mr. Soham earns the following incomes during the financial year 2023-24.
(1) Profits from a business in Japan, controlled from India, half of the
profits received in India 60,000
(2) Income from agriculture in Nepal, brought to India 10,000
(3) Income u/h house property in Bombay, received in UK 1,70,000
(4) Income u/h house property in USA, received there but subsequently
remitted to India 2,20,000
(5) Income u/h house property in USA, received there (₹50,000 remitted
in India) 1,00,000
(6) Salary received in India for services rendered in USA 60,000
(7) Income from profession in Paris, which was set up in India, received in
Paris 90,000
(8) Interest from deposit with an Indian company, received in Japan 19,000
(9) Income from profession in Bombay received in Paris 39,000
(10) Profits of business in Iran, deposited in a bank there, business
controlled from India (out of ₹4,80,000, ₹1,00,000 is remitted in India) 4,80,000
(11) Interest on German development bonds, half of which is received in
India 12,000
(12) Income under the head house property in Canada, one-fifth is received
in India 50,000
(Presume all the above incomes are computed income i.e. all the exemptions and
deductions have already been allowed)
Determine the gross total income of Mr. Soham if he is
(i) resident and ordinarily resident,
(ii) resident but not ordinarily resident,
(iii) non-resident in India during the financial year 2023-24.
Mr. Mohan is a citizen of India and is employed in Lenskart Ltd and is getting a salary
of ₹60,000 p.m. He purchased one building in India on 1st May, 2023 for ₹10,00,000
and its market value is ₹22,00,000 and value for the purpose of charging stamp duty is
₹13,00,000. He purchased gold for ₹8,00,000 and its market value is ₹11,00,000. He
was transferred out of India w.e.f. 1st Sept, 2023 and he left India on 1st Sept, 2023.
One of his friends gifted him one colour TV on this occasion, market value ₹1,00,000.
He has gone out of India in earlier years also.
P.Y. 2022-23 100 days
P.Y. 2021-22 200 days
26 | P a g e www.hakkseca.com/ 783-892-5588
27
2. Residential Status and Scope of Total Income
He visited India from 01.02.2024 to 14.02.2024 and salary for January, 2024 was
received in India.
He has purchased one house property in USA in December 2023 and sold in March
2024 and there was short term capital gain of ₹6,00,000 and the amount was received
in USA. Compute his tax liability for the A.Y. 2024-25 under normal provisions of the
Act.
Mrs. X is employed in ABC Ltd in India and she is an American citizen and is getting a
salary of ₹2,00,000 p.m.
She received gift of one painting in India from her friend on 01.07.2023 and its market
value is ₹49,000 and she also received gift in cash of ₹49,000 from the same friend
and gift of immovable property with value for the purpose of charging stamp duty is
₹51,000 from the same friend.
She purchased UK Development bond and interest equivalent of ₹2,00,000 was
received in USA.
She visited USA for 182 days during P.Y. 2023-24.
In the earlier year her stay in India was:
P.Y. 2022-23 110 days
P.Y. 2021-22 120 days
P.Y. 2020-21 300 days
P.Y. 2019-20 182 days
P.Y. 2018-19 185 days
P.Y. 2017-18 200 days
P.Y. 2016-17 300 days
Compute her tax liability in India for the A.Y. 2024-25 under normal provisions of the
Act.
The following are the income of Shri Subhash Chandra, a citizen of India for the
previous year 2023-24:
(i) Income from business in India ₹2,00,000. The business is controlled from London
and ₹60,000 were remitted to London.
(ii) Profits from business earned in Japan ₹70,000 of which ₹20,000 were received in
India. This business is controlled from India.
(iii) Untaxed income of ₹1,30,000 for the year 2020-21 of a business in England which
was brought in India on 3rd March, 2024.
(iv) Royalty of ₹4,00,000 received from Shri Ramesh, a resident for technical service
provided to run a business outside India.
(v) Agricultural income ₹90,000 in Bhutan.
27 | P a g e www.hakkseca.com/ 783-892-5588
28
2. Residential Status and Scope of Total Income
(vi) Income of ₹73,000 from house property in Dubai, which was deposited in bank at
Dubai. Compute Gross total income of Shri Subhash Chandra for the A.Y. 2024-25, if
he is –
(1) A resident and Ordinarily Resident, and
(2) A resident and Not Ordinarily Resident
Mr. Bachhan has provided the following details of his income for the year ended
31.03.2024.
(1) Short term capital gains on sale of shares in Indian company
received in Japan. 85,000
(2) Rent from property in Bangladesh deposited in a bank at Dhaka,
later on, remitted to India through approved banking channels. 96,000
Compute his total income for the Assessment Year 2024-25 in case of he is:
(i) Resident and ordinarily resident;
(ii) Resident but not ordinarily resident; or
(iii) Non-resident
Question: 20 [Residential status, Scope of total income] [Nov 2018 (Old Course)]
Mr. Surya, an Indian citizen, travelled frequently out of India for his business trip as
well as for his outings. He left India from Mumbai airport on 15th May 2023 as
stamped in the passport. He has been in India for less than 365 days during the 4
years immediately preceding the previous year and has not been in India for at least 60
days in the previous year.
Determine:
(A) Residential status of Mr. Surya and
(B) Total income for the assessment year 2024-25 from the following information:
(1) Short term capital gain on the sale of shares of Trena India Ltd. a listed Indian
company amounting to ₹35,000. The sale proceeds were credited to his Swiss bank
account.
(2) Interest on fixed deposit with State Bank of India (Mumbai) amounting to
₹8,000 was credited to his saving account.
28 | P a g e www.hakkseca.com/ 783-892-5588
29
2. Residential Status and Scope of Total Income
Mrs. X and Mrs. Y are sisters and they earned the following income during the Financial
Year 2023-24. Mrs. X is settled in Malaysia since 2018 and visits India for a month
every year. Mrs. Y is settled in Indore since her marriage in 2018. Compute the total
income of Mrs. X and Mrs. Y for the assessment year 2024-25:
29 | P a g e www.hakkseca.com/ 783-892-5588
30
2. Residential Status and Scope of Total Income
Indian company
(xi) Interest on Fixed Deposit with SBI in India 12,000 8,000
Mr. Dhanush, an Indian citizen aged 35 years, worked in ABC Ltd. in Mumbai. He got a
job offer from XYZ Inc., USA on 01.06.2022. He left India for the first time on
31.07.2022 and joined XYZ Inc. on 08.08.2022. During the P.Y. 2023-24, Mr. Dhanush
visited India from 25.05.2023 to 22.09.2023. He has received the following income for
the previous year 2023-24 –
Particulars Amount
Salary from XYZ Inc., USA received in USA 7,00,000
Dividend from Indian companies 5,50,000
Agricultural income from land situated in Punjab 55,000
Rent received/receivable from house property in Lucknow 4,00,000
Profits from a profession in USA, which was set up in India,
received there 6,00,000
Determine the residential status of Mr. Dhanush and compute his total income for the
A.Y. 2024-25.
30 | P a g e www.hakkseca.com/ 783-892-5588
31
3. Income from house property
Mr. John, a British national, is a resident and ordinarily resident (ROR) in India during
the P.Y. 2023-24. He owns a house in London, which he has let out at £10,000 p.m. The
municipal taxes paid to the Municipal Corporation of London is £8,000 during the P.Y.
2023-24. The value of one £ in Indian rupee to be taken at ₹95. Compute John’s Net
annual value of the property for the A.Y. 2024-25.
Solution:
For the P.Y. 2023-24, Mr. John, a British national, is resident and ordinarily resident in
India. Therefore, as per section 5(1) of the Act, income received by him by way of rent
of the house property located in London is to be included in the total income in India.
Computation of Net Annual Value of the property of Mr. John for A.Y. 2024-25
Note: Municipal taxes paid in London is be to allowed as deduction from the gross
annual value.
Illustration: 2
Mr. Sahil took a loan of ₹5,00,000 on 01.10.2020 @ 10% p.a. for construction of house
which was completed on 31.03.2024. Compute interest on capital borrowed allowable as
deduction for the previous year 2023-24.
Solution:
31 | P a g e www.hakkseca.com/ 783-892-5588
32
3. Income from house property
HW Question: 1
Mr. Sahid has taken a loan of ₹10,00,000 from SBI on 01.04.2021 @ 12% p.a. for
construction of one house which was completed on 01.07.2023 and was let out at a rent
of ₹30,000 per month. He paid municipal taxes ₹40,000. He has taken loan of
₹10,00,000 from PNB on 01.10.2023 @ 10% p.a. to repay the original loan. Compute
interest allowable u/s. 24(b) for A.Y. 2024-25.
HW Question: 2
Mr. Amjad has taken a loan of ₹10,00,000 from SBI on 01.04.2020 @ 10% p.a. for
construction of one house. The Assessee has taken a loan of ₹6,00,000 from PNB on
01.10.2022 @ 12% p.a. to repay loan to SBI. House was completed on 01.07.2023.
Compute interest allowable u/s. 24(b) for A.Y. 2024-25.
Illustration: 3
Mr. Ayush purchased a house property on 01.05.2019 for ₹15,00,000. He took a loan of
₹12,00,000 @ 10% p.a. to purchase the same. Loan is to be repaid in 24 equal monthly
instalments starting from 01.04.2022. Calculate interest allowable u/s. 24(b) for A.Y.
2024-25.
Solution:
HW Question: 3
Mr. Dinesh started the construction of the house on 01.04.2016 with the borrowed
capital of ₹22,00,000 @ 10% p.a. He completed the house on 31.03.2018. Repayment of
loan has been made in 11 annual equal instalments starting from 31.03.2019. Calculate
interest allowable u/s. 24(b) for A.Y. 2024-25.
32 | P a g e www.hakkseca.com/ 783-892-5588
33
3. Income from house property
Miss Charlie, an American national, got married to Mr. Radhey of India in USA on
02.03.2023 and came to India for the first time on 16.03.2023. She left for USA on
23.09.2023. She returned to India again on 27.03.2024. While in India, she had
purchased a show room in Mumbai on 22.04.2023, which was leased out to a company on
a rent of ₹25,000 p.m. from 01.05.2023. She had taken loan from a bank for purchase
of this show room on which bank had charged interest of ₹97,500 up to 31.03.2024.
Determine her residential status and compute the total income chargeable to tax along
with the amount of tax payable on such income for the Assessment Year 2024-25.
Solution:
Her stay in India during the previous year 2023-24 and in the preceding four years is
as under:
P.Y. 2023-24:
01.04.2023 to 23.09.2023 - 176 days and
27.03.2024 to 31.03.2024 - 5 days.
Total 181 days.
Four preceding previous years:
P.Y. 2022- 2023 [16.03.2023 to 31.03.2023] - 16 days
P.Y. 2021- 2022 [01.04.2021 to 31.03.2022] - Nil
P.Y. 2020- 2021 [01.04.2020 to 31.03.2021] - Nil
P.Y. 2019- 2020 [01.04.2019 to 31.03.2020] - Nil
Total 16 days
Since Miss Charlie is not able to comply with any of the conditions mentioned above,
she is non-resident during previous year 2023-24.
33 | P a g e www.hakkseca.com/ 783-892-5588
34
3. Income from house property
Notes:
1. Actual rent received has been taken as the gross annual value in the absence of other
information (i.e. Municipal value, fair rental value and standard rent) in the question.
Case 1: GAV where the property is let out throughout the year [Section
23(1)(a)/(b)]
Illustration: 5
Mr. Babu has one house property which is let out @ ₹80,000 p.m. Fair rent ₹90,000
p.m., Municipal Valuation ₹70,000 p.m., Standard Rent ₹81,000 p.m. Compute the GAV
of the house property.
Solution:
HW Question: 4
Mrs. Babu has let out one house property @ ₹62,000 p.m., municipal valuation ₹72,000
p.m., fair rent ₹90,000 p.m., standard rent ₹1,00,000 p.m. Compute GAV of the House
Property.
34 | P a g e www.hakkseca.com/ 783-892-5588
35
3. Income from house property
Mr. Jagmal owns five houses in Chennai, all of which are let-out. Compute the GAV of
each house from the information given below –
Illustration: 6
Mr. Ramesh has taken a loan of ₹15,00,000 on 01.07.2019 from State Bank of India @
12% p.a. for construction of one house which was completed on 01.04.2022 and was let
out @ ₹90,000 p.m. w.e.f. 01.04.2022 and fair rent is ₹1,00,000 p.m. and the assessee
has paid municipal tax of ₹30,000 in P.Y. 2023-24 and the assessee has repaid the loan
amount in annual instalment of ₹1,00,000 starting from 01.01.2022. Compute his income
tax liability for the assessment year 2024-25 assuming that he has not opted for new
tax regime u/s. 115BAC.
Solution:
Computation of income under the head House Property and tax liability for A.Y.
2024-25
Working Note: 1
(a) Fair Rent (1,00,000 x 12) 12,00,000
(b) Expected Rent 12,00,000
(c) Rent received /receivable (90,000 x 12) 10,80,000
GAV 12,00,000
Working Note: 2
Current period Interest (from 01.04.2023 to 31.03.2024)
(13,00,000 x 12% x 9/12) + (12,00,000 x 12% x 3 /12) = 1,53,000
Prior period interest (from 01.07.2019 to 31.03.2022)
15,00,000 x 12% x 30/12 = 4,50,000
14,00,000 x 12% x 3/12 = 42,000
Instalment = 4,92,000 / 5 = 98,400
35 | P a g e www.hakkseca.com/ 783-892-5588
36
3. Income from house property
HW Question: 6
Mr. Ashok has one house property which is let out @ ₹80,000 p.m. Fair rent ₹90,000
p.m., Municipal Valuation ₹70,000 p.m., Standard Rent ₹81,000 p.m. Municipal tax paid
₹60,000 and interest paid on loan for construction of house property is ₹ 50,000.
Compute his Income Tax Liability for A.Y 2024-25 assuming that he has opted for old
tax regime.
HW Question: 7
Mrs. Verma has let out one House property @ ₹62,000 p.m., Municipal Valuation
₹72,000 p.m., Fair Rent ₹90,000 p.m., Standard Rent ₹1,00,000 p.m., Municipal Tax paid
₹40,000 and Interest on loan taken for construction ₹60,000. She has completed the
age of 60 years on 01.04.2025. Compute Income Tax Liability for the A.Y. 2024-25
assuming that he has opted for old tax regime.
HW Question: 8
Mr. Rajeev has taken a loan of ₹10,00,000 from SBI on 01/04/2020 @ 10% p.a. for
construction of one house. The Assessee has taken a loan of 6,00,000 from PNB on
01/10/2022 @ 12% p.a. to repay loan to SBI. House was completed on 01/07/2023 and
was let out at a rent of ₹1,00,000 per month, paid municipal taxes ₹10,000. Compute his
income and tax liability for Assessment year 2024-25 assuming that he has not opted
for default tax regime u/s. 115BAC.
Case 2: GAV where let-out house property is vacant for part of the year
[Section 23(1)(c)] [IMPORTANT]:
Illustration: 7
Compute gross annual value in the following cases for the assessment year 2024-25:
36 | P a g e www.hakkseca.com/ 783-892-5588
37
3. Income from house property
Solution:
Illustration: 8
Mr. Vakil has taken a loan of ₹15,00,000 on 01.07.2019 from State Bank of India @
12% p.a. for construction of one house which was completed on 01.05.2023 and was let
out @ ₹90,000 p.m. w.e.f. 01.07.2023 and fair rent is ₹1,25,000 p.m. and the assessee
has paid municipal tax of ₹30,000 in P.Y. 2023-24 and the assessee has repaid the loan
amount in annual instalment of ₹1,00,000 starting from 01.01.2022. Compute his income
tax liability for the assessment year 2024-25 assuming that he has not opted for
section 115BAC.
Solution:
Computation of income under the head House Property for A.Y. 2024-25
37 | P a g e www.hakkseca.com/ 783-892-5588
38
3. Income from house property
Working Note: 1
(a) Fair Rent (1,25,000 x 11) 13,75,000
(b) Expected Rent 13,75,000
(c) Rent received /receivable (90,000 x 9) 8,10,000
If there was no vacancy, in that case rent received/receivable would have been
₹9,90,000 and it was still less than expected rent, therefore GAV shall be expected
rent i.e. ₹13,75,000
Working Note: 2
Current period Interest (from 01.04.2023 to 31.03.2024)
(13,00,000 x 12% x 9/12) + (12,00,000 x 12% x 3/12) = ₹1,53,000
Prior period interest (from 01.07.2019 to 31.03.2023)
15,00,000 x 12% x 30/12 = 4,50,000
14,00,000 x 12% x 12/12 = 1,68,000
13,00,000 x 12% x 3/12 = 39,000
Instalment = 6,57,000 / 5 = ₹1,31,400
Total Interest = ₹1,53,000 + ₹1,31,400 = ₹2,84,400
HW Question: 9
Mr. Kavish has let out one house at a rent of ₹50,000 p.m. Fair rent ₹55,000 p.m.
Municipal Valuation ₹52,000 p.m., standard rent ₹60,000 p.m. The house remains vacant
for 3 months. The assessee paid municipal tax ₹30,000. Interest on loan u/s. 24(b) is
₹20,000. Compute Income u/h house property for A.Y. 2024-25.
(b) Presume it is let out at a rent of ₹60,000 P.m.
(c) Presume it is let out at a rent of ₹55,000 P.m.
(d) Presume it is let out at a rent of ₹1,00,000 P.m.
38 | P a g e www.hakkseca.com/ 783-892-5588
39
3. Income from house property
Illustration: 9
Miss Kajal Maheshwari occupied two flats for her residential purpose, particulars of
which are as follows:
Particulars Flat I (in ₹) Flat II (in ₹)
Municipal Valuation p.m. 90,000 45,000
Fair Rent p.m. 1,20,000 40,000
Standard Rent p.m. 80,000 Not available
Municipal taxes paid 10% of municipal valuation 10% of municipal valuation
Fire insurance paid 1,000 600
Interest payable on capital borrowed
for renovation of flat 40,000 Nil
Income of Miss Kajal from her proprietary business, Kajal Warehousing Corporation is
₹6,50,000. Determine the taxable income and tax liability for the assessment year
2024-25 under old provisions of the Act. You are informed that Miss Kajal could not
occupy flat for 2 months commencing from 01.12.2023 and that she has attained the
age of 60 on 23.08.2023.
(ii) Determine the taxable income and tax liability for the assessment year 2024-25
under default provisions of the Act under section 115BAC.
Solution: (i)
As per the regular provisions of Act, in case of self-occupied property, the aggregate
amount of interest shall not exceed ₹30,000/ ₹2,00,000 as the case may be.
Flat I Flat II
Net annual value 0 0
Less: Interest u/s. 24(b) 40,000 0
Income from house property (30,000) 0
[Loss cannot exceed ₹30,000 in case
where loan is taken for renovation]
Total income from house property (30,000)
39 | P a g e www.hakkseca.com/ 783-892-5588
40
3. Income from house property
(ii) Under default tax regime, interest on self-occupied property under section 24(b) is
not allowed to be deducted while calculating income from house property. Therefore,
income from house property shall be Nil.
HW Question: 10
(a) Mr. Ompal has taken a loan of ₹5,00,000 on 01.10.1999 @ 10% p.a. for construction
of a house which was completed on 01.10.2022 and the house remained self-occupied
throughout the previous year 2023-24. The assessee has income under the head PGBP
₹4,00,000. Compute tax liability for assessment year 2024-25 under old tax regime.
(b) Presume in above question, the loan was taken on 01.10.2019. The assessee has
submitted a certificate confirming the amount of interest.
HW Question: 11
Mrs. Ritvik has taken a loan on 01.11.2019 from PNB @ 10% p.a. of ₹10,00,000 for
purchase of one house which was purchased on 01.01.2020 and was self-occupied and
municipal taxes paid in previous year 2023-24 is ₹30,000. She has repaid the loan
amount in annual installments of ₹50,000 starting from 01.01.2021. The house was
vacant for 1 month in previous year 2023-24. She has submitted a certificate
confirming the amount of interest. She has short term capital gains under section 111A
₹10,00,000. Compute his total income for assessment year 2024-25 under old tax
regime and new tax regime.
Case: 4 Where a house property is let-out for part of the year and self-
occupied for part of the year [Section 23(3)]:
40 | P a g e www.hakkseca.com/ 783-892-5588
41
3. Income from house property
Illustration: 10
Mr. Lokesh constructed one house in 2022 and it is let out for 4 months and self-
occupied for 6 months and vacant for 2 months during previous year 2023-24. Municipal
valuation of the house is ₹40,000 p.m. and fair rent ₹30,000 p.m. Standard rent of the
house is ₹38,000 p.m. It was let out @ ₹32,000 p.m. Compute the GAV of the house.
Solution:
HW Question: 12
Mrs. Sharma has one house property at Indira Nagar in Bangalore. She stays with her
family in the house. The rent of similar property in the neighbourhood is ₹25,000 p.m.
The municipal valuation is ₹23,000 p.m. Municipal taxes paid is ₹8,000. The loan of
₹20,00,000 was taken on 01.01.2017 from SBI Housing Finance Ltd. The construction
was completed on 30.11.2019. The accumulated interest up to 31.03.2019 is ₹3,00,000.
During the previous year 2023-24, Mrs. Sharma paid ₹ 1,88,000 which included
₹1,44,000 as interest. Compute Mrs. Sharma’s income from house property for A.Y.
2024-25 assuming that she has not opted for default tax regime. All the conditions for
higher deduction of interest in case of self-occupied property is satisfied.
Illustration: 11
41 | P a g e www.hakkseca.com/ 783-892-5588
42
3. Income from house property
Mr. Sumit has 3 houses which are self-occupied and the details of these houses is as
under:
Calculation of Income for each house property considering all as deemed let-out:
42 | P a g e www.hakkseca.com/ 783-892-5588
43
3. Income from house property
Second Option is the best Income under the head House Property is ₹1,96,800.
Calculation of Income for each house property considering all as deemed let-out:
43 | P a g e www.hakkseca.com/ 783-892-5588
44
3. Income from house property
Second Option is the best Income under the head House Property is ₹3,96,800.
Case: 7 In case of a house property, a portion let out and a portion self-
occupied
Mrs. Manavi owns a house in Madras. During the previous year 2023-24, 2/3rd portion
of the house was self-occupied and 1/3rd portion was let out for residential purposes
at a rent of ₹8,000 p.m. Municipal value of the property is ₹3,00,000 p.a., fair rent is
₹2,70,000 p.a. and standard rent is ₹3,30,000 p.a. He paid municipal taxes @ 10% of
municipal value during the year. A loan of ₹25,00,000 was taken by him during the year
2019 for acquiring the property. Interest on loan paid during the previous year 2023-
24 was ₹1,20,000. Compute her income from house property for the A.Y. 2024-25
under old tax regime. All the conditions for higher deduction of interest in case of
self-occupied property is satisfied.
Solution:
There are two units of the house. Unit I with 2/3rd area is used by Mrs. Manavi for
self-occupation throughout the year and no benefit is derived from that unit, hence it
will be treated as self-occupied and its annual value will be nil.
Unit 2 with 1/3rd area is let-out through-out the previous year and its annual value has
to be determined as per section 23(1).
Computation of income from house property of Mrs. Manavi for A.Y. 2024-25
44 | P a g e www.hakkseca.com/ 783-892-5588
45
3. Income from house property
Illustration: 13
Compute gross annual value in the following cases for the assessment year 2024-25:
Solution:
Situation 1:
Computation of Gross Annual Value
(a) Fair Rent 1,32,000 (11,000 x 12)
(b) Municipal Valuation 1,44,000 (12,000 x 12)
(c) Higher of (a) or (b) 1,44,000
(d) Standard Rent 1,56,000 (13,000 x 12)
(e) Expected Rent {Lower of (c) or (d)} 1,44,000
(f) Rent Received/Receivable 88,000 (8,000 x 11)
GAV = Higher of (e) or (f) 1,44,000
Gross Annual Value 1,44,000
Situation 2:
Computation of Gross Annual Value
(a) Fair Rent 1,56,000 (13,000 x 12)
(b) Municipal Valuation 1,32,000 (11,000 x 12)
(c) Higher of (a) or (b) 1,56,000
(d) Standard Rent 1,44,000 (12,000 x 12)
(e) Expected Rent {Lower of (c) or (d)} 1,44,000
45 | P a g e www.hakkseca.com/ 783-892-5588
46
3. Income from house property
Situation 3:
Computation of Gross Annual Value
(a) Fair Rent 1,92,000 (16,000 x 12)
(b) Municipal Valuation 2,16,000 (18,000 x 12)
(c) Higher of (a) or (b) 2,16,000
(d) Standard Rent 2,04,000 (17,000 x 12)
(e) Expected Rent {Lower of (c) or (d)} 2,04,000
(f) Rent Received/Receivable 1,36,000 (17,000 x 8)
If there was no vacancy, in that case rent received/receivable would have been
₹17,000 x 11= ₹1,87,000 and It was still less than expected rent, therefore GAV shall
be expected rent. Gross Annual Value 2,04,000
Situation 4:
Computation of Gross Annual Value
(a) Fair Rent 1,68,000 (14,000 x 12)
(b) Municipal Valuation 1,08,000 (9,000 x 12)
(c) Higher of (a) or (b) 1,68,000
(d) Standard Rent 96,000 (8,000 x 12)
(e) Expected Rent {Lower of (c) or (d)} 96,000
(f) Rent Received/Receivable 1,68,000 (21,000 x 8)
In this case, rent R/R is higher than the expected rent, GAV shall be Rent R/R Gross
Annual Value 1,68,000
Mr. Ramesh has a property whose municipal valuation is ₹2,50,000 p.a. The fair rent is
₹2,00,000 p.a. and the standard rent fixed by the Rent Control Act is ₹2,10,000 p.a.
The property was let out for a rent of ₹20,000 p.m. However, the tenant vacated the
property on 31.01.2024. Unrealised rent was ₹20,000 and all conditions prescribed by
Rule 4 are satisfied. He paid municipal taxes @ 8% of municipal valuation. Interest on
borrowed capital was ₹1,65,000 for the year. He has casual income ₹3,00,000. Compute
his tax liability for A.Y. 2024-25 under regular provisions of the Act.
(a) Presume he is resident and his date of birth 01.04.1964
(b) Presume he is non-resident and his date of birth 01.04.1964
Solution:
46 | P a g e www.hakkseca.com/ 783-892-5588
47
3. Income from house property
Computation of income from house property and total income of Mr. Ramesh for
A.Y. 2024-25
(a) He is a resident:
(b) He is a non-resident
47 | P a g e www.hakkseca.com/ 783-892-5588
48
3. Income from house property
Mrs. Lokesh owns a house property at Adyar in Chennai. The municipal value of the
property is ₹5,00,000, fair rent is ₹4,20,000 and standard rent is ₹4,80,000. The
property was let-out for ₹50,000 p.m. up to December 2023. Thereafter, the tenant
vacated the property and Mrs. Lokesh used the house for self-occupation. Rent for the
months of November and December 2023 could not be realised in spite of the owner’s
efforts. All the conditions prescribed under Rule 4 are satisfied. She paid municipal
taxes @ 12% during the year. She had paid interest of ₹25,000 during the year for
amount borrowed for repairs for the house property. Compute Income u/h House
Property for A.Y. 2024-25.
HW Question: 14
Mr. Shyam has let out one house property to Mr. Laxman @ ₹ 80,000 p.m. Fair rent
₹90,000 p.m. Municipal valuation ₹80,000 p.m. and Standard rent of the house ₹76,000
p.m. The house remained vacant for 2 months and there was unrealised rent for 3
months. Mr. Shyam has paid municipal tax of ₹60,000 and interest on loan for
construction of house property is ₹69,000. Compute his Income Tax Liability for A. Y.
2024-25 under regular provisions of the Act and default tax regime.
Illustration: 15
Mr. Ashok has let out one house at ₹70,000 per month, fair rent ₹80,000 per month,
municipal valuation ₹60,000 per month, standard rent ₹ 65,000 per month. Municipal
tax paid ₹40,000, Interest u/s 24 (b) ₹50,000. Assessee has recovered unrealized
rent of ₹60,000 plus interest ₹7,000. He has incurred legal expenses ₹12,000. Compute
his Income and Tax Liability A. Y. 2024-25 under old tax regime.
Solution:
Computation of income under the head house property for A.Y. 2024-25
48 | P a g e www.hakkseca.com/ 783-892-5588
49
3. Income from house property
Working Note 1:
(a) Fair Rent (80,000 x 12) 9,60,000
(b) Municipal Valuation (60,000 x 12) 7,20,000
(c) Higher of (a) or (b) 9,60,000
(d) Standard Rent (65,000 x 12) 7,80,000
(e) Expected Rent {Lower of (c) or (d)} 7,80,000
(f) Rent received /receivable (70,000 x 12) 8,40,000
(g) Higher of (e) or (f) shall be GAV 8,40,000
Illustration: 16
Mr. Somesh has let out his house to State Bank @ ₹20,000 p.m. The bank has increased
the rent on 01.07.2023 to ₹27,000 p.m. retrospectively w.e.f. 01.11.2022. The assessee
has paid municipal taxes of ₹7,000 during the previous year 2023-24. Compute income
under the head House Property for assessment year 2024-25.
Solution:
Computation of income under the head House Property for A.Y. 2024-25
49 | P a g e www.hakkseca.com/ 783-892-5588
50
3. Income from house property
HW Question: 15
Mr. Mukesh owns a house property which is let out. During the previous year ending
31.03.2024 he receives (i) arrears of rent of ₹30,000 and (ii) unrealised rent of
₹20,000. You are requested to
(a) state, how they should be dealt with as per the provisions of the Act, and
(b) compute the income chargeable under the head “Income from house property”.
Mr. Binod sold his residential house property in March 2023. In June 2023, he
recovered rent of ₹10,000 from Mr. Gaurav, to whom he had let out his house for two
years from April 2017 to March 2019. He could not realise two month rent of ₹20,000
from him and to that extent his actual rent was reduced while computing income from
house property for A. Y. 2019-20. Further, he had let out his property from April, 2019
to February, 2023 to Mr. Satish. In April, 2021, he had increased the rent from
₹12,000 to ₹15,000 per month and the same was a subject matter of dispute. In
September 2023, the matter was finally settled and Mr. Binod received ₹69,000 as
arrears of rent for the period April 2021 to February, 2023. Would the recovery of
unrealised rent and arrears of rent be taxable in the hands of Mr. Binod, and if so in
which year?
50 | P a g e www.hakkseca.com/ 783-892-5588
51
3. Income from house property
Solution:
Computation of income from house property of Mr. Ankit for A.Y. 2024-25
Two brothers Anshul and Vimal are the co-owners of a house property with equal share.
The property was constructed during the financial year 1998-1999. The property
consists of 8 identical units and is situated at Cochin.
During the financial year 2023-24, each co-owner occupied 1 unit for residence and the
balance of 6 units were let out at a rent of ₹12,000 per month per unit. The municipal
value of the house property is ₹9,00,000 and the municipal taxes are 20% of municipal
value, which were paid during the year. The other expenses were as follows:
(i) Repairs ₹40,000
(ii) Insurance premium (paid) ₹15,000
(iii) Interest payable on loan taken for construction of house ₹3,00,000
51 | P a g e www.hakkseca.com/ 783-892-5588
52
3. Income from house property
One of the let-out units remained vacant for four months during the year. Anshul could
not occupy his unit for six months as he was transferred to Chennai. He does not own
any other house.
The other income of Mr. Anshul and Mr. Vimal are ₹2,90,000 and ₹1,80,000
respectively, for the financial year 2023-24.
Compute the income under the head ‘Income from House Property’ and the total income
of two brothers for the assessment year 2024-25 if they opt to shift out of default
tax regime.
Also, show the computation of income under this head, if they both exercised the
option under section 115BAC.
Illustration: 18
Mr. Kamlesh has let out one house along with generator facility and has charged a sum
of ₹40,000 p.m. as rent, out of which ₹3,000 p.m. is attributable to the generator. He
has paid ₹2,300 and the tenant has paid ₹900 towards municipal taxes. The interest on
the capital borrowed for construction of the house is ₹7,000. Mr. Kamlesh has paid
repair charge of the generator ₹3,400, fuel charges ₹5,600 and operator’s salary ₹300
p.m. Compute the total income of Mr. Kamlesh for assessment year 2024-25.
Solution:
Computation of income under the head House Property for A.Y. 2024-25
52 | P a g e www.hakkseca.com/ 783-892-5588
53
3. Income from house property
53 | P a g e www.hakkseca.com/ 783-892-5588
54
3. Income from house property
Comprehensive Questions
Question: 1 [IFHP]
Mr. Vijendra has constructed one house on 01.09.2023 and it was let out @ ₹1,25,000
p.m. and municipal taxes paid are ₹35,000. The house was constructed after taking a
loan from outside India and interest allowed under section 24(b) is ₹2,10,000, but the
assessee has not deducted tax at source. Compute Assessee’s tax liability for
assessment year 2024-25.
(b) Presume in the above question, the person who has given the loan has one agent in
India as per section 163. Compute tax liability for the assessment year 2024-25.
Mrs. Verma, a citizen of the U.S.A., is a resident and ordinarily resident in India during
the financial year 2023-24. She owns a house property at Los Angeles, U.S.A., which is
used as her residence. The annual value of the house is $20,000. The value of one USD
($) may be taken as ₹ 65. She took ownership and possession of a flat in Chennai on
1.7.2023, which is used for self-occupation, while she is in India. The flat was used by
her for 7 months only during the year ended 31.3.2024. The municipal valuation is
₹32,000 p.m. and the fair rent is ₹4,20,000 p.a. She paid the following to Corporation
of Chennai:
Property Tax ₹16,200
Sewerage Tax ₹1,800
She had taken a loan from Standard Chartered Bank in June, 2021 for purchasing this
flat. Interest on loan was as under:
Period prior to 1.4.2023 ₹49,200
1.4.2023 to 30.6.2023 ₹50,800
1.7.2023 to 31.3.2024 ₹1,31,300
Certificate confirming the amount of Interest has been deposited. She had a house
property in Bangalore, which was sold in March, 2019. In respect of this house, she
received arrears of rent of ₹60,000 in March, 2024. This amount has not been charged
to tax earlier. Compute the income chargeable from house property of Mrs. Verma for
the assessment year 2024-25 assuming that she has exercised the option of shifting
out of the default tax regime u/s. 115BAC(1A).
Would your answer change if she pays tax under the default tax regime under section
115BAC?
Question: 3 [IFHP]
Mr. Vidhit has let out one house property @ ₹70,000 per month and there is unrealised
Rent of 2 months and there is vacancy of 3 month. Fair rent ₹60,000 per month,
54 | P a g e www.hakkseca.com/ 783-892-5588
55
3. Income from house property
municipal valuation ₹55,000 per month and standard rent ₹80,000 per month. Municipal
tax paid ₹62,000. Interest on loan for construction of the house property is
₹75,000.The assessee has unrealised Rent of ₹2,00,000 in P.Y. 2021-22 and he has
recovered ₹1,50,000 in P.Y. 2023-24 and interest of ₹18,000 and he has incurred
₹11,000 as legal expense. Compute his tax liability for assessment year 2024-25
assuming that he shifts out of default tax regime.
Question: 4 [IFHP]
Mr. Rajeev has let out one house @ ₹45,000 p.m., but this house was vacated on
01.11.2023. The house was self-occupied w.e.f. 01.01.2024. Fair rent of this house is
₹50,000 p.m., municipal valuation is ₹47,000 p.m. and standard rent is ₹48,000 p.m. The
assessee has paid municipal taxes @ 10% of municipal valuation. Interest on capital
borrowed is ₹42,000. Land revenue paid by the assessee is ₹11,000 and ground rent
paid by him is ₹3,000. The assessee has taken a loan for payment of municipal tax and
interest paid on loan is ₹500. Compute his income under the head house property and
tax liability for assessment year 2024-25 assuming that he opts for default tax
regime.
Question: 5 [IFHP]
Mr. Raju and Mr. Ramu constructed their houses on a piece of land purchased by them
at New Delhi. The built-up area of each house was 1,000 sq. ft. ground floor and an
equal area at the first floor. Mr. Raju started construction of the house on 01.04.2022
and completed it on 31.03.2023. Mr. Raju occupied the entire house on 01.04.2023. Mr.
Raju has availed a housing loan of ₹25 lakhs @ 12% p.a. on 01.04.2022 and has also
submitted a certificate from the lender certifying the amount of interest. Mr. Ramu
started construction on 01.04.2022 and completed it on 30.06.2023. Mr. Ramu occupied
the ground floor on 01.07.2023 and let out the first floor for a rent of ₹20,000 per
month. However, the tenant vacated the house on 31.12.2023 and Mr. Ramu occupied
the entire house during the period 01.01.2024 to 31.03.2024. Mr. Ramu has availed a
housing loan of ₹15 lakhs @ 10% p.a. on 01.07.2022 and has also submitted a certificate
from the lender certifying the amount of interest. Following are the other information:
(i) Fair rental value of each unit 1,20,000 Per annum (Ground floor / first floor)
(ii) Municipal value of each unit 92,000 Per annum (Ground floor / first floor)
(iii) Municipal taxes paid by Raju - 10,000, Ramu - 10,000
(iv) Repair and maintenance charges paid by Raju - 30,000, Ramu - 32,000
No repayment was made by either of them till 31.03.2024. Compute income from house
property for Mr. Raju and Mr. Ramu for the previous year 2023-24 (assessment year
2024-25) under regular provisions of the Act.
55 | P a g e www.hakkseca.com/ 783-892-5588
56
3. Income from house property
Question: 6 [IFHP]
Mr. Saurabh has a house property situated in Mumbai which has two units. Unit I has a
floor area of 70% whereas the unit II has a floor area of 30%. Both the units were
self-occupied by the assessee. As the assessee was allowed a rent free accommodation
by his employer w.e.f. 01.04.2023, he vacated both of the units and let out unit I at a
rent of ₹13,000 p.m. and unit II for ₹5,000 p.m. Unit I remained vacant for 1½ months
whereas unit II was vacant for one month. Other particulars of the house property are
as under:
Municipal Valuation 1,55,000
Fair Rent 1,75,000
Standard Rent 1,65,000
Municipal taxes paid 35,000
Ground rent due 15,000
Compute income from house property for the assessment year 2024-25.
Mr. Anand constructed a shopping complex. He had taken a loan of ₹25 lakhs for
construction of the said property on 01.08.2021 from SBI @ 10% for 5 years. The
construction was completed on 30.06.2022. Rental income received from shopping
complex ₹30,000 per month let out for the whole year. Municipal Taxes paid for
shopping complex ₹8,000. Arrears of rent received from shopping complex ₹1,20,000.
Interest paid on loan taken from SBI for purchase of house for use as own residence
for the period 2023- 2024 of ₹3 lakhs. The loan was taken after 01.04.1999 and house
was purchased within 5 years from the end of the year in which loan was taken and
assessee has submitted certificate certifying the amount of interest. You are required
to compute Income from House property of Mr. Anand for A.Y. 2024-2025 as per
Income Tax Act, 1961 under regular provisions of the Act.
Mr. Mukesh constructed one house in 2022 and it is let out for 4 months and self-
occupied for 6 months and vacant for 2 months during previous year 2023-24. Municipal
valuation of the house is ₹40,000 p.m. and fair rent ₹30,000 p.m. Standard rent of the
house is ₹38,000 p.m. It was let out @ ₹32,000 p.m. Municipal tax levied is ₹6,000 out
of which ₹2,000 was paid by the tenant and ₹2,000 by the assessee and balance ₹2,000
yet to be paid. Interest on the capital borrowed for construction of the house is
₹30,000. Long Term Capital Gains is ₹2,10,000 Compute his income and tax Liability for
the assessment year 2024-25 under regular provisions of the Act.
56 | P a g e www.hakkseca.com/ 783-892-5588
57
3. Income from house property
Mr. Umesh has one big house. 25% of it is being used by the assessee in his own
business/ profession and 50% of the house is let out @ ₹10,000 p.m. However, it
remained vacant for one month and there is unrealised rent for 1½ month. Remaining
25% is self-occupied throughout the year. Fair rent of the entire house is ₹25,000
p.m., municipal valuation ₹22,000 p.m. and municipal tax paid is ₹22,000. Insurance
premium paid is ₹6,000, repair charges ₹8,000, land revenue paid ₹4,000, ground rent
is ₹3,000 and depreciation of the house is ₹12,000. Assessee’s income under the head
business/profession before charging expenditure relating to house property is
₹2,00,000. Compute his total income and tax liability for assessment year 2024-25
under regular provisions of the Act.
Question: 10 [IFHP, PGBP] JULY – 2021 (NEW COURSE) Question 3(a) (6 Marks)
Mr. Ramlal constructed a big house (construction completed in Previous Year 2013-
2014) with 3 independent units. Unit-1 (50% of floor area) is let out for residential
purpose at monthly rent of ₹15,000. A sum of 3,000 could not be collected from the
tenant and a notice to vacate the unit was given to the tenant. No other property of
Mr. Ramlal is occupied by the tenant. Unit- 1 remains vacant for 2 months when it is not
put to any use. Unit – 2 (25% of the floor area) is used by Mr. Ramlal for the purpose
of his business, while Unit – 3 (the remaining 25%) is utilized for the purpose of his
residence. Other particulars of the house are as follows:
Municipal valuation - ₹ 1,88,000,
fair rent - ₹ 2,48,000,
Standard rent under the Rent Control Act - ₹ 2,28,000,
Municipal taxes - ₹ 20,000,
repairs - ₹ 5,000,
Interest on capital borrowed for the construction of the property - ₹ 60,000,
Ground rent – 6,000 and
Fire the insurance premium paid - ₹60,000.
Income of Ramlal from the business is ₹ 1,40,000 (without debiting house rent and
other incidental expenditure).
Determine the taxable income of Mr. Ramlal for the assessment year 2024-25 if he
does not opt to be taxed under section 115BAC.
Mr. Avinas owns two house properties one at Bomaby, wherein his family resides and
the other at Delhi, which is unoccupied. He lives in Chandigarh for his employment
purposes in a rented house. For acquisition of house property at Bombay, he has taken a
loan of Rs 30 lakh @ 10% p.a. on 01.04.2022. He has not repaid any amount so far. In
respect of house property at Delhi, he has taken a loan of Rs. 5 lakhs @11% p.a. on
57 | P a g e www.hakkseca.com/ 783-892-5588
58
3. Income from house property
1.10.2022 towards repairs. Compute the deduction which would be available to him
under section 24(b) for A. Y. 2024-25 in respect of interest payable on such loan if he
exercises the option of shifting out of the default tax regime provided under section
115BAC(1A).
Shiv has three houses all of which are self–occupied. The particulars of the houses for
the P. Y. 2023-24 are as under-
Compute Shiv’s income from house property for A. Y. 2024-25 and suggest which
houses should be opted by Shiv to be assessed as self-occupied so that his tax liability
is minimum.
Ms. Anjali co-owns a residential house property in Calcutta along with her sister Ms.
Charu, where her sister’s family resides. Both of them have equal share in the property
and the same is used by them for self- occupation. Interest is payable in respect of
loan of Rs. 50,00,000 @ 10% taken on 01.04.2022 for acquisition of such property. In
addition, Ms. Anjali owns a flat in Pune in which she and her parents reside. She has
taken a loan of Rs. 3,00,000 @ 12% on 01.10.2022 for repairs of this flat. Compute the
deduction which would be available to Ms. Anjali and Ms. Charu under section 24(b) for
A. Y. 2024-25, if both exercise the option of shifting out of the default tax regime
provided under section 115BAC(1A).
Mr. Vikas owns a house property whose Municipal Value, Fair Rent and Standard Rent
are Rs. 96,000, Rs. 1,26,000 and Rs. 1,08,000 (per annum), respectively. During the F. Y.
58 | P a g e www.hakkseca.com/ 783-892-5588
59
3. Income from house property
2023-24, one–third of the portion of the house was let out for residential purpose at a
monthly rent of Rs. 5,000. The remaining two–third portion was self-occupied by him.
Municipal tax @ 11% of municipal value was paid during the year.
The construction of the house began in June, 2016 and was completed on 31.05.2019.
Vikas took a loan of Rs. 1,00,000 on 01.07.2016 for the construction of building. He paid
interest on loan @12% per annum and every month such interest was paid.
Compute income from house property of Mr. Vikas for the A. Y. 2024-25 if he has
exercised the option of shifting out of the default tax regime provided under section
115BAC(1A).
59 | P a g e www.hakkseca.com/ 783-892-5588
60
4. Income under head Capital Gains
Illustration: 1
Mr. Anshul, aged 40 years sold an agricultural land for ₹56 lakhs on 14.11.2023 acquired
at a cost of ₹49 lakhs on 23.10.2022 situated at 8 kms from the jurisdiction of
municipality having population of 4,00,000 and also sold another agricultural land for
₹63 lakhs on 22.01.2024 acquired at a cost of ₹49 lakhs on 15.03.2022 situated at 1
kms from the jurisdiction of municipality having population of 11,000. Whether the
agricultural land of Mr. Anshul will be considered as capital asset in the assessment
year 2024-25?
Solution:
As per the definition of capital asset, a land shall be considered to be urban land if it is
situated within 2 kms from the limits of urban area having population more than 10,000
but not exceeding 1,00,000 and within 6 kms from the limits of urban area having
population more than 1,00,000 but not exceeding 10,00,000. In the given case, first
agricultural land is situated after 6 kms having population of 4,00,000 hence such land
is considered as rural agricultural land and second land is situated within 2 kms from
the municipality having population of 11,000 hence such land is considered as
agricultural land in urban area and considered as capital asset.
HW Question: 1
60 | P a g e www.hakkseca.com/ 783-892-5588
61
4. Income under head Capital Gains
Illustration: 2
Solution:
Tax on Short term capital gains ₹7,00,000 @ 15% u/s 111A 1,05,000
Tax on normal income 3,13,000 @ slab rate 3,150
Tax before health & education cess 1,08,150
Add: HEC @ 4% 4,326
Tax Liability 1,12,476
Rounded off u/s. 288B 1,12,480
61 | P a g e www.hakkseca.com/ 783-892-5588
62
4. Income under head Capital Gains
Tax on Short term capital gains ₹7,00,000 @ 15% u/s 111A 1,05,000
Tax on normal income 3,13,000 @ slab rate 650
Tax before health & education cess 1,05,650
Add: HEC @ 4% 4,226
Tax Liability 1,09,876
Rounded off u/s. 288B 1,09,880
Tax on Short term capital gains (₹7,00,000-₹1,87,000) @ 15% u/s 111A 76,950
Tax on normal income 3,13,000 @ slab rate Nil
Tax before health & education cess 76,950
Add: HEC @ 4% 3,078
Tax Liability 80,028
Rounded off u/s. 288B 80,030
HW Question: 2
Compute tax liability for the assessment year 2024-25 in the following situations under
normal provisions of the Act:
(i) Mr. Rohan (aged 45 years) is resident in India and has income under the head house
property of ₹40,000 and income under the head salary of ₹30,000 and short term
capital gains from sale of listed equity shares of ₹4,80,000 (STT Paid).
(ii) Presume in the above situation the assessee is Mrs. Rohan.
(iii) Presume in the above situation the assessee is Mrs. Rohan and she is aged about 70
years.
(iv) Presume in the above situation the assessee is Mr. Rohan and he is aged about 70
years.
(v) Presume in the above situation the assessee is Mrs. Rohan and she is aged about 85
years.
(vi) Presume in the above situation the assessee is Mr. Rohan and he is aged about 85
years.
Illustration: 3
Compute tax liability in following cases under normal provisions of the Act:
62 | P a g e www.hakkseca.com/ 783-892-5588
63
4. Income under head Capital Gains
(i) Mr. Harsha (resident aged 23 years) having PGBP income of ₹4,00,000, Salary of
₹5,00,000, Short term capital gain on sale of unlisted shares of ₹5,00,000, Short term
capital gain of sale of equity oriented mutual fund (STT paid) of ₹8,00,000. [Ans:
3,66,600]
(ii) Mr. Druv (resident aged 62 years) having Salary of ₹12,00,000, STCG u/s. 111A of
₹4,00,000, STCG of ₹3,00,000 and other income of ₹2,00,000 [Ans: 3,95,200]
(iii) Miss Dhawani (non-resident aged 82 years) having Salary of ₹1,00,000, STCG of
₹1,00,000 and STCG u/s. 111A of ₹2,90,000. [Ans: 45,240]
(iv) Mr. Ritik (resident aged 82 years) having PGBP income of ₹3,00,000, STCG of
₹50,000 and STCG u/s. 111A of ₹4,00,000. [Ans: 39,000]
(v) Mr. Kulkarni (resident aged 19 years) having PGBP of ₹20,00,000, STCG u/s. 111A of
₹25,00,000 and STCG (normal) of ₹15,00,000. [Ans: 14,15,700]
Illustration: 4
Compute tax liability in following cases assuming that the assessee shifts out of
default tax regime:
(i) Mr. Vikram (resident aged 23 years) having PGBP income of ₹4,00,000, Salary of
₹5,00,000, Short term capital gain on sale of unlisted shares of ₹5,00,000, Short term
capital gain of sale of equity oriented mutual fund (STT Paid) of ₹8,00,000 and long
term capital gain u/s. 112A of ₹7,00,000. [Ans: 4,29,000]
(ii) Mr. Aman (resident aged 62 years) having Salary of ₹12,00,000, STCG u/s. 111A of
₹4,00,000, STCG of ₹3,00,000, long term capital gain on sale of equity oriented mutual
fund of ₹3,00,000 and other income of ₹2,00,000. [Ans: 4,16,000]
(iii) Miss Shazia (non-resident aged 82 years) having Salary of ₹1,00,000, STCG of
₹1,00,000, LTCG u/s. 112A of ₹5,00,000 and STCG u/s. 111A of ₹2,90,000. [Ans:
86,840]
(iv) Mr. Sambhav (resident aged 82 years) having PGBP income of ₹3,00,000, STCG of
₹50,000, LTCG u/s. 112A of ₹50,000 and STCG u/s. 111A of ₹4,00,000. [Ans: 39,000]
63 | P a g e www.hakkseca.com/ 783-892-5588
64
4. Income under head Capital Gains
(v) Mr. Jeet (resident aged 19 years) having PGBP of ₹20,00,000, STCG u/s. 111A of
₹25,00,000, LTCG u/s. 112A of 10,00,000 and STCG (normal) of ₹15,00,000. [Ans:
15,18,660]
(vi) Mr. Ranvijay (aged 59 years) has LTCG u/s. 112A of ₹50,50,000. [Ans: 5,35,600]
(vii) Mr. Kohli (aged 35 years) has LTCG u/s. 112A of ₹51,00,000. [Ans: 5,43,400]
HW Question: 3
Compute tax liability in following cases assuming that the assessee shifts out of
the default tax regime:
Case (i) Mr. Raman has income under the head house property of ₹5,00,000 and LTCG
u/s. 112A of ₹1,00,000.
Case (ii) Mr. Raman has LTCG 112A ₹50,50,000.
Case (iii) Mr. Raman has LTCG 112A ₹51,00,000.
Illustration: 5
Compute tax liability in the following cases assuming that he shifts out of the
default tax regime:
(i) Mr. Abhishek a resident has long term capital gains ₹3,50,000.
(ii) Mr. Abhishek a resident has short term capital gains u/s 111A ₹3,50,000.
(iii) Mr. Abhishek a non-resident has long term capital gains ₹3,50,000.
(iv) Mr. Abhishek a non-resident has short term capital gains u/s 111A ₹3,50,000.
(v) Mr. Abhishek a non-resident aged 61 years has long term capital gains ₹3,50,000.
(vi) Mr. Abhishek a non-resident aged 61 years has short term capital gains u/s 111A
₹3,50,000.
Solution:
64 | P a g e www.hakkseca.com/ 783-892-5588
65
4. Income under head Capital Gains
Illustration: 6
Compute tax liability of Mr. Kishan when he has following incomes under normal
provisions of the Act:
a) Income from business in chennai of ₹5,50,000
b) STCG on sale of land in Delhi of ₹8,00,000
c) STCG on sale of listed euiqty shares (STT paid at the time of transfer) of
₹6,00,000
d) LTCG on sale of listed equity shares (STT paid only at the time of transfer) of
₹4,00,000
e) LTCG on sale of unlisted equity shares of ₹2,00,000
f) Total deductions under chapter VIA is ₹6,00,000
[Ans: ₹2,83,400]
HW Question: 4
65 | P a g e www.hakkseca.com/ 783-892-5588
66
4. Income under head Capital Gains
Compute tax liability for the assessment year 2024-25 in the following situations
under old tax regime:
(i) Mr. Shyam is resident in India and has income under the head house property
₹40,000 and income under the head salary ₹30,000 and long term capital gains
₹4,80,000.
(ii) Presume in the above situation the assessee is Mrs. Shyam and she is aged about 70
years.
(iii) Presume in the above situation the assessee is Mrs. Shyam and she is aged about
85 years.
(iv) Presume in the above situation (iii), the assessee is non-resident in India.
Illustration: 7
Case (i) Mr. Rohan has PGBP income of ₹20,00,000 and LTCG u/s. 112 of ₹25,00,000.
[Ans: 9,49,000]
Case (ii) Mr. Rohan has Salary income (computed) of ₹30,00,000, STCG u/s. 111A of
₹40,00,000 and LTCG u/s. 112A of ₹30,00,000. [Ans: 18,33,260]
Case (iii) Mr. Rohan has income under head House Property of ₹50,00,000, STCG of
₹40,00,000 and LTCG u/s. 112 of ₹60,00,000. [Ans: 44,40,150]
Case (iv) Mr. Rohan has other income of ₹1,50,00,000, PGBP income of ₹60,00,000 and
LTCG u/s. 112A of ₹30,00,000. [Ans: 82,93,090]
Case (v) Mr. Rohan has PGBP income of ₹6,00,00,000 and STCG u/s. 111A of
₹10,00,000. [Ans: 2,55,58,650]
Case (vi) Mr. Rohan has Salary income of ₹1,40,00,000 and LTCG u/s. 112 of
₹70,00,000. [Ans: 64,73,350]
Illustration: 8
Solution:
66 | P a g e www.hakkseca.com/ 783-892-5588
67
4. Income under head Capital Gains
Illustration: 9
Solution:
67 | P a g e www.hakkseca.com/ 783-892-5588
68
4. Income under head Capital Gains
Option 2: Taking Dividend tax @ 30% for surcharge and PGBP income on slab rate
68 | P a g e www.hakkseca.com/ 783-892-5588
69
4. Income under head Capital Gains
HW Question: 5
HW Question: 6
Mr. Sanjay has income from business ₹203 lakhs and short term capital gain under
section 111A ₹30 lakhs and long term capital gains under section 112A ₹41 lakhs.
Calculate his tax liability under old tax regime.
HW Question: 7
Mr. Vikrant has income from business of ₹501 lakhs and short term capital gain under
section 111A of ₹30 lakhs and long term capital gains under section 112A of ₹41 lakhs.
Calculate his tax liability under old tax regime.
HW Question: 8
69 | P a g e www.hakkseca.com/ 783-892-5588
70
4. Income under head Capital Gains
Mr. Vikas has income from business of ₹550 lakhs and short term capital gain under
section 111A of ₹30 lakhs and long term capital gains under section 112A of ₹41 lakhs.
Compute his tax liability under old tax regime.
HW Question: 9
Illustration: 10
Calculate income under head capital gain and tax liability in following cases under
normal tax provisions:
Case (i) Mr. Sahil, resident aged 68 years, has purchased one house property on
01.04.2011 for ₹10,00,000. He sold the property for ₹40,00,000 on 02.02.2024. [Ans:
21,08,700, 3,76,210]
Case (ii) Mr. Manoj, resident aged 45 years, has purchased 1,00,000 shares of a listed
company (STT paid) @ ₹30 each on 01.04.2022 and sold 50,000 shares on 03.04.2023
for ₹40 each and 50,000 shares on 04.05.2023 for ₹25 each (STT paid). [Ans:
2,50,000, 0]
Case (iii) Mr. Ashok, non-resident aged 80 years, has purchased 50,000 shares of
unlisted company @ ₹100 each on 01.06.2022 and sold 40,000 shares on 01.02.2024 for
₹120 each. [Ans: 8,00,000, 75,400]
Case (iv) Mr. Ritik, resident aged 20 years, purchased unlisted equity oriented mutual
funds on 01.05.2022 for ₹50,00,000 and sold such units on 01.06.2023 for ₹75,00,000.
[Ans: 22,43,200, 4,14,650]
HW Question: 10
Mr. Rohit purchased one house on 01.07.2005 for ₹3,50,000. He constructed its first
floor on 01.10.2014 by incurring ₹4,00,000 and constructed its second floor on
01.10.2015 by incurring ₹6,00,000 and third floor on 01.10.2017 by incurring ₹7,00,000.
Finally, sold the building on 01.01.2024 for ₹1,20,00,000 and selling expenses were 2%
70 | P a g e www.hakkseca.com/ 783-892-5588
71
4. Income under head Capital Gains
of the sale price. He has deposited ₹1,00,000 in NSC. Compute tax liability of the
assessee for the assessment year 2024-25 assuming that he shifts out of default tax
regime under section 115BAC.
Illustration: 11
# Nature Date & Date & Fair market Stamp duty Cost of
of Purchase Improvement value as on value as on acquisition
property price cost 01.04.2001 01.04.2001
1. Shares 01.04.1995 NA 12,00,000 NA
of XYZ 10,00,000
Ltd.
2. Land 01.04.1980 01.05.1999 45,00,000 40,00,000
30,50,000 10,00,000
3. Building 01.05.1970 01.04.2002 70,00,000 75,00,000
40,00,000 10,50,000
4. Villa 01.06.1997 01.03.2000 40,00,000 45,00,000
50,60,000 4,00,000
5. Bonus 01.04.1999 NA 3,50,000 NA
shares Nil
Illustration: 12
Compute capital gains and tax liability of Mr. Laxman for the assessment year
2024-25 under normal provisions of the Act:
Asset Gold Land Residential
House
Date of purchase 01.07.1992 01.04.1995 01.07.1997
Cost price 4,00,000 6,00,000 8,00,000
Cost of improvement 1,00,000 2,00,000 4,00,000
Year of improvement 1999-2000 2000-01 2005-06
Fair market value on
01.04.2001 30,00,000 60,00,000 5,00,000
Date of Sale 01.02.2024 01.03.2024 01.01.2024
Full value of consideration 2,00,00,000 3,00,00,000 4,00,00,000
71 | P a g e www.hakkseca.com/ 783-892-5588
72
4. Income under head Capital Gains
Solution:
Gold
Full value of consideration 2,00,00,000
Less: Indexed cost of acquisition
= 30,00,000 / Index of 01-02 x Index of 23-24
= 30,00,000 / 100 x 348 (1,04,40,000)
Long term capital gain 95,60,000
Land
Full value of consideration 3,00,00,000
Less: Indexed cost of acquisition
= 60,00,000 / Index of 01-02 x Index of 23-24
= 60,00,000 / 100 x 348 (2,08,80,000)
Long term capital gain 91,20,000
Residential House
Full value of consideration 4,00,00,000
Less: Indexed cost of acquisition
= 8,00,000 / Index of 01-02 x Index of 23-24
= 8,00,000 / 100 x 348 (27,84,000)
Less: Indexed cost of improvement
= 4,00,000 / Index of 05-06 x Index of 23-24
= 4,00,000 / 117 x 348 (11,89,744)
Long term capital gains 3,60,26,256
Income under head capital gains 5,47,06,256
72 | P a g e www.hakkseca.com/ 783-892-5588
73
4. Income under head Capital Gains
HW Question: 11
Mr. Aman acquired a residential house in January, 1999 for ₹2,00,000 and its market
value on 01.04.2001 is ₹1,80,000 and he constructed its 1st floor in September’ 2007
by incurring ₹3,00,000 and constructed second floor in October’ 2011 by incurring
₹4,00,000 and constructed its third floor in February’ 2013 by incurring ₹5,00,000 and
sold the house on 01.01.2024 for ₹1,00,00,000 and paid brokerage @ 1% and he
invested ₹20,000 in equity shares of infrastructure development company notified
under section 80C. Compute his tax liability for assessment year 2024-25 under normal
provisions of the Act.
HW Question: 12
Compute capital gains and tax liability of Mr. Anmol in the following Independent
situations for the assessment year 2024-25 under normal provisions of the Act:
Asset Gold Land Residential house
Date of purchase 01.07.1998 01.04.1996 01.07.1993
Cost price 3,00,000 5,00,000 7,00,000
Cost of improvement 20,000 1,00,000 3,00,000
Year of improvement 1999-00 2000-01 2016-17
Fair market value on 01.04.2001 35,00,000 45,00,000 55,00,000
Date of Sale 01.01.2024 01.01.2024 01.01.2024
Full value of consideration 150,00,000 320,00,000 400,00,000
Illustration: 13
Mr. Ameesh holds 500 shares of Aabra Ka Dabra Ltd. which were allotted to him on
22.04.2002 @ ₹30 per share. On 22.07.2023, Aabra Ka Dabra Ltd. made right issue to
the existing shareholders at the rate of one share for every five shares held @ ₹20
per share. Mr. Ameesh instead of exercising his rights to obtain right shares, has
exercised his right of renouncement by renouncing the said right entitlement in favour
of Mr. Nameesh @ ₹13 per right share entitlement on 04.08.2023.
(a) Determine the nature and amount of capital gain, if any, taxable in the hands of Mr.
Ameesh.
(b) What will be the cost of acquisition of shares purchased by Mr. Nameesh?
Solution:
73 | P a g e www.hakkseca.com/ 783-892-5588
74
4. Income under head Capital Gains
HW Question: 13
Mr. Soham is a shareholder of Somani Ltd. holding 1,000 shares of the face value of
₹10 each. The company made a right issue in the ratio of 1:1 on 01.01.2024 at a premium
of ₹50 per share. He renounced it in favour of Mr. Mohan at a price of ₹10 per share.
What is the capital gain chargeable in the hands of Mr. Soham? What will be the cost
of the shares in the hands of Mr. Mohan?
Illustration: 14
Mr. Pawan purchased 100 equity shares in Mambani Ltd. on 01.10.1996 @ ₹10 per share.
The company has issued 100 bonus shares on 01.10.1998 and market value of the shares
on 01.04.2001 was ₹7 per share. The company has again issued 100 bonus shares on
01.10.2013.
The company has offered 100 right shares on 01.04.2023 @ ₹140 per share though the
market value is ₹250 per share. Mr. Pawan purchased half of the shares and remaining
half were renounced by him in favour of his friend Mr. Jagmal. He has charged ₹20 per
share from Mr. Jagmal for renouncing the right.
All the shares were sold by Mr. Pawan and Mr. Jagmal @ ₹300 per share on 01.01.2024.
Mr. Pawan has income under the head house property ₹2,20,000 and has invested
₹1,00,000 in NSC.
Mr. Jagmal has income under the head house property ₹3,50,000 and has invested
₹30,000 in NSC.
Compute tax liability of Mr. Pawan and Mr. Jagmal (No STT was paid) if they opts out
of default has regime under section 115BAC.
Solution:
Original Shares
Full value of consideration (100 x 300) 30,000
Less: Indexed Cost of Acquisition (100 x 10)/100 x 348 (3,480)
Long Term Capital Gain u/s. 112 26,520
74 | P a g e www.hakkseca.com/ 783-892-5588
75
4. Income under head Capital Gains
Right Shares
Full value of consideration (50 x 300) 15,000
Less: Cost of Acquisition (50 x 140) (7,000)
Short Term Capital Gain 8,000
75 | P a g e www.hakkseca.com/ 783-892-5588
76
4. Income under head Capital Gains
TP: 11 Cost of acquisition where long term capital u/s. 112A are
acquired before 01.02.2018
Illustration: 15
Mr. Ashish purchased 100 equity shares in ABC Ltd. on 01.10.1995 @ ₹10 per share. The
company has issued 100 bonus shares on 01.10.1998 and market value of the shares on
01.04.2001 was ₹7 per share. The company has again issued 100 bonus shares on
01.10.2013.
The company has offered 100 right shares on 01.04.2023 @ ₹140 per share though the
market value is ₹250 per share. Mr. Ashish purchased half of the shares and remaining
half were renounced by him in favour of his friend Mr. Manish. He has charged ₹20 per
share from Mr. Manish for renouncing the right.
All the shares were sold by Mr. Ashish and Mr. Manish @ ₹300 per share on 01.01.2024
and securities transaction tax has been paid. (market value on 31-01-2018 is ₹200 per
share)
Mr. Ashish has income under the head house property ₹2,20,000 and has invested
₹1,00,000 in NSC.
Mr. Manish has income under the head house property ₹3,50,000 and has invested
₹30,000 in NSC.
Compute tax liability of Mr. Ashish and Mr. Manish under old tax regime.
Solution:
Original Shares
76 | P a g e www.hakkseca.com/ 783-892-5588
77
4. Income under head Capital Gains
Right Shares
Full value of consideration (50 x 300) 15,000
Less: Cost of Acquisition (50 x 140) (7,000)
Short Term Capital Gain u/s 111A 8,000
77 | P a g e www.hakkseca.com/ 783-892-5588
78
4. Income under head Capital Gains
HW Question: 14
Mr. Sunil purchased 100 equity shares in ABC Ltd. (listed) on 01.10.1996 @ ₹10 per
share. The company had issued 100 bonus shares on 01.10.2000 and market value of the
share as on 01.04.2001 is ₹8 per share.
Company has again issued 100 bonus shares on 01.10.2006.
The company has further offered 100 right shares on 01.05.2023 @ ₹150 per share
and Mr. Sunil has purchased half of the shares and balance half was renounced in
favour of Mr. Jeet by charging ₹5 per share.
78 | P a g e www.hakkseca.com/ 783-892-5588
79
4. Income under head Capital Gains
Mr. Sunil and Mr. Jeet both have transferred all the shares on 01.01.2024 @ ₹200 per
share and securities transaction tax has been paid. Market value as on 31-01-2018 ₹100
per share.
Mr. Sunil has income under the head business/profession ₹20,00,000 and he has
invested ₹70,000 in public provident fund.
Mr. Jeet has income under the head business/profession ₹10,00,000 and he has
invested ₹50,000 in public provident fund.
Compute tax liability of Mr. Sunil and Mr. Jeet assuming that he shifts out of default
tax regime under section 115BAC.
HW Question: 15
Mrs. Kamla purchases 1,000 equity shares in Adani Ltd. at a cost of ₹ 15 per share
(brokerage 1%) in January 1998. She gets 100 bonus shares in August 2000. She again
gets 1100 bonus shares by virtue of her holding on February 2005. Fair market value of
the shares of Adani Ltd. On April 1, 2001 is ₹25. In January 2024, she transfers all
her shares @ ₹ 120 per share (brokerage 2%). (market value on 31-01-2018 is ₹70 per
share).
Compute the capital gains taxable in the hands of Mrs. Kamla for the A.Y. 2024-25
assuming:
(a) Adani Ltd. is an unlisted company and securities transaction tax was not applicable
at the time of sale.
(b) Adani Ltd. is a listed company and the shares are sold in a recognised stock
exchange and securities transaction tax was paid at the time of sale.
Mr. Arvind holding 1000 shares of Logistic Ltd acquired on 01.07.2022 for ₹600 per
share, sold 500 shares to Mr. Sunil, on 01.05.2023 for ₹550 per share. Logistic Ltd.
declared dividend @ ₹65 per share on 20.07.2023, being the record date for
declaration of dividend. Mr. Sunil sold 300 equity shares at ₹475 per share on
28.09.2023 and the balance 200 equity shares at ₹450 per share on 28.10.2023. Apart
from above mentioned information, Mr. Sunil was having only long-term capital gains
from sale of unlisted shares of ₹50,000. Compute his total income for A.Y. 2024-25.
Illustration: 16
79 | P a g e www.hakkseca.com/ 783-892-5588
80
4. Income under head Capital Gains
Mr. Ramit & sons, HUF, purchased a land for ₹40,000 in 2001-02. In 2005-06, a
partition takes place when Mr. Rohit, a coparcener, is allotted this plot valued at
₹2,00,000. In 2006-07, he had incurred expenses of ₹1,85,000 towards fencing of the
plot. Mr. Rohit sells this plot of land for ₹15,00,000 in 2023-24 after incurring
expenses of ₹20,000. You are required to compute the capital gain for the A.Y. 2024-
25.
Solution:
Illustration: 17
Mr. Jatin purchased one house on 01.10.1998 for ₹2,00,000 and incurred ₹1,00,000 on
its improvement on 01.10.1999. Its fair market value on 01.04.2001 is ₹4,50,000.
Mr. Jatin expired on 01.05.2006 and the house was inherited by his son Mr. Jenil and
value for the purpose of charging stamp duty was ₹10,00,000.
Mr. Jenil has sold the house on 01.11.2023 for ₹72,00,000.
Compute tax liability of Mr. Jenil for the assessment year 2024-25 under old tax
regime.
Solution:
80 | P a g e www.hakkseca.com/ 783-892-5588
81
4. Income under head Capital Gains
Illustration: 18
Manish held 2000 shares in a company ABC Ltd. an Indian company. This company
amalgamated with another Indian company XYZ Ltd. during the previous year ending
31.03.2024. Under the scheme of amalgamation, Manish was alloted 1000 shares in a
new company. The market value of shares alloted is higher by ₹50,000 than the value
of holding in ABC Ltd. The assessing officer proposes to treat the transaction as an
exchange and to tax ₹50,000 as capital gain. Is he justified?
Solution:
Illustration: 19
Mr. Druv purchased 100 debentures in ABC Ltd. on 01.10.2004 @ ₹300 per debenture
and subsequently the company has converted the debentures into shares on 01.10.2017
and for each debenture 3 shares were issued and market value of the shares on the
date of conversion was ₹250 per share and market value as on 31-01-2018 is ₹300 and
subsequently assessee has sold all these shares on 01.04.2023 @ ₹500 per share and
has paid brokerage @ 1% of the sale price. Compute capital gains in the hands of Mr.
Druv in the following cases:
(a) STT not paid
81 | P a g e www.hakkseca.com/ 783-892-5588
82
4. Income under head Capital Gains
Solution:
Illustration: 20
Discuss the Tax consequences and compute tax for the Assessment Year 2024-25
under old tax regime.
Solution:
82 | P a g e www.hakkseca.com/ 783-892-5588
83
4. Income under head Capital Gains
Any amount received by the senior citizen as a loan, either in lump sum or in
installments, in a transaction of reverse mortgage is exempt from income-tax under
section 10(43). Therefore, the monthly installment of ₹30,000 received by Mrs. Monica
is exempt from income-tax under section 10(43).
However, capital gains tax liability would be attracted in the P.Y. 2023-24 when the
bank sells the mortgaged property for the purposes of recovering the loan.
Computation of Total income in the hands of Mrs. Monica for A.Y. 2024-25
Long-term Capital gains [Since the residential house property was held by Mrs.
Mahalakshmi for more than 24 months immediately preceding the date of its transfer]
HW Question: 17
Mr. Jatin (62 years old), pledged his residential house to a bank under a notified
reverse mortgage scheme. He was getting loan from bank in monthly instalments. Mr.
Jatin did not repay the loan on maturity and hence gave possession of the house to the
bank to discharge his loan. How will the treatment of long-term capital gain be on such
reverse mortgage transaction?
HW Question: 18
83 | P a g e www.hakkseca.com/ 783-892-5588
84
4. Income under head Capital Gains
Is the amount received on reverse mortgage chargeable to tax in the hands of Mr.
Jatin under the head ‘Capital Gains’?
Illustration: 21
Solution:
In this case, capital gain shall be computed during P.Y. 2022-23 (year of conversion) as
follows:
FVC (Being FMV on date of transfer): 5,00,000
Cost of Acquisition (purchase price): 3,00,000
Short term capital gain (POH is 35 months): 2,00,000
Above capital gain shall be taxable in P.Y. 2023-24 i.e. year of sell of stock.
Illustration: 22
84 | P a g e www.hakkseca.com/ 783-892-5588
85
4. Income under head Capital Gains
₹5,00,000. He sells the same on 31.03.2024 for ₹6,00,000 and gets a profit of
₹1,00,000. Discuss the tax implication in his hands under the head “Capital Gains”.
Solution:
Since car is not a capital asset in hands of Mr. Anshu, no capital gain shall arise on
conversion of the same as stock in trade.
Note: Recall the definition of “Capital assets”, it specifically excludes personal movable
effects from its definition and therefore, no capital gain is calculated in above case.
Illustration: 23
Mrs. Bansal purchased one house on 01.07.2021 for ₹4,00,000 and the house was
acquired by the Government on 01.08.2022 and compensation fixed is ₹6,00,000 and
half of the amount was paid by the Government on 01.04.2023 and balance half on
01.04.2024. She also received enhanced compensation of ₹2,00,000 on 01.06.2024 and
₹1,00,000 on 01.07.2025. Compute capital gains of Mrs. Bansal for various assessment
years assuming that income tax provisions are same for all assessment years.
Solution:
Computation of income under the head Capital Gains Capital gain shall be computed in
the year in which the asset was acquired by the Government i.e. in the previous year
2021-22 and shall be taxed in the year in which the first payment has been received by
the assessee i.e. in the previous year 2023-24
Enhanced compensation shall be taxed in the year of receipt and shall be taxable as
“short term” or “long term” based on original tax. Therefore, enhanced compensation of
₹2,00,000 shall be taxable as short-term capital gain in the P.Y. 2024-25 and
₹1,00,000 as short-term capital gain in the P.Y. 2025-26.
HW Question: 19
Mr. Suyash purchased one house on 01.10.2003 for ₹5,00,000 and this house was
acquired compulsorily by the Government on 01.07.2015. Compensation fixed by the
85 | P a g e www.hakkseca.com/ 783-892-5588
86
4. Income under head Capital Gains
Government was ₹55,00,000. Government has paid half of the amount on 01.10.2023
and balance half on 01.10.2024.
The assessee was not satisfied with the compensation and he has filed an appeal in the
High Court. The High Court has given decision on 31.03.2026 directing the Government
to pay additional compensation of ₹5,00,000 and the Government has paid ₹3,00,000 on
10.04.2026 and balance ₹2,00,000 on 10.04.2027.
Compute capital gains for the various years and tax liability for assessment year 2024-
25 under old tax regime.
Ms. Nehal has entered into an agreement with M/s DLF Build Limited on 25.05.2018 in
which she agrees to allow such Company to develop a residential property on land owned
by her in New Delhi. She purchased such land on 05.04.2010 in ₹15,00,000. In
consideration, M/s DLF Build Limited will provide 20% share in residential property to
Nehal. The certificate of completion of residential property was issued by authority as
on 26.11.2023. On such date, Stamp duty value of residential property was
₹4,14,00,000. Subsequently on 18.02.2024, she sold her 15% share in residential
property to Mr. Keshav in consideration of ₹65,00,000.
Compute total income of Ms. Nehal for the assessment year 2024-25 assuming that she
has not opted provisions under section 115BAC.
Solution:
Computation of total income of Ms. Nehal for the A.Y. 2024 -25
86 | P a g e www.hakkseca.com/ 783-892-5588
87
4. Income under head Capital Gains
Since Ms. Nehal transferred her share in the project after issue of completion
certificate, capital gains on transfer of land handed over to developer under specified
agreement in the P.Y. 2018-19 would be taxable in the previous year 2023-24, being the
year in which certificate of completion is issued as per section 45(5A). Accordingly,
capital gain arising in respect of land would be-
Full value of consideration, being 20% share in shopping mall
[Stamp duty value on the date of issue of completion certificate
(₹4,14,00,000 x 20%)] 82,80,000
Less: Indexed of cost of acquisition [₹15,00,000 x 280/167] 25,14,970
Long-term capital gain 57,65,030
Less: Deduction under section 54F
Deduction in respect of amount invested for purchase of a residential
house acquired within one year prior to date of transfer would be allowable
proportionately, since amount invested is less than the net consideration.
Accordingly, deduction would be ₹32,02,794
(₹57,65,030 x ₹46,00,000 / ₹82,80,000) (32,02,794)
Long-term capital gains 25,62,236
Less: Set-off of loss from house property
[It is beneficial to set-off loss from house property against long-term
capital gains, since in case of Ms. Nehal total income comprises of LTCG
taxable @ 20% and STCG taxable at normal slab rates; and she can claim
deduction of ₹2,80,000 under Chapter VI-A against STCG of ₹2,90,000.
Moreover, the remaining STCG would also not be taxable since it would be
below the basic exemption limit] (2,00,000)
23,62,236
87 | P a g e www.hakkseca.com/ 783-892-5588
88
4. Income under head Capital Gains
Illustration: 25
Mr. Tanmay has the following Assets which are eligible for depreciation at 15% on
Written Down Value (WDV) basis:
01.04.2020 WDV of plant ‘X’ and Plant ‘Y’ ₹2,00,000
10.12.2023 Acquired a new plant ‘Z’ for ₹2,00,000
22.01.2024 Sold plant ‘Y’ for ₹4,00,000
Expenditure incurred in connection with transfer ₹10,000
Compute eligible depreciation claim/chargeable capital gain if any, for the Assessment
Year 2024-25.
Solution:
Computation of depreciation and capital gains of Mr. Tanmay for the A.Y. 2024-
25
88 | P a g e www.hakkseca.com/ 783-892-5588
89
4. Income under head Capital Gains
Illustration: 26
Other information:
(i) Revaluation reserve is created by revising upward the value of the building of unit 1.
(ii) No individual value of any asset is considered in the transfer deed.
Compute the capital gain for the assessment year 2024-25.
Solution:
89 | P a g e www.hakkseca.com/ 783-892-5588
90
4. Income under head Capital Gains
Machinery 5,00,000
Debtors 3,00,000
Other assets 3,50,000
Total assets 20,50,000
Less:
Bank Loan (2,80,000)
Creditors (77,500)
Net worth 16,92,500
HW Question: 20
Liabilities ₹ Assets ₹
Own Capital 10,50,000 Building 5,00,000
Bank Loan 5,00,000 Furniture 5,00,000
Trade Creditors 2,50,000 Debtors 2,00,000
Unsecured Loan 2,00,000 Other Assets 8,00,000
Total 20,00,000 Total 20,00,000
Other Information:
1. No individual value of any asset is considered in the transfer deed.
2. Other assets include trademarks valuing ₹2,00,000 as on 01.04.2023 on which no
depreciation has been provided.
3. Furniture of ₹1,50,000 purchased on 05.11.2023 on which no depreciation has been
provided.
4. Unsecured loan includes ₹50,000 as advance received from his wife, which she has
agreed to waive off.
Compute the capital gain for A.Y. 2024-25.
HW Question: 21
90 | P a g e www.hakkseca.com/ 783-892-5588
91
4. Income under head Capital Gains
₹ ₹ ₹ ₹
Own Capital 15,00,000 Building 12,00,000 2,00,000 14,00,000
Revaluation Reserve 3,00,000 Machinery 3,00,000 1,00,000 4,00,000
(for building of unit
1)
Bank Loan 2,00,000 Debtors 1,00,000 40,000 1,40,000
(70% for unit 1)
Trade creditors 1,50,000 Other 1,50,000 60,000 2,10,000
(25% for unit 1 assets
Total 21,50,000 Total 17,50,000 4,00,000 21,50,000
Other information:
(i) Revaluation reserve is created by revising upward the value of the building of Unit 1.
(ii) No individual value of any asset is considered in the transfer deed.
(iii) Other assets of Unit 1 include patents acquired on 01.04.2021 for ₹50,000 on
which no depreciation has been charged. (Rate of depreciation is 25% on wdv basis)
HW Question: 22
Compute the net taxable capital gains and tax liability of Smt. X (under old tax regime)
on the basis of the following information :
A house was purchased on 01.05.2008 for ₹4,50,000 and was used as a residence by
the owner. The owner had contracted to sell this property in June, 2023 for ₹15 lacs
and had received an advance of ₹70,000 towards sale. The intending purchaser did not
proceed with the transaction and the advance was forfeited by the owner. The
property was sold in December, 2023 for ₹19,00,000.
Illustration: 27
Mr. Amilal purchased one residential house on 01.07.2001 for ₹2,00,000 and it was sold
by him on 01.07.2023 for ₹100 lakhs and he purchased one house in 01.07.2024 for
₹20,00,000. He sold this house on 01.07.2025 for ₹22,00,000. Compute his Tax
91 | P a g e www.hakkseca.com/ 783-892-5588
92
4. Income under head Capital Gains
Liability for A.Y. 2024-25 under old provisions of the Act and also capital gains for
various years.
Solution:
HW Question: 23
Mr. Rohan purchased one residential house on 01.10.2002 for ₹5,00,000 and sold the
house on 01.07.2023 for ₹100,00,000 and purchased one house on 01.01.2024 for
₹20,00,000 and this house was sold by him on 01.01.2025 for ₹25,00,000.
92 | P a g e www.hakkseca.com/ 783-892-5588
93
4. Income under head Capital Gains
Compute his income tax liability for assessment year 2024-25 and also capital gains for
all the years.
(b) Presume the house purchased on 01.01.2024 was sold on 31.01.2024
(c) Presume the house purchased on 01.01.2024 was purchased on 01.10.2024 and was
not sold upto 01.09.2027.
(d) Presume no house was purchased but the amount was deposited in capital gains
account scheme on 31.07.2024 and the amount remained unutilized.
HW Question: 24
Mr. Rasiq purchased one residential house on 01.04.2002 for ₹5,00,000. This house was
acquired compulsorily by the Government on 01.10.2013 and compensation of
₹50,00,000 was fixed by the government but the amount was paid by the Government
on 01.03.2024. The assessee has purchased one residential house on 01.02.2024 for
₹2,00,000 and the house was sold by him on 01.02.2025 for ₹4,00,000. Compute his tax
liability for the assessment year 2024-25 under old provisions of the Act and also
capital gains for the various years.
(b) Presume the house was purchased on 01.09.2024 instead of 01.02.2024.
Illustration: 28
Mr. Aman purchased one house on 01.04.2001 for ₹2,00,000 and sold the house on
01.07.2023 for ₹70,00,000 and purchased one house on 01.09.2023 for ₹12,00,000 and
it was sold by him on 01.01.2024 for ₹15,00,000. He is aged 82 years. Compute his
income and tax liability for assessment year 2024-25 assuming that he shifts out of
new tax regime.
Solution:
The assessee has the option either not to avail exemption under section 54 or to avail
exemption under section 54 and also it will be withdrawn.
93 | P a g e www.hakkseca.com/ 783-892-5588
94
4. Income under head Capital Gains
94 | P a g e www.hakkseca.com/ 783-892-5588
95
4. Income under head Capital Gains
HW Question: 25
Mr. Manohar purchased a residential house on 31st July 2021 for ₹10,00,000 and made
some additions to the house incurring ₹2,00,000 in August 2021. He sold the house
property in April 2023 for ₹20,00,000. Out of the sale proceeds, he spent ₹5,00,000
to purchase another house property in September 2023. What is the amount of capital
gains taxable in the hands of Mr. Manohar for the A.Y. 2024-25?
Illustration: 29
Mr. Shyamlal purchased agricultural land in urban area on 01.10.2002 for ₹3,00,000 and
it was being used for agricultural purposes by him. It was sold on 01.01.2024 for
₹50,00,000. The assessee has purchased one agricultural land in the rural area on
10.01.2024 for ₹10,00,000 and this land was sold by him on 11.02.2024 for ₹11,00,000
and has invested ₹30,000 in National Saving Certificate. He is aged about 86 years.
Compute his tax liability for assessment year 2024-25.
(b) Presume the land was purchased in the urban area instead of rural area.
Solution (a):
95 | P a g e www.hakkseca.com/ 783-892-5588
96
4. Income under head Capital Gains
Solution (b):
The assessee has the option either not to avail exemption under section 54B or to avail
exemption under section 54B.
96 | P a g e www.hakkseca.com/ 783-892-5588
97
4. Income under head Capital Gains
Hence, the assessee should opt for option–I and his tax liability shall be 7,43,750.
HW Question: 26
Mr. Jonny purchased agricultural land in urban area for ₹3,00,000 on 01.10.2005 and
this land was transferred by him on 01.07.2023 for ₹32,00,000 (this agricultural land
is used for agricultural purpose since its purchase). Mr. Jonny purchased one
agricultural land on 30.09.2024 in the urban area for ₹6,00,000. The agricultural land
was sold on 01.01.2025 for ₹10,00,000. He has one business also with turnover
₹105,00,000 and has income from business ₹1,10,000.
97 | P a g e www.hakkseca.com/ 783-892-5588
98
4. Income under head Capital Gains
Compute capital gains for various years and also tax liability for assessment year 2024-
25 under old provisions of the Act.
HW Question: 27
Mr. Mohan has an agricultural land (costing ₹6 lakh) in Lucknow and has been using it
for agricultural purposes since 01.04.2003 till 01.08.2013 when the Government took
over compulsory acquisition of this land. A compensation of ₹ 10 lakh was settled. The
compensation was received by Mr. Mohan on 01.07.2023. Compute the amount of capital
gains taxable in the hands of Mr. Mohan.
Will your answer be any different if Mr. Mohan had by his own will sold this land to his
friend Mr. Soham? Explain.
Will your answer be different if Mr. Mohan had not used this land for agricultural
activities? Explain.
Will your answer be different if the land belonged to Mohan Ltd. and not Mr. Mohan
and compensation on compulsory acquisition was received by the company? Explain.
Illustration: 30
Mr. Naman has one industrial undertaking in Noida industrial area and the building
which is being used for industrial purposes was purchased on 01.10.2007. Since then it
was being used for industrial purpose and was purchased for ₹23,00,000 and its w.d.v.
as on 01.04.2014 is ₹10,38,000. This building was acquired by the Government on
01.01.2015 and compensation fixed was ₹25,00,000. Entire payment was released by
the Government on 01.07.2023. The assessee has purchased one building for the
purpose of industrial undertaking in Gazipur Industrial Area on 01.01.2024 for
₹6,00,000. Compute his tax liability for assessment year 2024-25 assuming that he has
shifted out of new tax provisions u/s. 115BAC.
Solution:
98 | P a g e www.hakkseca.com/ 783-892-5588
99
4. Income under head Capital Gains
Computation of capital gains and tax liability for the assessment year 2024-25
Capital gain shall be taxed in the year in which payment has been given by the
Government i.e. in the previous year 2023-24
Illustration: 31
Mr. Saurav purchased agricultural land in the urban area on 01.04.2001 for ₹2,00,000.
It was being used for agricultural purposes since then and was sold by the assessee on
01.07.2023 for ₹1,23,00,000. He made following investments:
(i) Bonds of National Bank for Agriculture and Rural Development on 01.06.2023 for
₹1,50,000 which are redeemable after 5 years.
(ii) He purchased agricultural land on 01.09.2023 for ₹2,00,000.
(iii) He has invested ₹75,000 on 01.10.2023 in the bonds of National Highway Authority
of India redeemable after five years.
He sold the bonds of National Highway Authority of India on 15.04.2024 for
₹3,00,000.
Compute his capital gains for various years and also tax liability for assessment year
2024-25 under old tax provisions.
Solution:
99 | P a g e www.hakkseca.com/ 783-892-5588
100
4. Income under head Capital Gains
HW Question: 28
Softech Ltd. purchased one commercial building on 01.07.1995 for ₹2,00,000 and paid
brokerage of ₹20,000 and its market value as on 01.04.2001 is ₹2,10,000. The company
sold the building on 01.07.2023 for ₹5,00,00,000 and invested ₹60,00,000 in bond of
NHAI redeemable after five years. Compute tax liability of the company for
Assessment Year 2024-25.
(b) Presume building was sold for ₹11,72,00,000.
HW Question: 29
Mr. Vijendra a senior citizen (aged 65 years) sold residential building at Alwar for
₹40,00,000 on October 1st, 2023. This building was acquired by his father on
01.01.1999 for ₹1,00,000. On the death of his father on July 5th, 2006, he inherited
this building. Fair market value of this property on 01.04.2001 was ₹1,50,000. He paid
brokerage @ 1% to the real estate agent at the time of sale of the building. He
purchased a residential building at Bangalore on March 7th, 2024 for ₹8,00,000 and
deposited ₹3,00,000 on April 20th, 2024 in the bonds of National Highways authority
of India redeemable after one year. His other incomes are ₹ 50,000. He deposited
₹10,000 in public provident fund. Compute total income and tax liability of Mr. Vijendra
for the assessment year 2024-25 under old tax regime.
HW Question: 30
Mr. Bhuvam sold his residential house property on 08.06.2023 for ₹70 lakhs which was
purchased by him for ₹20 lakhs on 05.05.2006.
He paid ₹1 lakh as brokerage for the sale of said property. The stamp duty valuation
assessed by sub-registrar was ₹90 lakhs.
He bought another house property on 25.12.2023 for ₹11 lakhs.
He deposited ₹8 lakhs on 10.11.2023 in the capital gain bond of National Highway
Authority of India (NHAI).
He deposited another ₹10 lakhs on 10.07.2024 in the capital gain deposit scheme with
SBI for construction of additional floor of house property.
Compute income under the head “Capital Gains” for A.Y. 2024-25 as per Income Tax
Act, 1961 and also Income tax payable on the assumption that he has no other income
chargeable to tax.
7. Exemption from capital gain on transfer of any capital asset (other than
residential house property) [section 54F]
Illustration: 32
Mr. Ashok purchased gold on 01.04.1991 for ₹3,00,000 and its market value on
01.04.2001 is ₹2,00,000. This gold was sold by him on 01.01.2024 for ₹35,00,000 and
selling expenses are ₹37,000. He has purchased one house on 01.05.2024 for ₹4,00,000
because he did not have any house in his name and he deposited ₹3,00,000 in capital
gain account scheme on 30.09.2024.
Mr. Ashok is also engaged in a business and he has turnover of his business
₹105,00,000 and cost of goods sold ₹100,00,000 and other expenses ₹5,10,000.
He has withdrawn ₹2,00,000 from capital gain account scheme on 01.01.2025 and
constructed 1st floor of the house which was purchased by him on 01.05.2024.
Solution:
HW Question: 31
On 25.04.2023, Mr. Vijay sold an urban agricultural land for ₹60,00,000 which he had
been using for agricultural purposes for several years. He acquired that land in 2000
for ₹2,50,000. The market value of such land as on 01.04.2001 was ₹5,00,000. He
purchased rural agricultural land for ₹8,00,000 on 25.06.2023 which was sold for
₹12,50,000 on 18.01.2024. A sum of ₹12,50,000 was also invested by him in purchase of
residential property on 25.07.2023. He did not own any house property before this
date. The new house property was sold on 28.03.2024 for ₹15,00,000.
Compute tax liability for assessment year 2024-25 under old tax provision.
HW Question: 32
Mr. Akshay sold gold for ₹5,50,000 on 01.10.2023 which had been acquired by him in
October, 2004 for ₹55,000. He wants to utilize the said amount of sale consideration
for purchase or construction of a new residential house. He already owns one
residential house at the time of sale of the gold on 01.10.2023.
He has deposited ₹4,00,000 under the capital gains deposit scheme with a specified
bank on 30.04.2024.
Ascertain the capital gains taxable in Mr. Akshay’s hands for assessment year 2024-25
and advise him as to what further action he has to take to avail the exemption.
Illustration: 33
Mr. Manit purchased 500 debentures on 01.07.2001 of Empire Ltd. @ ₹390 per
debenture and paid brokerage @ 1.5%. The debentures were converted into share @ 3
share for each debenture on 01.07.2011. Market value on the date of conversion was
₹170 per share. All the shares were sold on 01.07.2023 @ ₹900 per share and no
securities transaction tax has been paid and paid brokerage @ 1.5%.
A sum of ₹1,00,000 was invested in purchasing a house on 28.06.2024 because the
assessee did not have any house and ₹1,00,000 was deposited in capital gain account
scheme on 30.06.2024 for availing exemption under section 54F and ₹ 50,000 was
withdrawn on 02.07.2024 to construct first floor of the house purchased on
28.06.2024.
Compute total income and tax liability for assessment year 2024-25 and capital gains
for various years under old tax provisions.
Solution:
Computation of Capital Gains for Previous Year 2023-24 Assessment Year 2024-25
Comprehensive Questions
Question: 1
Mr. Rohit (aged 56 years) sold the following assets during the previous year 2023-24:
1. He purchased one house in rural area on 01.10.1991 for ₹2,00,000 and incurred
₹50,000 on its improvement on 01.07.2000. Its market value on 01.04.2001 is
₹2,30,000. It was sold on 01.04.2023 for ₹5,00,000.
2. He purchased agricultural land in the rural area for ₹2,00,000 on 01.07.2002 and
sold it on 01.07.2023 for ₹3,00,000.
3. He purchased one T.V. for his personal use on 01.01.2003 for ₹25,000 and sold it on
30.12.2023 for ₹20,000.
4. He purchased gold on 01.07.2018 for ₹3,00,000 and sold it on 01.04.2023 for
₹4,50,000.
5. He has one motor car in his business with written down value as on 01.04.2023 of
₹2,00,000 and it was sold by him on 01.07.2023 for ₹2,50,000.
6. He purchased one house on 01.10.2002 for ₹7,00,000 and incurred ₹4,50,000 on
01.10.2012 to construct its first floor and subsequently the house was sold on
01.01.2024 for ₹90,00,000 and selling expenses were 2% of the sale price.
Compute tax liability of Mr. Rohit for the assessment year 2024-25 under regular
provisions of the Act.
Mr. Mohit purchased one house property on 01.07.1992 for ₹3,00,000 and incurred
₹1,00,000 on its improvement in 1995-96 and its market value as on 01.04.2001 was
₹32,00,000 and he incurred ₹5,00,000 on its improvement in 2014-15 and sold the
house on 01.11.2023 for ₹2,00,00,000.
He purchased one commercial building on 01.04.2020 for ₹50,00,000 and it was let out
@ ₹2,00,000 p.m. to XYZ Ltd. and XYZ Ltd. has deducted tax at source. Mr. Mohit has
paid Municipal Tax of ₹20,000 p.m. Compute Income Tax Payable for A.Y. 2024-25
under old tax provisions and also amount of tax deducted at source by XYZ Ltd. and
also tax deducted at source by the person who has purchased the house property.
Mr. Lokesh died on 16.08.2008 and as per his will these assets were transferred to his
son Mukesh. Mr. Mukesh now sells these assets on 10.06.2023 for ₹20,00,000 and
₹3,00,000 respectively and securities transaction tax has been paid on sale of equity
shares.
Find out the amount of capital gains chargeable to tax and also tax liability for the
assessment year 2024-25 assuming that he opts out of the provisions of section
115BAC.
Mr. Mihir has submitted information regarding sale of certain assets as given below:
1. He purchased one house on 01.10.1998 for ₹5,00,000 and paid brokerage ₹25,000. He
entered into an agreement to sell this house on 01.04.2001 for ₹5,10,000 but the buyer
backed out. He constructed its first floor on 01.01.2014 by incurring ₹4,00,000 and
subsequently this house was sold on 01.01.2024 for ₹1,60,00,000 and selling expenses
were ₹85,000.
2. He purchased Preference shares in ABC Ltd. on 01.07.2013 for ₹1,50,000 and sold
these shares on 31.03.2024 for ₹1,00,000.
3. He purchased one motor car for personal use on 28.02.2003 for ₹2,00,000 and sold
it on 01.04.2023 for ₹2,10,000.
4. He purchased gold ornaments on 01.10.2000 for ₹2,10,000. Its market value on
01.04.2001 is ₹2,00,000 and it was sold by him on 01.07.2023 for ₹8,00,000.
5. He purchased silver utensils on 01.07.2002 for ₹30,000 and these utensils were sold
by him on 01.01.2024 for ₹23,000.
6. He has invested ₹35,000 in the units of UTI.
Compute his income tax liability for assessment year 2024-25 under regular provisions
of the Act.
Examine the taxability of Capital gains in the following scenarios for the Assessment
Year 2024-25, determine the taxable amount and rate of tax applicable:
(i) On 28th February, 2024, 10,000 shares of Ram Ltd., a listed company are sold by
Mr. Bhishwa @ 550 per share and STT was paid at the time of sale of shares. These
shares were acquired by him on 5th April, 2017 @ 395 per share by paying STT at the
time of purchase. On 31st January, 2018, the shares of Ram Ltd. were traded on a
recognized stock exchange at the Fair Market Value of ₹ 390 per Share.
(ii) Mr. Ashok is the owner of residential house which was purchased on 1st September,
2016 for ₹9,00,000. He sold the said house on 4th September, 2023 for ₹ 19,00,000.
Valuation as per stamp valuation authorities was ₹ 45,00,000. He invested ₹ 19,00,000
in NHAI Bonds on 21st March 2024.
Mr. Patel is a proprietor of Star Stores since 20.05.2020. He has transferred his shop
by way of slump sale for a total consideration of ₹ 40 Lakh. The professional fees &
brokerage paid for this sale are ₹80,000. His Balance Sheet as on 31.03.2024 is as
under:
Liabilities Assets
Own Capital 10,50,000 Building 5,00,000
Bank Loan 5,00,000 Furniture 5,00,000
Trade Creditors 2,50,000 Debtors 2,00,000
Unsecured Loan 2,00,000 Other Assets 8,00,000
Total 20,00,000 Total 20,00,000
Other Information:
1. No individual value of any asset is considered in the transfer deed.
2. Other assets include trademarks valuing ₹ 2,00,000 as on 01.04.2023 on which no
depreciation has been provided.
3. Furniture of ₹ 1,50,000 purchased on 05.11.2023 on which no depreciation has been
provided.
4. Unsecured loan includes ₹ 50,000 as advance received from his wife, which she has
agreed to waive off.
Computer the capital gain for A.Y. 2024-25.
Mr. Rajan provides you the following details with regard to sale of certain securities by
him during F.Y. 2023-24:
(i) Sold 10,000 shares of A Ltd. on 05.04.2023 @ ₹650 per share
A Ltd. is a listed company. These shares were acquired by Mr. Rajan on 05.04.2016 @
₹100 per share. STT was paid both at the time of acquisition as well as at the time of
transfer of such shares which was affected through a recognized stock exchange. On
31.01.2018, the shares of A Ltd. were traded on a recognized stock exchange as under:
Highest price - ₹300 per share
Average price - ₹290 per share
Lowest price - ₹280 per share
C Ltd. is an un-listed company. These shares were issued by the company as bonus
shares on 30.09.1997. The Fair Value of these shares as on 01.04.2001 was ₹50 per
share.
Calculate the amount chargeable to tax under the head ‘Capital Gains’ and also calculate
tax on such gains for A.Y. 2024-25 assuming that the other incomes of Mr. Rajan
exceed the maximum amount not chargeable to tax. (Ignore surcharge and cess).
Mr. Joy owned a residential house in Noida. It was acquired on 09.09.2009 for
₹30,00,000. He sold it for ₹1,57,00,000 on 07.01.2020. Mr. Joy utilized the sale
proceeds of the above property to acquire a residential house in Panchkula for
₹2,05,00,000 on 20.07.2020. The said house property was sold on 01.06.2023 and he
purchased another residential house in Delhi for ₹2,57,00,000 on 02.03.2024. The
property at Panchkula was sold for ₹3,25,00,000.
Calculate capital gains chargeable to tax for the assessment year 2020-21 and 2024-
25. All workings should form part of your answer.
Discuss the tax consequences and compute tax for the Asst. Year 2024-25.
Mr. Anil entered into an agreement with Mr. Dhawan to sell his residential house
located at Navi Mumbai on 16.08.2023 for ₹80,00,000. The sale proceeds were to be
paid in the following manner;
(i) 20% through account payee bank draft on the date of agreement.
(ii) 60% on the date of the possession of the property.
(iii) Balance after the completion of the registration of the title of the property.
Mr. Dhawan was handed over the possession of the property on 15.12.2023 and the
registration process was completed on 14.01.2024. He paid the sale proceeds as per the
sale agreement.
The value determined by the Stamp Duty Authority on 16.08.2023 was ₹90,00,000
whereas, on 14.01.2024 it was ₹91,50,000.
Mr. Anil had acquired the property on 01.04.2001 for ₹10,00,000. After recovering the
sale proceeds from Dhawan, he purchased another residential house property for
₹35,00,000.
Compute the income under the head “Capital Gains” for the Assessment Year 2024-25
and also compute tax liability under old tax provisions.
Mr. Ramawatar bought a vacant Land for ₹80 lakhs in May 2004. Registration and other
expenses were 10% of the cost of land. He constructed a residential building on the
said land for ₹100 lakhs during the financial year 2006-07.
He entered into an agreement for sale of the above said residential house with Mr.
John (not a relative) in April 2016. The sale consideration was fixed at ₹700 lakhs and
on 23.4.2016, Mr. Ramawatar received ₹20 lakhs as advance in cash by executing an
agreement. The sale deed was executed and registered on 14.01.2024 for the agreed
consideration. However, the State stamp valuation authority had revised the values;
hence the value of property for stamp duty purposes was ₹780 lakhs. Mr. Ramawatar,
paid 1% as brokerage on sale consideration received.
Subsequent to sale, Mr. Ramawatar made following investments:
(i) Acquired a residential house at Delhi for ₹110 lakhs.
(ii) Acquired a residential house at London for ₹190 lakhs.
(iii) Subscribed to NHAI capital gains bond (approved under section 54EC) for ₹45
lakhs on 29.03.2024 and for 50 lakhs on 12.05.2024.
Compute the income chargeable under the head 'Capital Gains'. The choice of
exemption must beneficial to the assessee.
Mr. Piyush, a resident individual, aged 55 years, purchased 10 Plots in the financial year
2003-04 for ₹12 Lakh. On 1stApril 2004, he started a business of property dealing and
converted all 10 plots as stock in trade of his business and recorded the cost at ₹40
lakhs in his books being the fair market value on 1stApril 2004.
On 31st March 2011, he sold all 10 Plots for ₹55 Lakh and purchased a residential house
property for ₹50 Lakh. He has constructed 2 rooms in this residential house in June
2011 and has spent ₹8 Lakh.
He sold the above residential house on 5th Feb 2024, for ₹85 Lakh. The valuation
adopted by Stamp valuation authority for the payment of stamp duty was ₹128 Lakh.
On the request of Mr. Piyush, A.O. made a reference to the valuation officer. The
Valuation Officer determined the value at ₹130 Lakh. Mr. Piyush paid brokerage 1% of
sale consideration.
Compute the total Income and total Tax liability of Mr. Piyush for the Assessment year
2024-25 under normal provisions of the Act.
Mr. Sahil sold a house, held as a capital asset, to his friend Mr. Soheb on 1st December,
2023 for a consideration of ₹25,00,000. The Sub-Registrar refused to register the
document for the said value, as according to him, stamp duty valuation based on State
Government guidelines was ₹45,00,000. Mr. Sahil preferred an appeal to the Revenue
Divisional Officer, who fixed the value of the house as ₹35,00,000 (₹22,00,000 for
land and the balance for building portion). The differential stamp duty was paid,
accepting the said value determined. Mr. Sahil had purchased the land on 1st June,
2006 for ₹5,19,000 and completed the construction of the house on January, 2021 for
₹14,00,000.
Briefly discuss the tax implications in the hands of Mr. Sahil for the assessment year
2024-25 and compute the capital gains chargeable to tax.
Mrs. X had purchased 500 equity shares in A Ltd. At a cost of ₹30 per share
(brokerage 1%) in February 1999.
She got 50 bonus shares in September 2000.
She again got 550 bonus shares by virtue of her holding on March, 2005. Fair market
value of the shares of A Ltd. on April, 2001 is ₹50. Market value as on 31.01.2018 ₹200
per share.
In January 2024, she transferred all her shares @ 300 per share (brokerage 2%).
Compute the capital gains taxable in the hands of Mrs. X for the Assessment Year
2024-25 assuming.
(a) A Ltd. is an unlisted company and securities transaction tax was not applicable at
the time of sale.
(b) A Ltd. is a listed company and the shares are sold in a recognized stock exchange
and securities transaction tax was paid at the time of sale.
Illustration: 1
Aruna Ltd. has 1,00,000 equity shares of ₹10 each and Mr. Arun purchased 10,000 equity
shares on 01.01.2023 of ₹10 each and the company goes into liquidation on 31.07.2023 and
company has net distributable amount of ₹60 lakhs after discharging all the liabilities and
it includes accumulated profits of ₹20 lakhs and the entire amount was distributed among
the shareholders. Calculate his total income.
Solution:
Mr. Anil purchased 10,000 equity shares of Anila Pvt. ltd. on 28.02.2023 for ₹1,20,000.
The company was wound up on 31.07.2023. The following is the summarized financial
position of the company as on 31.07.2023:
Liabilities Assets
1,00,000 Equity Shares 10,00,000 Land 42,00,000
General Reserve 40,00,000 Cash at bank 10,50,000
Provision for Taxation 2,50,000
52,50,000 52,50,000
The tax liability was ascertained at ₹3,00,000. The remaining assets were distributed to
the shareholders in the proportion of their shareholding. The market value of land as on
31.07.2023 is ₹1,00,00,000. The land received above was sold by Mr. Anil on 28.02.2024
for ₹15,00,000. Discuss the tax consequences in the hands of the company and Mr. Anil.
Solution:
Dividend u/s 2(22)(c) of ₹3,95,000 will be Taxable in the hands of Mr. Anil (Shareholder)
during the F.Y. 2023- 24.
Mr. Anil has received the land from the company for ₹10 lakh but it has been sold by him
for ₹15 lakh, in this case capital gains shall be computed in the manner given below:
Mr. Mihit has 15% share-holding in IPL (P) Ltd. and has also 50% share in Mihit & Sons, a
partnership firm. The accumulated profit of IPL (P) Ltd. is ₹20 Lakh. Mihit & Sons had
taken a loan of ₹25 Lakh, from IPL (P) Ltd. Explain, whether the above loan is treated as
dividend, as per the provision of Income Tax Act, 1961.
Solution:
As per Section 2(22)(e), if the loan or advance has been given to any concern in which
shareholder has substantial interest, such loan or advance shall be considered to be
dividend in the hands of such concern but only to the extent of accumulated profits
excluding capitalized profits.
In this case dividend in the hands of the shareholder is nil and in hands of the firm are
₹20 lakhs.
Illustration: 4
Sohil, a resident Indian, holding 28% of equity shares in a company, took a loan of
₹5,00,000 from the same company. On the date of granting the loan, the company had
accumulated profit of ₹4,00,000. The company is engaged in some manufacturing activity.
(i) Is the amount of loan taxable as deemed dividend, if the company is a company in which
the public are substantially interested?
(ii) What would be your answer, if the company is a private limited company (i.e. which is
not a company in which the public are substantially interested)?
Solution:
Any payment by a company, other than a company in which the public are substantially
interested, of any sum by way of advance or loan to an equity shareholder who is the
beneficial owner of shares holding not less than 10% of the voting power, is deemed as
dividend under section 2(22)(e), to the extent the company possesses accumulated profits.
(i) The provisions of section 2(22)(e), however, will not apply where the loan is given by a
company in which public are substantially interested. In such a case, the loan would not be
taxable as deemed dividend.
(ii) However, if the loan is taken from a private company (i.e. a company in which the public
are not substantially interested), which is a manufacturing company and not a company
where lending of money is a substantial part of the business of the company, then, the
provisions of section 2(22)(e) would be attracted, since Sohil holds more than 10% of the
equity shares in the company. The amount chargeable as deemed dividend cannot, however,
exceed the accumulated profits held by the company on the date of giving the loan.
Therefore, the amount taxable as deemed dividend would be limited to the accumulated
profit i.e., ₹4,00,000 and not the amount of loan which is ₹5,00,000.
Illustration: 5
Mr. Sahil has earned the following incomes during P.Y. 2023-24:
• Income from business of owning and maintaining race camels of ₹60,000
• Income from owning and maintaining of race horses of ₹10,000; and
• Income from horse races of ₹7,000.
Compute his tax liability for A.Y. 2024-25 under old tax provisions.
Solution: Computation of Total Income and tax liability for A.Y. 2024-25:
• ₹3,80,000, 11% debentures of Ashok Ltd (non-listed) (interest is paid on June 30 every
year).
Apart from the aforesaid securities, Mr. Akshay has invested in some other securities and
received the following interest net of 10% TDS, wherever applicable:
• ₹36,000 on securities issued by UP Government;
• ₹90,000 on securities issued by Central Government of India; and
• ₹1,08,000 on debentures of Sumit Ltd.
His business income is ₹24,32,000. He pays ₹6,000 as commission to his bank for
collecting interest on securities. Determine the taxable income of Mr. Akshay for A.Y.
2024-25.
Solution:
Note: No Tax is Deducted at Source under section 193 where the interest is paid by
central government/ state government on securities issued by them.
OLX Investments (P) Ltd. was incorporated during P.Y. 2021-22 having a paid-up capital of
₹10 Lakhs. In order to increase its capital, the company further issues, 1,00,000 shares
(having face value of ₹100 each) during the year at par as on 01.08.2023. The FMV of such
share as on 01.08.2023 was ₹85.
(i) Determine the tax implications of the above transaction in the hands of company,
assuming it is the only transaction made during the year.
(ii) Will your answer change, if shares were issued at ₹105 each?
(iii) What will be your answer, if shares were issued at ₹105 and FMV of the share was
₹120 as on 01.08.2023?
Solution:
As per section 56(2)(viib), where a company, not being a company in which public are
substantially interested, receives in any previous year, from any resident person, any
consideration for issue of shares that exceeds the face value of the shares, aggregate
consideration received for such shares as exceeds the Fair market value of the shares
shall be taxable under the head other sources.
(i) In the given case, shares are issued at face value and does not issue in price exceeding
face value of the shares. Hence no amount is taxable.
(ii) In the given case, shares are issued exceeding face value hence taxable amount shall
be (Issue Price – FMV of Shares) * No. of Shares = (105-85) x 1,00,000 = ₹20,00,000.
(iii) In the given case, shares are issued exceeding face value but shares are issued lower
than FMV, hence no amount shall be taxable in hands of company.
Illustration: 8
Mr. RAM has taken a loan of ₹1,00,000 @ 10%. The amount was invested by him in the
securities of one company. During the year he has received gross interest of ₹18,000 and
has paid collection charges to the bank ₹500. He has paid interest ₹10,000 on the loan
taken by him for investment and has long term capital gain under section 112A of
₹4,00,000 and casual income ₹10,000. Deductions allowed under section 80C to 80U
₹10,000. Compute his tax liability for assessment year 2024-25 under old tax provisions.
Solution:
Mrs. Sita is getting family pension of ₹7,000 p.m. She also has dividend income from
domestic company of ₹31,000. She has long term capital gain under section 112A
₹3,00,000. Compute her tax liability for assessment year 2024-25 under regular tax
provisions.
Solution:
Illustration: 10
Interest on enhanced compensation received by Mr. Sukhilal during the previous year
2023-24 is ₹6,50,000. Out of this interest, ₹2,00,000 relates to the previous year 2020-
21, ₹2,15,000 relates to previous year 2021-22 and ₹2,35,000 relates to previous year
2022-23. Discuss the tax implication, if any, of such interest income for A.Y. 2024-25.
Solution:
The entire interest of ₹6,50,000 would be taxable in the year of receipt, namely, P.Y.
2023-24.
Interest on enhanced compensation taxable u/s 56 6,50,000
Less: Deduction under section 57 @ 50% (3,25,000)
Interest chargeable under the head “Income from other sources” 3,25,000
Illustration: 11
Solution:
Section 145B provides that interest received by the assessee on enhanced compensation
shall be deemed to be the income of the assessee of the year in which it is received,
irrespective of the method of accounting followed by the assessee and irrespective of the
financial year to which it relates.
Section 56(2)(viii) states that such income shall be taxable as ‘Income from other
sources’. 50% of such income shall be allowed as deduction by virtue of section 57(iv) and
no other deduction shall be permissible from such Income. Therefore, legal expenses
incurred to receive the interest on enhanced compensation would not be allowed as
deduction from such income.
Illustration: 12
A sum of ₹2,00,000 is received as gift from non-relatives by Mr. Rajpal on the occasion of
marriage of his son. Explain the taxability as gift.
Solution:
Illustration: 13
Solution:
The provisions of section 56(2)(x) would not apply to any sum of money or any property
received from any trust or institution registered under section 12AB. Therefore, the cash
gift of ₹1 lakh received from Atma Charitable Trust, being a trust registered under
section 12AB, for meeting medical expenses would not be chargeable to tax under section
56(2)(x) in the hands of Mr. Chezian.
Comprehensive Questions
Discuss the taxability or otherwise in the hands of the recipient as per the provision of
the Income Tax Act:
(i) Healthy Pvt. Ltd., a closely held company, issued 10,000 shares at ₹130 per share.
(The face value of the share is ₹100 per share and the fair market value of the
share is ₹120 per share).
(ii) Mr. Akash received an advance of ₹50,000 on 01.09.2023 against the sale of his
house. However, due to non-payment of installment in time, the contract has
cancelled and the amount of ₹50,000 was forfeited.
(iii) Mr. Naveen, a member of his father’s HUF, transferred a house property to the
HUF without consideration. The value of the house is ₹10 lacks as per the Registrar
of Stamp Duty.
(iv) Mr. Kumar gifted a car to his sister’s son (Sunil) for achieving good marks in CA
Final Exam. The fair market value of the car is ₹5,00,000.
Find the tax liability of Mrs. Abhey (age 40 years), a resident individual, for the
assessment year 2024-25 from the following particulars of her incomes and spending for
the previous year ending March 31, 2024.
Income from house property (Computed) 90,000
Dividend from UTI 35,000
Family pension (gross) 90,000
Interest on bank FD (gross) 14,000
Dividend from foreign company 36,000
Gift received from her sister 26,000
Winnings from lotteries (gross) 70,000
Long-term capital gain 1,20,000
Payment for purchase of National Savings Certificates 35,000
Assume that he has shifted out from the provisions of section 115BAC.
Aruna Ltd. has 1,00,000 equity shares of ₹10 each and Mr. Arun purchased 10,000 equity
shares on 01.01.2022 of ₹10 each and the company goes into liquidation on 31.07.2022 and
company has net distributable amount of ₹60 lakhs after discharging all the liabilities and
it includes accumulated profits of ₹20 lakhs and the entire amount was distributed among
the shareholders.
Minor son of Mr. Arun has interest income of ₹2 lakhs from one bank deposit which was
gifted to him by his grand-father.
Mrs. Arun has one business and income from business is ₹1 lakh entire capital was gifted
by Mr. Arun.
Mr. Arun is growing flowers and has income of ₹7 lakh from sale of flowers.
Compute his tax liability for Assessment Year 2024-25 assuming that the Assessee has
not opted for section 115BAC.
Question: 5 [Residential status & scope of total income + House property + Other
sources] [NOV 2019 (New Course)]
Mr. Jagdish, aged 61 years, has set-up his business in Thailand and is residing in Thailand
since last 20 years. He owns a house property in Bangkok, half of which is used as his
residence and half is given on rent (such rent received, converted in INR is ₹6,00,000).
The annual value of the house in Thailand is ₹50,00,000 i.e. converted value in INR.
He purchased a flat in Pune during F.Y. 2019-20, which has been given on monthly rent of
₹27,500 since 01.07.2022. The annual property tax of Pune flat is ₹40,000 which is paid
by Mr. Jagdish whenever he comes to India. Mr. Jagdish last visited India in July 2021. He
has taken a loan Union Bank of India for purchase of the Pune flat amounting to
₹15,00,000. The interest on such loan for the F.Y. 2023-24 was ₹84,000. However,
interest for March 2024 quarter has not yet been paid by Mr. Jagdish.
He had a house in Jaipur which was sold in May 2019. In respect of this house, he received
arear of rent of ₹96,000 in Feb 2024 (not taxed earlier).
He also derived some other incomes during F.Y. 2023-24 which are as follows:
Compute the total income of Mr. Jagdish for Assessment Year 2024-25 chargeable to
income tax in India.
Mr. Mahadev, a noted bhajan singer of Rajasthan and his wife Mrs. Dariya furnish the
following information relating to the Assessment Year 2024-25.
Compute total taxable income of Mr. Mahadev & Mrs. Dariya for the Assessment Year
2024-25.
State with reasons whether the following receipts are taxable or not under the provisions
of Income tax Act, 1961?
Discuss the taxability or otherwise in the hands of the recipients, as per the provisions of
the Income-tax Act, 1961:
(i) Abhishek Private Limited, a closely held company, issued 10,000 shares at ₹130 per
share. (The face value of the share is ₹100 per share and the fair market value of the
share is ₹120 per share).
(ii) Mr. Abhijeet received an advance of ₹50,000 on 01.09.2023 against the sale of his
house. However, due to non-payment of instalment in time, the contract has cancelled and
the amount of ₹50,000 was forfeited.
Discuss with reason, whether the following transactions are true or false, as per the
provisions of Income Tax Act, 1961:
Dividend received by a dealer in shares or one engaged in buying/selling of shares, is
chargeable under the head “Income from other sources”.
Ms. Anima received following amounts during the previous year 2023-24.
(1) Received loan of ₹5,00,000 from the ABC Private Limited, a closely held company
engaged in bicycle business. She is holding 10% of the equity share capital in the said
company. The accumulated profit of the company was ₹2,00,000 on the date of the loan.
(2) Received Interest on enhanced compensation of ₹5,00,000. Out of this interest,
₹1,50,000 relates to the previous year 2019-20, ₹1,90,000 relates to previous year 2020-
21 and ₹1,60,000 relates to previous year 2021-22. She paid 1 lakh to her advocate for his
efforts in the matter.
Discuss the tax implications, if any, arising from these transactions in her hand with
reference to Assessment Year 2024-25.
State with brief reasoning whether the following receipts are chargeable to income-tax or
are exempt (if chargeable, the amount taxable is to be mentioned) for the assessment
year 2024-25:
(i) Interest on enhanced compensation received on 12.03.2024 for
acquisition of urban land, of which 40% relates to the earlier year. 96,000
(ii) Rent received for letting out agricultural land for a movie shooting. 72,000
Computation is NOT required.
Question: 12 [Gift]
From the following transactions relating to Mrs. Natasha, dealer in shares, determine the
amount chargeable to tax in her hands for the A.Y. 2024-25. Your answer should be
supported by the reasons:
(i) On 01.01.2024, being her birthday, she received a gift of ₹40,000 by means of cheque
from her father's maternal uncle.
(ii) On 12.02.2024, she acquired a vacant site from her friend for ₹1,32,000. The State
stamp valuation authority fixed the value of site at ₹ 2,00,000 for stamp duty purpose.
(iii) She bought 50 equity shares of a private company from another friend for ₹ 75,000.
The fair market value of such shares on the date of purchase was ₹ 1,33,000.
Question: 13 [Gift]
Mr. Sumit, a dealer in shares, received the following without consideration during the P.Y.
2023-24 from his friend Mr. Rohan, -
(1) Cash gift of ₹75,000 on his anniversary, 15th April, 2023.
(2) Bullion, the fair market value of which was ₹60,000, on his birthday, 19th June, 2023.
(3) A plot of land at Faridabad on 1st July, 2023, the stamp value of which is ₹5 lakh on
that date. Mr. Rohan had purchased the land in April, 2016.
(4) Mr. Sumit purchased from his friend Miss Roshni, who is also a dealer in shares, 1000
shares of Adani Ltd. @ ₹400 each on 19th June, 2023, the fair market value of which was
₹600 each on that date. Mr. Sumit sold these shares in the course of his business on 23rd
June, 2023.
(5) On 1st November, 2023, Mr. Sumit took possession of property (building) booked by
him two years back at ₹20 lakh. The stamp duty value of the property as on 1st November,
2023 was ₹32 lakh and on the date of booking was ₹ 23 lakh. He had paid ₹ 1 lakh by
account payee cheque as down payment on the date of booking.
Compute the income of Mr. Sumit chargeable under the head “Income from other sources”
for A.Y. 2024-25.
Question: 14 [Gift]
Discuss the taxability or otherwise of the following in the hands of the recipient under
section 56(2)(x) the Income-tax Act, 1961 –
(i) X HUF received ₹75,000 in cash from niece of Mr. X (i.e., daughter of Mr. X’s sister).
Mr. X is the Karta of the HUF.
(ii) Miss. X, a member of her father’s HUF, transferred a house property to the HUF
without consideration. The stamp duty value of the house property is ₹9,00,000.
(iii) Mr. X received 100 shares of A Ltd. from his friend as a gift on occasion of his 25th
marriage anniversary. The fair market value on that date was ₹100 per share. He also
received jewellery worth ₹45,000 (FMV) from his nephew on the same day.
(iv) X HUF gifted a car to son of Karta for achieving good marks in XII board examination.
The fair market value of the car is ₹5,25,000.
Examine whether the following are chargeable to tax and the amount liable to tax:
(i) A sum of ₹1,20,000 was received as gift from non-relatives by Raj on the occasion of
the marriage of his son Pravin.
(ii) Interest on enhanced compensation of ₹96,000 received on 12.03.2024 for acquisition
of urban land, of which 40% relates to P.Y. 2022-23.
Question: 16 [Gift]
The following details have been furnished by Mrs. Hemali pertaining to the year ended
31.03.2024:
(i) Cash gift of ₹51,000 received from her friend on the occasion of her “Shastiaptha
Poorthi”, a wedding function celebrated on her husband completing 60 years of age. This
was also her 25th wedding anniversary.
(ii) On the above occasion, a diamond necklace worth ₹2 lacs presented by her sister living
in Dubai.
(iii) When she celebrated her daughter's wedding on 21.02.2024, her friend assigned in
Mrs. Hemali's favour, a fixed deposit held by the said friend in a scheduled bank; the value
of the fixed deposit and the accrued interest on the said date was ₹52,000.
Question: 17 [Gift]
Mr. Subramani sold a house plot to Mrs. Vimala for ₹45 lakhs on 12.05.2023. The valuation
determined by the stamp valuation authority was ₹53 lakhs. Discuss the tax consequences
of above, in the hands of each one of them, viz, Mr. Subramani & Mrs. Vimala. Mrs. Vimala
has sold this plot to Ms. Padmaja on 21.03.2023 for ₹55 lakhs. The valuation as per stamp
valuation authority remains the same at ₹53 lakhs. Compute the capital gains arising on
sale of the house plot by Mrs. Vimala.
Note: None of the parties viz Mr. Subramani, Mrs. Vimala & Ms. Padmaja are related to
each other; the transactions are between outsiders.
Question: 20 [Residential status, Scope of total income, house property, IFOS] [MAY
– 2007 (6 Marks)]
Miss Charlie, an American national, got married to Mr. Radhey of India in USA on
02.03.2023 and came to India for the first time on 16.03.2023. She left for USA on
23.09.2023. She returned to India again on 27.03.2024. While in India, she had purchased
a show room in Mumbai on 22.04.2023, which was leased out to a company on a rent of
₹25,000 p.m. from 01.05.2023. She had taken loan from a bank for purchase of this show
room on which bank had charged interest of ₹97,500 up to 31.03.2024.
She had received the following gifts from her relatives and friends during 01.04.2023 to
30.06.2023:
- From parents of husband 51,000
- From married sister of husband 11,000
- From two very close friends of her husband, ₹1,51,000 and ₹21,000 1,72,000
Determine her residential status and compute the total income chargeable to tax along
with the amount of tax payable on such income for the Assessment Year 2024-25 under
old tax provisions. .
Illustration: 1
Particulars ₹
(1) Opening balance of plant and machinery as on 40,00,000
01.04.2023 (rate 15%)
(2) New plant and machinery purchased and put to 10,00,000
use on 08.06.2023
(3) New plant and machinery acquired and put to 5,00,000
use on 15.12.2023
(4) Computer acquired and installed in the office 7,00,000
premises on 02.01.2024 (rate 40%)
Compute the amount of depreciation as per the Income tax Act, 1961 for the A.Y.
2024-25. Assume that all the assets were purchased by way of account payee
cheque.
Solution:
Illustration: 2
Solution:
Particulars Amount
Step 1 Opening w.d.v. as on 01.04.2023 3,00,000
Step 2 - Purchased and put to use for less than 180 days 5,00,000
- Purchased and put to use for 180 days or more 4,00,000
12,00,000
Step 3 Sold during the year (4,00,000)
Step 4 W.D.V. before depreciation 8,00,000
Step 5 Depreciation allowance
- Assets purchased and put to use for less than 180 25,000
days [5,00,000 * 10%/2]
- Assets purchased and put to use for 180 days or 30,000
more [(8,00,000-5,00,000) * 10%]
Total depreciation allowance 55,000
What if, building C, which is “purchased and put to use for less than 180 days” is
sold instead of building B in above example?
(Now, you may think that since we have sold the asset which was purchased and put
to use for less than 180 days, therefore, depreciation on the remaining assets shall
be allowed at full rate on balance amount calculated in step 4. Please keep this in
mind that we NEVER give importance to individual asset (i.e. building A, B or C)
while calculating depreciation under “Block of Assets” method. Therefore, we shall
NOT consider the individual asset and IGNORE the names of assets if given in the
questions.)
Illustration: 3
Solution:
Particulars Amount
Step 1 Opening w.d.v. as on 01.04.2023 3,00,000
Step 2 - Purchased and put to use for less than 180 days 5,00,000
- Purchased and put to use for 180 days or more 4,00,000
12,00,000
Step 3 Sold during the year (8,00,000)
Step 4 Wdv before depreciation 4,00,000
Step 5 Depreciation allowance
- Assets purchased and put to use for less than 180 20,000
days [4,00,000 * 10%/2]
- Assets purchased and put to use for 180 days or 0
more
Total depreciation allowance 20,000
What if, building C, which is “purchased and put to use for less than 180 days” is
sold instead of building B in above example?
(Now, you may think that since we have sold the asset which was purchased and put
to use for less than 180 days, therefore, depreciation on the remaining assets shall
be allowed at full rate on balance amount calculated in step 4. Please keep this in
mind that we NEVER give importance to individual asset (i.e. building A, B or C)
while calculating depreciation under “Block of Assets” method. Therefore, we shall
NOT consider the individual asset and IGNORE the names of assets if given in the
questions.)
Illustration: 4
Solution:
Particulars Amount
Step 1 Opening w.d.v. as on 01.04.2023 3,00,000
Step 2 - Purchased and put to use for less than 180 5,00,000
days
- Purchased and put to use for 180 days or 4,00,000
more
12,00,000
Step 3 Sold during the year (14,00,000)
Step 4 Wdv before depreciation (2,00,000)
Step 5 Depreciation allowance
- Assets purchased and put to use for less 0
than 180 days
- Assets purchased and put to use for 180 days 0
or more
Total depreciation allowance 0
Illustration: 5
Solution:
Particulars Amount
Step 1 Opening w.d.v. as on 01.04.2023 3,00,000
Step 2 - Purchased and put to use for less than 180 5,00,000
days
- Purchased and put to use for 180 days or more 4,00,000
12,00,000
Step 3 Sold during the year (10,00,000)
Step 4 Wdv before depreciation 2,00,000
Step 5 Depreciation allowance
- Assets purchased and put to use for less than 0
180 days
- Assets purchased and put to use for 180 days 0
or more
Total depreciation allowance 0
Illustration: 6
Mr. Ravin started his business on 01.04.2020 and purchased various plants and
machinery as given below:
He has purchased plant P1 on 01.04.2020 which was put to use on 01.06.2020 for
₹20,00,000.
He has purchased plant P2 on 01.05.2020 which was put to use on 01.07.2020 for
₹25,00,000.
Solution:
Particulars Amount
Opening written down value as on 01.04.2020 0
Add: Assets purchased and put to use for less than 0
180 days
Add: Assets purchased and put to use for 180 days
or more
- Purchased P1 on 01.04.2020, put to use on 20,00,000
01.06.2020
- Purchased P2 on 01.05.2020, put to use on 25,00,000
01.07.2020
- Purchased P3 on 01.06.2020, put to use on 25,00,000
01.09.2020
- Purchased P4 on 01.07.2020, put to use on 35,00,000 1,05,00,000
01.09.2020
Less: Assets sold during the year
- Sale P1 on 01.01.2021 11,00,000
Written down value P2, P3 and P4 on 31.03.2021 94,00,000
Particulars Amount
Written down value P2, P3 and P4 on 31.03.2021 79,90,000
Particulars Amount
Written down value P2, P3, P4, P5 and P6 on 1,10,46,500
01.04.2022
Add: Assets purchased and put to use for less than
180 days
- Purchased P7 on 01.06.2022, put to use on 10,00,000 10,00,000
10.12.2022
Add: Assets purchased and put to use for 180 days or 0
more
Less: Assets sold during the year
- Sale P2 on 31.03.2023 9,00,000 9,00,000
Written down value P3, P4, P5, P6 and P7 on 1,11,46,500
31.03.2023
Illustration: 7
Arman Ltd. is a manufacturing company. On April 1st, 2023, it owns plant A and plant B
(depreciation rate: 15 per cent; depreciated value of block being ₹2,40,000). Plant C
(depreciation rate: 15 per cent) is purchased by the company on June 10th, 2023 for
₹60,000 and it was used in the office premises. It is put to use on the same day.
Find out the tax consequences in the following different situations:
1. Plant B is destroyed by fire on January 25th, 2024. ₹10,000, being the compensation,
is paid by the insurance company on February 10th, 2024;
2. If the insurance compensation in situation (1) is ₹3,70,000;
3. Plant A, B And C is destroyed by fire on January 25th, 2024. Compensation paid by
insurance company on February 10th, 2024 is ₹20,000;
4. If the insurance compensation in situation (3) is ₹4 lakhs.
Solution:
Situation 1
Written down value of Plant A and Plant B as on 01.04.2023 2,40,000
Add: Plant C purchased on 10.06.2023 and
put to use on the same date 60,000
Less: Insurance claim of plant B (10,000)
Situation 2
Full value of consideration (Insurance claim) 3,70,000
Less: Written down value of Plant A and Plant B as on 01.04.2023 (2,40,000)
Less: Plant C purchased and put to use on 10.06.2023 (60,000)
Short term capital gain as per section 50 70,000
No depreciation is allowed
Situation 3
Full value of consideration (Insurance claim) 20,000
Less: Written down value of Plant A and Plant B as on 01.04.2023 (2,40,000)
Less: Plant C purchased and put to use on 10.06.2023 (60,000)
Short term capital loss as per section 50 2,80,000
No depreciation is allowed
Situation 4
Full value of consideration (Insurance claim) 4,00,000
Less: Written down value of Plant A and Plant B as on 01.04.2023 (2,40,000)
Less: Plant C purchased and put to use on 10.06.2023 (60,000)
Short term capital gain as per section 50 1,00,000
No depreciation is allowed
HW Question: 1
A newly qualified Chartered Accountant Miss Manavi, commenced practice and has
acquired the following assets in her office during F.Y. 2023-24 at the cost shown
against each item. Calculate the amount of depreciation that can be claimed from her
professional income for A.Y. 2024-25:
HW Question: 2
Mr. Babu has the following Assets which are eligible for depreciation at 15% on
Written Down Value (WDV) basis:
01.04.2020 WDV of plant ‘X’ and Plant ‘Y’ ₹2,00,000
10.12.2023 Acquired a new plant ‘Z’ for ₹2,00,000
22.01.2024 Sold plant ‘Y’ for ₹4,00,000
Expenditure incurred in connection with transfer ₹10,000
Compute eligible depreciation claim/ chargeable capital gain if any, for the Assessment
Year 2024-25.
HW Question: 3
Illustration: 8
Solution:
Please note that remaining 10% additional depreciation on new plant and machinery of
₹15,00,000 which is purchased and put to use for less than 180 days will be allowed in
subsequent P.Y. i.e. P.Y. 2024-25.
Illustration: 9
Solution:
Please note that no additional depreciation shall be allowed on new plant and machinery
of ₹15,00,000 which is purchased and installed in office.
Illustration: 10
Solution:
Please note that no additional depreciation shall be allowed on second hand plant and
machinery of ₹10,00,000.
Illustration: 11
Solution:
Particulars Amount
Opening w.d.v. as on 01.04.2023 30,00,000
- Purchased and put to use for less than 180 days 15,00,000
- Purchased and put to use for 180 days or more 10,00,000
55,00,000
Sold during the year 0
Balance Amount 55,00,000
Depreciation allowance
- Assets purchased and put to use for less than 180 1,12,500
days [15,00,000 * 15%/2]
- Assets purchased and put to use for 180 days or 6,00,000
more [40,00,000 * 15%]
Total depreciation allowance 7,12,500
Additional depreciation allowance
- New plant and machinery purchased and put to use 0
for less than 180 days
- New plant and machinery purchased and put to use 0
for 180 days or more
Total additional depreciation allowance 0
Note: Please note that no additional depreciation shall be allowed on P&M installed in
guest house and on motor vehicle.
Illustration: 12
Solution:
HW Question: 4
HW Question: 5
Mohan Ltd., a manufacturing company purchased the following new Plant and Machinery.
Date of Acquisition and Installation Actual Cost (in Crores)
25.05.2023 10.00
31.10.2023 22.00
From the above information compute the amount of depreciation available u/s 32,
additional depreciation, if any for the Assessment Years 2024-25
HW Question: 6
Laxman Ltd. is engaged in manufacturing and company has purchased new plant and
machinery during the previous year 2023-24:
1. ₹20.00 crore (purchased and put to use on 01.07.2023)
2. ₹30.00 crore (purchased and put to use on 01.11.2023)
Compute depreciation/ additional depreciation and also w.d.v as on 01.04.2024.
HW Question: 7
Rahul Ltd. an industrial undertaking has started manufacturing on 01.05.2023 and the
company has purchased the following assets:
1. Plant and machinery for use in the factory ₹30 lakhs, purchased on 01.07.2023 and
put to use on 15.07.2023.
2. Air-conditioner and generator for ₹2,00,000, purchased on 01.08.2023 and put to
use on 10.08.2023 for use in office premises.
3. One motor car for ₹10 lakhs for use of business, purchased on 01.09.2023 and put to
use on 10.09.2023.
4. One T.V. and one fridge for ₹50,000, purchased and put to use on 01.05.2023.
5. Furniture and fixture for use in factory ₹5,00,000, purchased and put to use on
01.06.2023.
Compute depreciation and additional depreciation allowance for A.Y. 2024-25.
HW Question: 8
Mr. Shyam is engaged in the business of generation and distribution of electric power.
He always claims depreciation on written down value. From the following details,
HW Question: 9
Compute the amount of depreciation and additional depreciation as per the Income-tax
Act, 1961 for the A.Y. 2024-25. Assume that all the assets were purchased by way of
account payee cheque.
Illustration: 13
Lights and Power Ltd. engaged in the business of generation of power, furnishes the
following particulars pertaining to P.Y. 2023-24. Compute the depreciation allowable
under section 32 for A.Y. 2024-25. The company has opted for the depreciation
allowance on the basis of written down value.
Particulars ₹
Solution:
Normal depreciation
40% on air pollution control equipment 1,00,000
Depreciation on plant and machinery put to use
For less than 180 days@ 7.5%
(2,00,000 + 3,25,000) 39,375
15% on ₹ 13,68,000 2,05,200
Additional depreciation
- Machinery Y (₹ 8,00,000 × 20%) 1,60,000
- Machinery Z (₹ 3,25,000 × 10%) 32,500
Total depreciation 4,37,075 1,00,000
Illustration: 14
Dr. Sagar purchased a residential building on 01.12.2020 for ₹12,00,000 and it was put
to use on the same date. Till 01.12.2022 the same was self-occupied as residence. On
this date, the building was brought into use for the purpose of his medical profession
(it was used as residential building). What would be the depreciation allowable for the
Assessment Year 2023-24?
Solution:
In this case notional depreciation shall be allowed as per section 43(1) and depreciation
allowable for the Assessment Year 2023-24 shall be computed in the manner given
below:
Illustration: 15
A car purchased by Dr. Soman on 10.08.2019 for ₹5,25,000 for personal use is brought
into professional use on 1.07.2022 by him, when its market value was ₹2,50,000.
Compute the actual cost of the car and the amount of depreciation for the assessment
year 2023-24 assuming the rate of depreciation to be 15%.
Solution:
As per section 43(1), the expression “actual cost” would mean the actual cost of asset
to the assessee. The purchase price of ₹5,25,000 is, therefore, the actual cost of the
car to Dr. Soman. Market value (i.e. ₹2,50,000) on the date when the asset is brought
into professional use is not relevant. Therefore, amount of depreciation on car as per
section 32 for the A.Y.2023-24 would be ₹78,750, being ₹5,25,000 x 15%.
146 | P a g e www.hakkseca.com/ 783-892-5588
TP: 9 Depreciation in case of succession or business
reorganization or amalgamation or demerger of companies
Illustration: 16
Mr. Lokesh carrying on business as proprietor converted the same into a limited
company by name Lokesh Cycles (P) Ltd. from 01.07.2023. The details of the assets are
given below:
₹
Block - I WDV of plant & machinery (rate of depreciation @ 15%) 12,00,000
Block - II WDV of building (rate of depreciation @ 10%) 25,00,000
The company Lokesh Cycles (P) Ltd. acquired plant and machinery in December 2023 for
₹ 10,00,000. It has been doing the business from 01.07.2023.
Solution:
Note: The depreciation on the plant which was purchased after conversion shall be
allowed to the company and further it is presumed that the plant was put to use on the
date of purchase.
HW Question: 10
M/s Hari & Co., a sole proprietary concern is converted into a company, Hari Co. Ltd.
With effect from December 29, 2023. The written down value of assets as on April 1st,
2023 is as follows:
Further, on October 15, 2023, M/s Hari & Co. purchased a plant for ₹1,00,000 (rate of
depreciation 15%) and it was put to use on the same date. After conversion, the
company added another plant worth ₹50,000 (rate of depreciation 15%) on 01.01.2024
and put to use on the same date.
Compute the depreciation available to (i) M/s Hari & Co. and (ii) Hari Co. Ltd. for A.Y.
2024-25.
HW Question: 11
Rohan Ltd. has a block of assets carrying 15% rate of depreciation, whose written down
value on 01.04.2023 was ₹40 lacs. It purchased another asset of the same block on
01.11.2023 for ₹14.40 lacs and put to use on the same day. Rohan Ltd. was amalgamated
with Mohan Ltd. with effect from 01.01.2024. You are required to compute the
depreciation allowable to Rohan Ltd. & Mohan Ltd. for the previous year ended on
31.03.2024.
Illustration: 17
Solution:
Illustration: 18
Solution:
The deduction available under section 35 for scientific research will, therefore, be:
Particulars ₹
(a) Land Nil
(b) Building 25,00,000
(c) Revenue expenses of last 3 years 2,20,000
(d) Capital expenditure of last 3 years: Plant and machinery 5,00,000
(e) Current year revenue expenditure 1,80,000
Deduction under section 35 34,00,000
Illustration: 19
Kundan Ltd. was incorporated on 01.01.2023 for manufacture of tyres and tubes for
motor vehicles. The manufacturing unit was set up on 01.05.2023. The company
commenced its manufacturing operations on 01.06.2023. The total cost of the plant and
machinery installed in the unit is ₹55 crore. The said plant and machinery included
second hand plant and machinery bought for ₹20 crore and new plant and machinery for
scientific research relating to the business of the assessee acquired at a cost of ₹15
crore.
Compute the amount of depreciation allowable under section 32 of the Income-tax Act,
1961 in respect of the assessment year 2024-25.
Solution:
Particulars ₹ in crore
Total cost of plant and machinery 55.00
Illustration: 20
Mr. Gamma, a proprietor started a business of manufacture of tyres and tubes for
motor vehicles on 01.01.2023. The manufacturing unit was set up on 01.05.2023. He
commenced his manufacturing operations on 01.06.2023. The total cost of the plant and
machinery installed in the unit is ₹120 crores. The said plant and machinery included
second hand plant and machinery bought for ₹20 crores and new plant and machinery
for scientific research relating to the business of the assessee acquired at a cost of
₹15 crores.
Compute the amount of depreciation allowable under section 32 of the Income-tax Act,
1961 in respect of the assessment year 2024-25. Assume that all the assets were
purchased by way of account payee cheque and Mr. Gamma has not opted for the
provisions of section 115BAC.
Solution:
Computation of depreciation allowable for the A.Y. 2024-25 in the hands of Mr.
Gamma
HW Question: 13
A Ltd. which is engaged in manufacturing, furnishes the following particulars for the
P.Y. 2023-24. Compute the deduction allowable under section 35 for A.Y. 2024-25,
while computing its income under the head “Profits and gains of business or profession”.
Particular ₹
1 Amount paid to Indian Institute of Science, Bangalore, a notified 1,00,000
research organization for scientific research
2 Amount paid to IIT, Delhi for an approved scientific research 2,50,000
Programme
3 Amount paid to X Ltd, a company registered in India which has as 4,00,000
its main object
4 Expenditure incurred on in-house research and development
facility as approved by the prescribed authority
(a) Revenue expenditure on scientific research 3,00,000
(b) Capital expenditure (including cost of acquisition of land 7,50,000
₹5,00,000) on scientific research
HW Question: 14
Mr. Abhimanyu has furnished the following particulars relating to payments made and
expenditure incurred towards scientific research for the year ended 31.3.2024:
Illustration: 21
Solution:
HW Question: 15
HW Question: 16
XYZ Ltd. commenced operations of the business of a new three-star hotel in Madurai,
Tamil Nadu on 01.04.2023. The company incurred capital expenditure of ₹50 lakh
during the period January, 2023 to March, 2023 exclusively for the above business,
and capitalized the same in its books of account as on 1st April, 2023. Further, during
the P.Y. 2023-24, it incurred capital expenditure of ₹2 crore (out of which ₹1.50 crore
was for acquisition of land) exclusively for the above business.
Compute the income under the head “Profits and gains of business or profession” for
the A.Y. 2024-25, assuming that XYZ Ltd. has fulfilled all the conditions specified for
claim of deduction under section 35AD. The profits from the business of running this
hotel (before claiming deduction under section 35AD) for the A.Y. 2024-25 is ₹25
lakhs.
Assume that the company also has another existing business (specified business) of
running a four-star hotel in Coimbatore, which commenced operations 2 years back, the
profits from which are ₹120 lakhs for the A.Y. 2024-25.
Illustration: 22
Lake Ltd. an Indian company has incurred expenditure before the commencement of
business as under:
1. Expenditure on advertisements ₹3,00,000
2. Expenditure on preparation of project report and the report was
prepared by a concern which is approved by the Board ₹85,000
3. Expenditure in connection with travelling and stay in hotels ₹45,000
4. Expenditure on drafting and printing of memorandum and articles
of associations ₹4,00,000
All the above expenditures have been debited to the profit and loss account and the
company has computed income to be ₹7 lakhs.
The company has commenced its business on 01.06.2023.
Company’s project cost is ₹50 lakhs and capital employed is Rs. 57 lakhs.
154 | P a g e www.hakkseca.com/ 783-892-5588
Compute company’s Tax Liability for Assessment Year 2024-25 under old provisions.
Solution:
WN: 1
Eligible expenditure under section 35D
1. Expenditure on preparation of project report 85,000
2. Expenditure on drafting and printing of memorandum and articles
of associations 4,00,000
Total = 4,85,000
Expenditure allowed under section 35D cannot exceed 5% of the
capital employed 57,00,000 x 5% = 2,85,000
Instalment allowed 2,85,000/5 = 57,000
Expenditure disallowed = 4,85,000 – 57,000 = 4,28,000
Illustration: 23
Aman Co. Ltd. paid ₹120 lakhs as compensation as per approved Voluntary Retirement
Scheme (VRS) during the financial year 2023-24.
How much is deductible under section 35DDA for the assessment year 2024-25?
Solution:
(7) Employer’s contribution to the account of the employee under a Pension Scheme
referred to in section 80CCD [Section 36(1)(iva)]
Illustration: 24
Manit Ltd. contributes 20% of basic salary to the account of each employee under a
pension scheme referred to in section 80CCD. Dearness Allowance is 40% of basic
salary and it forms part of pay of the employees.
Compute the amount of deduction allowable under section 36(1)(iva), if the basic salary
of the employees aggregate to ₹10 lakhs. Would disallowance under section 40A(9) be
attracted, and if so, to what extent?
Solution:
Particulars
Basic Salary 10,00,000
Dearness Allowance@40% of basic salary [DA forms part of pay] 4,00,000
Salary for the purpose of section 36(1)(iva) (Basic Salary + DA) 14,00,000
Actual contribution (20% of basic salary i.e., 20% of ₹10 lakh) 2,00,000
Less: Permissible deduction under section 36(1)(iva) (10% of basic
salary plus dearness pay = 10% of ₹14,00,000 = ₹1,40,000) 1,40,000
Excess contribution disallowed under section 40A(9) 60,000
Illustration: 25
ABC Limited is a company engaged in the business of biotechnology. The net profit of
the company for the financial year ended 31.03.2024 is ₹15,25,890 after debiting the
following items:
1. Purchase price of raw material used for the purpose of in-house
research and development 1,80,000
2. Purchase price of asset used for in-house research and
development
156 | P a g e www.hakkseca.com/ 783-892-5588
(1) Land 5,00,000
(2) Building 3,00,000
3. Expenditure incurred on advertisement in the souvenir published
by a political party 75,000
Compute the income under the head “Profits and gains of business or profession” for
the A.Y. 2024-25 of ABC Ltd.
Solution:
Computation of income under the head “Profits and gains of business or profession”
for the A.Y. 2024-25
During the financial year 2023-24, the following payments/ expenditure were made/
incurred by Mr. Vishal, a resident individual (whose turnover during the year ended
31.03.2022 was ₹99 lacs):
(i) Interest of ₹12,000 was paid to ABC & Co., a resident partnership firm, without
deduction of tax at source;
(ii) Interest of ₹4,000 was paid as interest to Mr. John, a non-resident, without
deduction of tax at source;
(iii) ₹3,00,000 was paid as salary to a resident individual without deduction of tax at
source;
(iv) He had sold goods worth ₹5 lacs to Mr. Anand. He paid Commission of ₹15,000 to
Mr. Sohil on 02.07.2022. In none of these transactions, tax was deducted at source.
Briefly discuss whether any disallowance arises under the provisions of section 40(a) of
the Income tax Act, 1961.
(i) Since, turnover of Mr. Vishal is less than 100 lakhs in the preceding year hence
expenditure is allowed even if tax has not been deducted at source.
(ii) In the case of interest paid to a non-resident, there is obligation to deduct tax at
source under section 195, hence non-deduction of tax at source will attract
disallowance.
(iii) Disallowance under section 40(a) is attracted for failure to deduct tax at source
under section 192 from salaries. Therefore, 70% of salary i.e. ₹3,00,000 x 70% =
₹2,10,000 shall be allowed in previous year 2023-24 and balance i.e. ₹90,000 is
disallowed.
(iv) Since, turnover of Mr. Vishal is less than 100 lakhs in the preceding year hence
expenditure is allowed even if tax has not been deducted at source.
M/s ABC Ltd., submits the following details of expenditures pertaining to the financial
year 2023-24:
(i) Payment of professional fees to Mr. M of ₹50,000. Tax not deducted at source.
(ii) Interior works done by Mr. H for ₹2,00,000 on a contract basis. Payment made in
the month of March 2024. Tax deducted in March 2024, was paid on 30.06.2024.
(iii) Factory Rent paid to Mrs. R of ₹15,00,000. Tax deducted at source and paid on
01.11.2024.
(iv) Interest paid on Fixed Deposits of ₹2,00,000. Tax deducted on 31.12.2023 and paid
on 28.09.2024.
(v) Payment made to M/s G & Co. towards import of Raw Materials of ₹25,00,000. No
tax was deducted at source. The supplier G & Co. is located in London.
Examine the above with reference to allowability of the same in the Assessment Year
2024-25 under the Income Tax Act, 1961. Your answer must be with reference to
Section 40(a) read with relevant tax deduction at source provisions.
Illustration: 27
X & Y, a partnership firm consisting of two partners, reports a net profit of ₹7,00,000
before deduction of the following items:
(1) Salary of ₹20,000 each per month payable to two working partners of the firm (as
authorized by the deed of partnership).
(2) Depreciation on plant and machinery under section 32 (computed) of ₹1,50,000.
(3) Interest on capital at 15% per annum (as per the deed of partnership). The amount
of capital eligible for interest of ₹5,00,000
Compute:
Solution:
(i) Computation of Book profit under section 40(b) of Income Tax Act 1961
HW Question: 18
XYZ are the partners in a firm with profit sharing ratio 5:3:2 and profit and loss
account of the partnership firm is as given below:
Additional information:
1. Capital contributed by Mr. X is ₹5,00,000 and by Mr. Y ₹3,00,000 and by Mr. Z
₹2,00,000.
2. Salary paid to Mr. X is ₹3,00,000 and to Mr. Y is ₹2,50,000 and to Mr. Z is
₹1,50,000.
3. The partnership firm has brought forward business loss for assessment year 2021-
22 amounting to ₹1,00,000.
4. Municipal tax was paid on 01.11.2024
Illustration: 28
Mr. Shyamsundar (age 82 years) has one house which is 50% in business/ profession
and 50% is let out @ 10,000 p.m. and municipal taxes for the entire house are ₹7,000
which were paid on 10.04.2024 and business income of Mr. Shyamsundar before
debiting any expense of house property is ₹7,80,000. Compute tax liability for the
Assessment Year 2024-25 under old provisions.
➢ Conversion of interest into loan/ borrowings/ debentures etc.: Where any sum
payable as interest on any loan is converted into loan/ borrowing/ debenture etc.,
the interest so converted (not actually paid) shall not be allowed as deduction. Such
converted interest shall be allowed as deduction when such portion of loan has been
repaid.
Illustration: 29
₹
(i) Andhra Pradesh State Financial Corporation (P.Y. 2022-23 & 2023-24 15,00,000
161 | P a g e www.hakkseca.com/ 783-892-5588
(ii) Indian Bank (P.Y. 2023-24) 30,00,000
45,00,000
Both APSFC and Indian Bank, while restructuring the loan facilities of Hari during the
year 2023-24, converted the above interest payable by Hari to them as a loan
repayable in 60 equal installments. During the year ended 31.3.2024, Hari paid 5
installments to APSFC and 3 installments to Indian Bank.
Hari claimed the entire interest of ₹ 45,00,000 as an expenditure while computing the
income from business of purchase and sale of agricultural commodities. Examine
whether his claim is valid and if not what is the amount of interest, if any, allowable.
Solution:
According to section 43B, any interest payable on the term loans to specified financial
institutions and any interest payable on any loans and advances to, interalia, scheduled
banks shall be allowed only in the year of payment of such interest irrespective of the
method of accounting followed by the assessee. Where there is default in the payment
of interest by the assessee, such unpaid interest may be converted into loan. Such
conversion of unpaid interest into loan shall not be construed as payment of interest
for the purpose of section 43B. The amount of unpaid interest so converted as loan
shall be allowed as deduction only in the year in which the converted loan is actually
paid.
In the given case of Hari, the unpaid interest of ₹15,00,000 due to APSFC and of
₹30,00,000 due to Indian Bank was converted into loan. Such conversion would not
amount to payment of interest and would not, therefore, be eligible for deduction in
the year of such conversion. Hence, claim of Hari that the entire interest of
₹45,00,000 is to be allowed as deduction in the year of conversion is not tenable. The
deduction in the year of conversion is not tenable. The deduction shall be allowed only
to the extent of repayment made during the financial year. Accordingly, the amount of
interest eligible for deduction for A.Y. 2024- 25 shall be calculated as follows:
HW Question: 19
Mr. Naman has computed his income of ₹3,50,000 and some of the amounts debited to
the profit and loss account are as given below:
1. Household expense of ₹5,000
2. Rent for own building of ₹1,20,000 (half of the building is self-occupied and balance
half in business use).
162 | P a g e www.hakkseca.com/ 783-892-5588
3. Municipal tax of the building of ₹3,000 (amount was paid on 01.04.2024)
4. Expenditure on repairs of the building of ₹4,000.
5. Premium paid for insurance of the building of ₹2,000.
6. Mr. X has purchased one motor car for ₹3,00,000 on 01.01.2024 and it was put to
use on the same date. The car was used for personal purpose as well as official use
(50% official and 50% personal). Assessee has also debited petrol expenses of ₹5,000.
7. He has debited ₹20,000 being the amount invested in public provident fund.
Compute his tax liability for the Assessment Year 2024-25 under normal provisions of
the tax.
Illustration: 30
Mr. Ashu is engaged in a business with turnover ₹1,70,00,000 (all payments received by
account payee cheque, bank draft or through electronic clearing) and expenses incurred
in connection with earning of income are ₹1,60,00,000. He has LTCG ₹5,00,000. He has
brought forward business loss of ₹1,00,000 of P.Y. 2019-20. Compute his Income and
Tax Liability for previous year 2023-24, in two situations -
(i) He has opted for section 44AD.
(ii) He has not opted for section 44AD.
Solution:
Note: The Assessee shall be exempt from maintaining books of accounts and also from
Audit.
Note: The Assessee shall be liable to maintain books of accounts and also liable to
Audit.
HW Question: 20
Mr. Mohan engaged in Retails Trade, reports a turnover of ₹58,50,000 (all payments
received in account payee cheque) for the financial year 2023-24. His income from the
said business as per books of account is computed at ₹2,90,000. Retail trade is the only
source of income for Mr. Mohan.
(i) Is Mr. Mohan eligible to opt for presumptive determination of his income chargeable
to tax for the Assessment Year 2024-25?
(ii) Is so, determine his income from retail trade as per the applicable presumptive
provision.
HW Question: 21
Mr. Naman is engaged in the business of producing and selling TV. During the previous
year 2023-24, his turnover was ₹1.75 crores. He opted for paying tax as per
presumptive taxation scheme laid down in section 44AD. He has no other income during
the previous year. Is he liable to pay advance tax and if so, what is the minimum amount
of advance tax to be paid and the due date for payment of such advance tax assuming
that whole of the turnover represents cash receipts assuming that he has not opted
for section 115BAC?
Illustration: 31
Mr. Aman is engaged in specified profession and has gross receipt ₹42,00,000. He has
Long term Capital Gain ₹7,00,000 and brought forward business loss ₹30,000 of A.Y.
2020-21. He invested ₹20,000 in LIC in his own name. Compute his Tax Liability for the
Assessment Year 2024-25. He has opted for Section 44ADA.
Solution:
Illustration: 32
Mr. Mohan retired from Govt. service in March 2023. He got ₹20,00,000 on account of
retirement benefits. Out of the aforesaid sum, he purchased on 23rd April 2023 a few
motor vehicles and got their delivery on that date.
The particulars of the vehicles are given below–
Vehicle Number Cost of the vehicle
Heavy goods vehicle (15 ton) 2 ₹9,00,000
Medium goods vehicle (8 ton) 4 ₹4,50,000
Light commercial Vehicle (4 ton) 3 ₹3,20,000
He started plying the vehicles from 04.06.2023. On an average every vehicle remains
off the road for about a week for repairs and maintenance. He maintains a rough
record of the receipts and outgoings which is given below –
Receipts ₹3,70,000
Less: Expenses (Excluding depreciation and salaries to Mr. Soham) (₹60,000)
₹3,10,000
You are required to compute the total income of Mr. Mohan from the business of goods
carriage for the previous year 2023-24.
Solution:
HW Question: 22
An assessee owns a heavy commercial vehicle having gross vehicle weight of 15 ton each
for 9 months 15 days, a medium goods vehicle having gross vehicle weight of 8 ton for 9
Mr. Jatin (aged 38) owned 6 heavy goods vehicles having gross vehicle weight of 16 ton
(16,000 Kg) each as on 01.04.2023. He acquired 2 more light goods vehicles having
gross vehicle weight of 8 ton (8,000 Kg) each on 01.07.2023. He is solely engaged in the
business of plying goods vehicles on hire since financial year 2019-20.
He did not opt for presumptive provision contained in section 44AE for the financial
year 2022-23. His books were audited under section 44AB and the return of income
was filed on 05.08.2023. He has unabsorbed depreciation of ₹70,000 and Business loss
of ₹1,00,000 for the financial year 2022-23. Following further information is provided
to you:
(i) Paid medical insurance premium of ₹23,000 for his parents (both aged above 70) by
means of bank demand draft.
(ii) Paid premium on life insurance policy of his married daughter of ₹25,000.
(iii) Repaid principal of ₹40,000 and interest of ₹15,000 to Canara Bank towards
education loan of his daughter, who completed B.E. two years ago. She is employed
after completion of her studies.
Assuming that Mr. Jatin has opted for presumptive provision contained in section 44AE
of the Income-tax Act, 1961, compute the total income of Mr. Jatin for the A.Y. 2024-
25.
HW Question: 24
Mr. Akshay is engaged in the business of plying goods carriages. On 1st April, 2023, he
owns 10 trucks (out of which 6 are heavy goods vehicles having capacity of 18 ton and
balance 4 trucks having capacity of 8 ton). On 2nd May, 2023, he sold two of the heavy
goods vehicles and purchased two light goods vehicles having capacity of 8 ton on 6th
May, 2023. Those new vehicles could however be put to use only on 15th June, 2023.
Compute the Total Income and Tax Liability of Mr. Akshay for the Assessment Year
2024-25, taking note of the following data in two situations i.e. presumptive basis and
normal basis.
Freight charges collected 9,90,000
Less: operational expenses 5,25,000
Depreciation as per sec 32 1,85,000
Other office expenses 15,000
(7,25,000)
Net Profit 2,65,000
Other business and non-business income 1,00,000
Illustration: 33
The Profit & Loss account of Mr. X for the previous year ending 31.03.2024 is as given
below:
Solution:
Question: 1 [PGBP]
The following is the receipts and payments account of a medical practitioner for the
year ending 31.03.2024
Additional information:
1. A cash payment of ₹75,000 was given to him by a patient in appreciation of his
medical services but was not recorded in books.
2. Flat was purchased on 01.04.2023 and was self occupied for residence for a month
from the date of its purchase. Thereafter it was let out @ ₹5,500 p.m., the municipal
value of the flats is ₹66,000 p.a. and municipal taxes assessed, though not paid, is
₹4,500.
3. One–third of motor car expenses relate to his personal use. Depreciation on car
allowable under Income Tax Act for professional use is ₹12,000.
4. The rate of depreciation on surgical equipment is 15%. The written down value of
equipment on 01.04.2023 is ₹60,000. He sold some of the equipment for ₹30,000
during the year. New equipment was purchased on 01.11.2023 for ₹90,000 and was put
to use on the same date.
Compute his Total Income and Tax Liability for the Assessment Year 2024-25 under
old tax provisions.
Mr. Jonny is an advocate in Delhi High Court. He keeps his books on cash basis. His
receipts and payments account for the financial year 2023-24 is given below:
170 | P a g e www.hakkseca.com/ 783-892-5588
Receipts Amount Payments Amount
₹ ₹
Balance b/d 49,200 Rent Paid for Building 1,44,000
Consultancy fee 55,35,500 Office expenses 46,000
Remuneration from 7,000 New car purchased and put to 3,00,000
university as evaluator of use on 01.05.2023
LLB exams
Sale proceeds of residential 5,00,000 Computer purchased and put 50,000
house (it was purchased on to use on 01.04.2023
01.07.2022 for ₹3,00,000)
Salary from law faculty for 45,000 Legal books purchased 30,000
working as part time
lecturer
Car expenses 42,000
Advance Income tax paid 22,000
Electricity and water charges 16,000
for the entire house
Son’s college tuition fee paid 54,000
Gift to daughter 25,000
Life insurance premium paid 12,000
on own life (sum assured
₹50,000)
Balance c/f 53,95.700
61,36,700 61,36,700
Additional information:
1. On 31.03.2024 legal fees outstanding amounted to ₹22,000
2. Rent is payable @ ₹12,000 p.m.
3. 70% of the use of the car is for official purpose and 30% for personal purpose.
4. Legal books for ₹12,000 was purchased on 01.05.2023 and put to use on the same
date and for ₹18,000 on 01.11.2023 and put to use on the same date.
5. Half of the house taken on rent is being used for residential purposes.
Compute the Total Income and Tax Payable of Mr. Jonny for the Assessment Year
2024-25 under old tax provisions.
Question: 3
ABC Ltd. has net profits of ₹7,00,000 after debiting municipal taxes of ₹12,000
relating to the previous year 2023-24, which were paid on 20.09.2024. Municipal taxes
are related to a building which is owned by the company, the ground floor and first
floor (which is 2/3rd of the complete building) was being used by company. The
company has debited market rent of ₹20,000 p.m. to the profit & loss account for using
the building and credited rent of ₹10,000 p.m. to the profit & loss account for the
Question: 4
Mr. Akash is a Chartered Accountant and has prepared the following income and
expenditure account as on 31.03.2024.
You are required to compute his Total Income and Tax Liability (old provisions) for the
Assessment Year 2024-25 considering the following points –
1. The car is used equally for official and personal purposes.
2. ₹1,500 for domestic servant’s salary is included in employee’s salary.
3. Books were purchased on 01.09.2023 and were put to use on the same date.
4. Payment of stationery ₹20,500 was made by a bearer cheque and ₹ 500 was paid in
cash.
Question: 5
Question: 6
Mr. Ashok (age 79 years) is running a shop at Chandni Chowk and has submitted the
following profit and loss account for the Assessment Year 2024-25.
Additional information:
1. Purchases includes purchase of ₹1,00,000 from a relative and it is excessive by
₹20,000 and payment was made in cash.
2. Salary includes ₹14,000 paid outside India without deducting tax at source and
₹7,000 were paid to one of the relatives which is more than the market rate by ₹1,000.
3. Business is being run in a commercial building which is owned by the assessee and its
written down value on 01.04.2023 is ₹10 lakhs and addition was made to the building on
01.01.2024 and brought into immediate use and no depreciation has been debited to
profit and loss account.
Mr. Ashok has not opted for presumptive taxation of Income u/s 44AD. Compute his
Total Income and Tax Liability for the Assessment Year 2024-25 under old tax
provisions.
Question: 7
Mrs. Verma submitted the following profit & loss account for the Assessment Year
2024-25.
Question: 8
ABC Ltd. submits the profit & loss account for the year ending 31st March 2024.
Determine the Total Income and Tax Liability of company for the Assessment year
2024-25 under old tax provisions.
Question: 9
From the following profit and loss account of Mr. Rahman for the year ending March
31st, 2024, compute his Total Income and Tax Liability for the Assessment Year 2024-
25.
Additional information:
(i) Purchases include ₹1,10,000 paid in cash to a cultivator for purchase of an
agricultural produce.
(ii) Purchases also include ₹15,000 paid by way of compensation to a supplier as the
assessee was unable to take the delivery of goods due to lack of storage space and
finance.
(iii) Opening stock was overvalued by 25% and closing stock was undervalued by 25%.
(iv) Salary includes ₹ 25,000 paid as customary bonus on the occasion of Diwali over and
above the bonus payable under the Payment of Bonus Act, 1965.
(v) Rent, rates and taxes include:
(a) ₹3,000 on account of municipal taxes for property let out and payment was made on
31.03.2024.
(b) Penalty imposed by GST department of ₹25,000.
(vi) Provision for Gratuity is on actuarial basis.
(vii) Mrs. Rahman is a law graduate and actively working in the assessee firm and salary
paid is reasonable.
(viii) He has invested ₹1,00,000 in equity shares of infrastructure development
companies notified u/s 80C.
(ix) He has loss from owning and maintaining of race horses ₹20,000.
Mr. Rahman has not opted for presumptive taxation of Income u/s 44AD.
Question: 10 [PGBP]
Net profit as per the profit and loss account of Mr. Rakesh is ₹7,70,000 for the year
ending 31st March, 2024.
The following information is noted from the accounts:
(a) Advertisement expenditure debited to profit and loss account includes the
following:
(i) Expenditure incurred outside India: ₹56,000 (Tax has been deducted at source and
paid during the year)
(ii) Articles presented by way of advertisement (60 articles cost of each being ₹700,
176 | P a g e www.hakkseca.com/ 783-892-5588
and 36 articles cost of each being ₹1,500);
(iii) ₹20,000 being the cost of advertisement which appeared in a newspaper owned by
a political party;
(iv) ₹14,400 being capital expenditure on advertisement; (eligible for dep. @ 25%)
(v) ₹9,000 paid in cash
(vi) ₹9,000 paid to a concern in which Rakesh has substantial interest (amount is
excessive to the extent of ₹1,800)
(b) Out of salary to the employees debited to the profit and loss account:
(i) ₹60,000 is employee’s contribution to the recognized provident fund, ₹47,500 of
which is credited in the employee’s account in the relevant fund before the due date
for provident fund;
(ii) ₹58,000 is bonus which is paid on 13th November, 2024;
(iii)₹44,000 is commission which is paid on 1st December, 2024;
(iv) ₹25,000 is incentive to workers, which is paid on 10th December, 2024.
(v) ₹46,000 is paid outside India in respect of which tax is not deducted at source;
(vi) ₹6,000 being capital expenditure for promoting family planning amongst employees;
and
(vii) ₹55,000 being entertainment allowance given to employees.
(c) Entertainment expenses debited to profit and loss account is ₹ 12,000.
Determine the Total Income and Tax Liability of Mr. Rakesh for the Assessment Year
2024-25 under old tax regime.
Question: 11 [PGBP]
The profit and loss account of Mr. Mahesh for the year ending 31st March, 2024
discloses net profit of ₹3,90,000. Travelling expenses debited to the profit and loss
account include the following:
(i) ₹1,80,000 being expenditure incurred on a foreign tour, out of which ₹15,000 is
incurred in Indian currency and ₹1,65,000 in foreign currency for a visit of 8 days to
Germany; out of 8 days, 2 days are utilized by Mr. Mahesh for attending personal work.
(ii) ₹45,000 being expenditure on air–fare in India by a sales manager.
(iii) ₹6,500 incurred for purchasing a machine for factory. (Put to use for more than
180 days)
(iv) ₹66,000 being hotel expenses as follows:
(a) 4 days visit to Madras: ₹18,000
(b) 3 days visit to Bombay: ₹8,000
(c) 17 days visit to Bangalore: ₹40,000
Salary to employees include the following:
(1) Own salary of Mr. Mahesh: ₹ 26,000
(2) Commission on purchases to employees
(which is actually paid on 1st November, 2024) ₹42,000
Find out the Total Income and Tax Liability of Mr. Mahesh for the Assessment Year
2024-25 under old tax provisions.
From the following profit and loss account of Mr. X for the year ended 31st March,
2024, compute his Total Income and Tax Liability for the Assessment Year 2024-25
under old tax provisions:
Additional information:
(i) Purchases include:
(a) Purchase of ₹ 1,00,000 from a relative (market price ₹80,000) and payment was
made in cash.
(b) Purchase of ₹25,000 being the products manufactured without aid of power in a
cottage industry and the payment was made to its producer and payment was made in
cash.
(c) Purchases of ₹35,000 from a person who is residing in a village having no bank and
payment was made in cash.
The profit and loss account of ABC Ltd. for the year ended 31st March, 2024 showed a
net profit of ₹8,00,000 and some of the debits and credits are as given below:
(A) Debit side of profit and loss account included the following:
(i) The depreciation provided in the books of ₹60,000, however the amount computed
under the Income Tax Act of ₹1,20,000.
(ii) ₹30,000 was paid to the company’s lawyer for arguing appeals of the company
before the Income Tax Appellate Tribunal against levy of penalty for some earlier
cases where appeals have been dismissed by the tribunal.
(iii) ₹2,000 being fine imposed by the municipality for violating their regulations.
(iv) Provision for Income Tax of ₹35,000.
(B) The credit side of the profit and loss account included the following:
(i) Income from units of UTI of ₹35,000
(ii) Dividend from Indian company of ₹20,000
(C) It is also observed that both the opening stock of ₹90,000 and closing stock of
₹1,08,000 are undervalued by 10% on cost.
Compute the Total Income and Tax Liability of the company for the Assessment Year
2024-25 under old tax provisions.
Question: 14 [PGBP]
ABC Ltd., a manufacturing company, which maintains accounts under mercantile system
has disclosed a net profit of ₹12.50 lakhs for the year ending 31st March, 2024. You
are required to compute the Total Income and Tax Liability of the company for the
Assessment Year 2024-25, after considering the following information, duly explaining
the reasons for each item of adjustment:
(i) Advertisement expenditure includes the sum of ₹60,000 paid in cash to the sister
concern of a director, the market value of which is ₹52,000.
(ii) Repairs of plant and machinery includes ₹1.80 lakhs towards replacement of worn-
out parts of machineries.
(iii) A sum of ₹6,000 on account of liability foregone by a creditor has been taken to
general reserve. The same was charged to the revenue account in the Assessment Year
2021-22.
(iv) Sale proceeds of import entitlements amounting to ₹1 lakh has been credited to
profit and loss account, which the company claims as capital receipt not chargeable to
income tax.
(v) The company has donated ₹2,00,000 to National Urban Poverty Eradication Fund.
The amount has been debited to the profit and loss account.
(vi) Being also engaged in the biotechnology business, the company incurred the
following expenditure on in-house research and development as approved by the
prescribed authority:
(a) Research equipment purchased for ₹1,50,000.
(b) Remuneration paid to scientists for ₹50,000.
Mr. Sunil is a leading lawyer of Mumbai. He deposits in the bank all the receipts and
always pays all the expenses by cheque. The analysis of his bank account for the year
ended 31st March, 2024 is as under:
Mr. Sunil has not opted for presumptive taxation of Income u/s 44ADA. Compute his
Total Income, Tax Liability and Tax Payable (old tax regime) after taking into account
the following information:
(i) 10% of the motor car expenses relate to personal use.
(ii) Salaries include employer’s contribution to Recognised Provident Fund of ₹18,000
which was credited on 01.07.2024.
(iii) Mr. Sunil stays in his house, the gross annual value of which is ₹16,800.
Following are the expenses which have been included in the above account in respect of
this house:
(a) Municipal taxes: ₹2,000.
Question: 16 [PGBP]
(i) Gross total income of Mrs. Bansal, aged 60, a resident of Delhi for the financial year
2023-24 is ₹4,00,000. It includes an income of ₹20,000 from the business of dealing in
shares on which she has paid securities transaction tax of ₹1,800 and it has not been
debited to the profit and loss account. She has also deposited ₹10,000 in her public
provident fund account with the State Bank of India.
Compute her Tax Liability for the Assessment Year 2024-25 under old provisions.
(ii) ABC Ltd., a domestic company, is engaged in the business of sale/purchase of shares
and the company has computed its income ₹11,00,000 after debiting securities
transaction tax of ₹1,85,000.
Compute Tax Payable by the company for the Assessment Year 2024-25.
(iii) Mr. Rohan is engaged in the business of sale/purchase of shares and he has
computed its income ₹18,00,000 after debiting securities transaction tax of ₹2,10,000.
Compute Tax Payable by Mr. Rohan under old provisions.
Determine the previous year in which the expenditure is allowable in the following cases
(TDS is supposed to be deducted with regard to all the payments and all the payments
are in India):
(i) ABC Ltd. has made payment of interest on 10th, June 2023 and has deducted tax at
source on the same date and has deposited the amount on 08.07.2023.
(ii) The company has paid commission on 10.03.2024 and has deducted tax on the same
date but it was paid on 05.04.2024.
(iii) The company has paid fees for professional services on 31.03.2024 and deducted
tax at source on the same date but the tax was paid on 07.04.2024.
(iv) The company has paid to a contractor on 31.03.2024 and tax was deducted on the
same date but it was paid on 01.06.2024.
(v) The company has paid technical fees on 01.01.2024 and no tax has been deducted at
source.
(vi) The company has paid brokerage on 01.04.2024 and has deducted the tax on the
same date and has paid it on 07.04.2024.
Following is the profit & Loss account of Mr. Aman, a dealer in shares and securities
for the year ended on 31st March, 2024:
Compute Total Income and Tax Liability of Mr. Aman for Assessment Year 2024-25
under old tax provisions.
Profit and loss account of Mr. Kishan for the previous year 2023-24 is as under:
Additional information:
(i) Salaries and wages include the sum of ₹1,60,000 paid to Mr. Kishan
(ii) Payment of interest includes:
(a) Interest to his major son ‘Ram’ amounting to ₹30,000 @ 15% on a deposit of
Question: 20 [PGBP]
Mr. X has computed his income under the head business/profession ₹10,00,000 and he
has debited the following amount.
(1) Cost of goods sold ₹7,00,000, out of which ₹4,00,000 paid to a relative for
purchasing stock and its market value is ₹3,00,000 and Mr. X has paid ₹2,00,000 by
account payee cheque and ₹2,00,000 in cash.
(2) He has debited ₹45,000 in connection with purchase of a computer which was
purchased on 27.10.2023 and was put to use on the same date and payment was made in
cash.
(3) He has purchased one generator from his relative for ₹45,000 and payment was
made in cash and market value was ₹40,000 and it was purchased on 01.10.2023 and was
put to use on 07.10.2023.
(4) He has paid advance tax being income tax of ₹45,000 on 01.10.2023.
(5) He has paid ₹21,000 to a Chartered Accountant for filing a return of income, out of
which ₹9,500 was paid in cash and balance by an account payee cheque.
(6) He has donated ₹ 20,000 to an approved research association and research work
taken up by such association is not related to the business/ profession of assessee.
(7) He has purchased household furniture for ₹12,000 for personal use.
(8) He has paid ₹20,000 in cash in connection with his medical treatment.
(9) Salary paid to the proprietor is ₹36,000.
(10) Interest on capital is ₹9,000.
(11) He has invested ₹25,000 in National Saving Certificate.
(12) He has invested ₹10,000 in public provident fund in the name of his minor child.
(13) He has debited rent of ₹35,000 in connection with his own building which is being
used in his business/profession.
(14) Opening stock debited is ₹4,50,000 which is overvalued by 10%.
(15) He has incurred ₹7,000 on printing and distribution of diaries and calendars.
Compute his Tax Liability for the Assessment Year 2024-25 under old tax regime.
Mr. Raju, aged 75 years, has submitted his profit and loss account for the year ending
31.03.2024 as given below:
Additional information:
1. Opening and closing stocks are undervalued by 10%.
2. Franchises were purchased on 01.07.2023 and were put to use on 03.10.2023.
3. Advertisement expenditure relates to a neon sign board which was purchased and put
to use on 01.08.2023.
4. Office building has written down value of ₹56,00,000 as on 01.04.2023 and addition
was made to the building by constructing additional room on the roof. Construction was
completed on 01.11.2023 and it was put to use on the same date. The expenditure of
₹45,000 includes cost of wiring and switches of ₹4,500. No depreciation has been
debited with regard to the building.
5. Sale includes sale of ₹1,20,000 to the proprietor and the cost of these goods was
₹1,00,000 and market price ₹1,25,000.
6. Bad debts recovered were allowed earlier.
Mr. Raju has not opted for presumptive taxation of Income u/s 44AD. Compute his Tax
Liability for the Assessment Year 2024-25 under old tax provisions.
Mr. Bablu furnishes the following trading, profit and loss account for the previous year
ending on 31.03.2024.
Mr. Bablu has not opted for presumptive taxation of Income u/s 44AD. You are
required to compute Tax Liability (old provision) after taking the following into
consideration:
1. Purchases include a purchase of ₹20,100. Its payment was made by a bearer cheque
and also includes a purchase from a relative of ₹23,000 and the payment was made in
cash and market price of the purchases is ₹22,000.
2. Factory rent, rates and taxes includes municipal tax of the factory building, which
was paid on 31.07.2024.
3. Assessee has always valued the stocks at cost price but since 2023-24 he has valued
it at market price, which was in excess of the cost price by 10%.
4. Office salaries paid include ₹12,400 to the proprietor of the business.
5. Diwali expenses include gifts of ₹1,000 made to the relatives.
6. The written down value of the block consisting of machinery as on 01.04.2023 is
₹59,000
7. The written down value of the block consisting of factory building as on 01.04.2023
is ₹85,000. An addition was made to building on 01.08.2023 at a cost of ₹12,000.
8. Service charge for air-conditioner were paid in two instalment of ₹10,000 and
₹1,000 on 10.01.2024 and 11.01.2024 in cash.
9. Employer’s contribution was made through an account payee cheque on 10.04.2024
and the cheque realized on 20.04.2024 and the due date for the purpose of provident
fund was 15.04.2024.
10. Computer was purchased on 31.03.2023 and it was put to use on 31.03.2024.
Illustration: 1
Mr. Shyam has invested some amount in ABC Ltd. and the company has paid him interest of
₹5,40,000 after deducting tax at source @ 10%. The cheque was collected by the bank
and the bank charges were 1%. He has income under the head house property ₹10,00,000.
Compute his tax liability and tax refund for assessment year 2024-25 assuming that he
has shifted out of new tax regime u/s. 115BAC.
Solution:
Illustration: 2
Mr. Suresh has invested some amount in ABC Ltd. and the company has paid him interest
of ₹1,80,000 after deducting tax at source @ 10%. The cheque was collected by the bank
and the bank charges were 1%.
Compute his tax liability and tax refund for assessment year 2024-25 assuming he opts
out of new tax regime.
Solution:
Illustration: 3
Mr. Mahesh has borrowed ₹1,00,000 from the market. The amount was invested in
security of some company and the assessee has received a cheque for ₹45,000 (after TDS
@ 10%) being the amount of interest and assessee has paid interest of ₹11,000. He has
casual income ₹2,00,000.
The cheque was given for collection to a bank and the bank has deducted collection
charges of 2%. Mr. Mahesh has income under the head house property ₹2,50,000.
Compute his tax liability/ tax payable for assessment year 2024-25 under old tax
provisions.
Solution:
Illustration: 4
Mrs. Nitisha is getting a family pension of ₹7,000 p.m. She has also received interest on
fixed deposit of ₹90,000 after deducting tax at source of ₹10,000. The bank has
deducted collection charges @ 1.5%. She has short term capital gains under section 111A
of ₹4,00,000. Deductions under section 80C to 80U is ₹67,650.
Compute her tax liability for assessment year 2024-25 under normal tax regime.
Solution:
Examine the implications of tax deduction at source under section 194A in the cases
mentioned hereunder, based on the provisions of the Income-tax Act, 1961.
(i) On 01.10.2023, Mr. Mohan made a six-month fixed deposit of ₹12 lakh @ 8% p.a. with
Theta Co-operative Bank. The fixed deposit matures on 31.3.2024.
(ii) Mr. Harish made fixed deposits carrying interest @10% p.a. with the following
branches of Omega Bank, a bank which has adopted CBS.
Branch Amount (₹) Date of deposit Date of Maturity
Adyar 60,000 01.06.2023 31.03.2024
Anna Nagar 80,000 01.07.2023 31.03.2024
Nungambakkam 75,000 01.08.2023 31.03.2024
(iii) On 01.04.2023, Ms. Meera started a 1-year recurring deposit of ₹50,000 per month @
10% p.a. with Gamma Bank. The recurring deposit matures on 31.3.2024. Gamma bank pays
interest of ₹43,000.
Solution:
(i) Theta Co-operative Bank has to deduct tax at source @ 10% on the interest of ₹48,000
(8% × ₹12 lakh × ½) under section 194A.
(ii) Since Omega Bank has adopted CBS, the aggregate interest credited/paid by all
branches has to be considered, and if the same exceeds ₹40,000, tax is deductible under
section 194A. Omega Bank is not required to deduct tax at source @10% under section
194A, since the aggregate interest on fixed deposit with the three branches of the bank
is ₹16,000, which does not exceed the threshold limit of ₹40,000.
(iii) Tax has to be deducted @ 10% under section 194A by Gamma Bank on the interest of
₹43,000 on recurring deposit on 31.3.2024 to Ms. Meera, since –
(1) recurring deposit has been included in the definition of “time deposit”; and
(2) such interest exceeds the threshold limit of ₹40,000.
Examine the TDS implications under section 194A in the cases mentioned hereunder –
(i) On 01.10.2023, Mr. Harish made a six-month fixed deposit of ₹10 lakh @ 9% p.a. with
ABC Co-operative Bank. The fixed deposit matures on 31.3.2024.
(ii) On 01.06.2023, Mr. Ganesh made three nine-month fixed deposits of ₹2 lakh each
carrying interest @ 9% per annum with Dwarka Branch, Janakpuri Branch and Rohini
Branches of XYZ Bank, a bank which has adopted CBS. The fixed deposits mature on
28.02.2024.
(iii) On 01.04.2023, Mr. Rajesh started a 1-year recurring deposit of ₹60,000 per month @
8% p.a. with PQR Bank. The recurring deposit matures on 31.03.2024. PQR bank pays
interest of ₹50,400.
Solution:
(i) ABC Co-operative Bank has to deduct tax at source @10% on the interest of ₹45,000
(9% × ₹10 lakh × ½) under section 194A. The tax deductible at source under section 194A
from such interest is, therefore, ₹4,500.
(ii) XYZ Bank has to deduct tax at source @10% under section 194A, since the aggregate
interest on fixed deposit with the three branches of the bank is ₹40,500 [₹2,00,000 × 3 ×
9% × 9/12], which exceeds the threshold limit of ₹40,000. Since XYZ Bank has adopted
CBS, the aggregate interest credited/paid by all branches has to be considered. Since the
aggregate interest of ₹40,500 exceeds the threshold limit of ₹40,000, tax has to be
deducted @ 10% under section 194A.
(iii) Tax has to be deducted under section 194A @ 10% by PQR Bank on the interest of
₹50,400 on recurring deposit on 31.03.2024 to Mr. Rajesh, since –
(1) “recurring deposit” has been included in the definition of “time deposit”; and
(2) such interest exceeds the threshold limit of ₹40,000.
Illustration: 8
Mr. X purchased 20 lottery tickets of ₹250 each with a winning of ₹2.80 lakhs (after TDS
@ 30%). He has also received interest of ₹72,000 after deducting tax at source @ 10% in
connection with fixed deposit. and the cheque was collected by bank and service charges
@ 2% was taken by the bank. He has income from subletting of house property ₹9,000 p.m.
He has received family pension of ₹4,000 p.m. Compute his Income under the head Other
Sources for assessment year 2024-25.
Solution:
Illustration: 9
Moon TV, a television channel, made payment of ₹50 lakhs to a production house for
production of programme for telecasting as per the specifications given by the channel.
The copyright of the programme is also transferred to Moon TV. Would such payment be
liable for tax deduction at source under section 194C? Discuss.
Also, examine whether the provisions of tax deduction at source under section 194C would
be attracted if the payment was made by Moon TV for acquisition of telecasting rights of
the content already produced by the production house.
Solution:
In this case, since the programme is produced by the production house as per the
specifications given by Moon TV, a television channel, and the copyright is also transferred
to the television channel, the same falls within the scope of definition of the term ‘work’
under section 194C. Therefore, the payment of ₹50 lakhs made by Moon TV to the
production house would be subject to tax deduction at source under section 194C.
If, however, the payment was made by Moon TV for acquisition of telecasting rights of the
content already produced by the production house, there is no contract for “carrying out
any work”, as required in section 194C. Therefore, such payment would not be liable for
tax deduction at source under section 194C.
Illustration: 10
Examine the applicability of the provisions for tax deduction at source under section
194DA in the following cases–
(i) Mr. X, a resident, is due to receive ₹4.50 lakhs on 31.03.2024, towards maturity
proceeds of LIC policy taken on 01.4.2020, for which the sum assured is ₹4 lakhs and the
annual premium is ₹1,25,000.
(ii) Mr. Y, a resident, is due to receive ₹3.25 lakhs on 31.03.2024 on LIC policy taken on
31.03.2012, for which the sum assured is ₹3 lakhs and the annual premium is ₹35,000.
(iii) Mr. Z, a resident, is due to receive ₹95,000 on 01.08.2023 towards maturity proceeds
of LIC policy taken on 01.08.2014 for which the sum assured is ₹90,000 and the annual
premium was ₹12,000.
Solution:
(i) Since the annual premium exceeds 10% of sum assured in respect of a policy taken on
01.04.2020, the maturity proceeds of ₹ 4.50 lakhs are not exempt under section 10(10D)
in the hands of Mr. X. Therefore, tax is required to be deducted @ 5% under section
194DA on the amount of income of ₹75,000 (₹4,50,000- ₹3,75,000).
(ii) Since the annual premium is less than 20% of sum assured in respect of a policy taken
before 01.04.2012, the sum of ₹3.25 lakhs due to Mr. Y would be exempt under section
10(10D) in his hands. Hence, no tax is required to be deducted at source under section
194DA on such sum payable to Mr. Y.
(iii) Even though the annual premium exceeds 10% of sum assured in respect of a policy
taken after 31.03.2012, and consequently, the maturity proceeds of ₹95,000 due on
01.08.2023 would not be exempt under section 10(10D) in the hands of Mr. Z, the tax
deduction provisions under section 194DA are not attracted since the maturity proceeds
are less than ₹1 lakh.
Illustration: 12
Examine whether TDS provisions would be attracted in the following cases, and if so,
under which section. Also specify rate of TDS applicable in each case. Assume that all
payments are made to residents.
Solution:
State in brief the applicability of tax deduction at source provisions, the rate and amount
of tax deduction in the following cases for the financial year 2023-2024 under the Income
tax Act, 1961. Assume that all payments are made to residents:
(i) Sanjay, a resident individual, not deriving any income from business or profession makes
payment of ₹12 lakhs in January, 2024, ₹20 lakh in February, 2024 and ₹20 lakh in March,
2024 to Mohan, a contractor for reconstruction of his residential house.
(ii) ABC Ltd. makes the payment of ₹1,50,000 to Ramlal, an individual transporter who
owned 6 goods carriages throughout the previous year, He does not furnish his PAN.
(iii) Smt. Sarita paid ₹5,000 on 17th April, 2023 to Smt. Deepa from the deposits in
National savings Scheme account.
Solution:
(i) Yes, under section 194M since the aggregate of payments (i.e., ₹52 lakhs) exceeds ₹50
lakhs and his turnover is below ₹100 lakhs in the P.Y. 2022-23. Hence, TDS provisions
under section 194C are not attracted in respect of payments made in the P.Y. 2023-24 and
section 194M gets attracted as the aggregate payments exceeds ₹50 lakhs, hence he is
liable to deduct TDS @ 5% on 52,00,000 = ₹2,60,000.
(ii) As per section 194C, no tax shall be deducted at source in case of payment to a
contractor in connection with transportation of goods where such contractor do not own
more than 10 goods carriages at any time during the year and also submitted a declaration
in this regard and has also furnished permanent account number. But in the given case
transporter has not furnished his PAN hence ABC limited can deduct TDS u/s. 194C.
As transporter has not furnished his PAN then section 206AA shall also be applicable and
TDS shall be deducted @ 20% on 1,50,000 = 30,000.
(iii) As per section 194EE, the person responsible for paying to any person any amount
from deposits under National saving scheme shall, at the time of payment thereof, deduct
income-tax thereon at the rate of 10% provided amount is exceeding 2,500 in a financial
year. In the given case amount exceeds 2,500 hence TDS shall be deducted @ 10% on
5,000 = 500.
Mr. Ajay Sahu, proprietor of M/s Blue Bird Enterprises having turnover of ₹ 65 lakhs and
Turnover is less than 100 lakhs during P.Y. 2022-23, has received two bills for payment.
The first bill is for ₹42,00,000 from Vijay Associates, an advocate and property dealer
firm, for his daughter’s hearing and ₹21,00,000 from same Vijay Associates for brokerage
service provided in relation to purchase of one property. Both bills were raised on
21.12.2023 but payment were made in instalments. 1st Instalment of ₹5,00,000 as advance
was payment on 15.11.2023, 2nd Instalment of ₹45,00,000 on 25.03.2024 and balance
amount ₹13,00,000 on 31.03.2024.
Determine the TDS liability for Mr. Ajay Sahu, if any, for A.Y. 2024-25?
(a) ₹ 2,50,000
(b) ₹ 3,15,000
(c) ₹ 65,000
(d) Nil
Solution: (b)
As per section 194M, any individual and HUF (not liable to deduct tax at source u/s
194C/194J/194H) is liable to deduct tax at source @ 5% if aggregate of such payment is
exceeding 50,00,000. Payment is for commission or brokerage or by way of fees for
professional services.
In the given case, it is covered u/s 194M, hence TDS shall be deducted @ 5% on
63,00,000 = 3,15,000
Mr. Nihar maintains a Savings A/c and a Current A/c in Mera Bank Ltd. The details of
withdrawals on various dates during the previous year 2023-24 are as follows:
Is Mera Bank Limited required to deduct tax at source on the withdrawals made by Mr.
Nihar during the previous year 2023-24? If yes, what would the amount of tax deducted
at source? If Mr. Nihar has filed ITRs and Not filed ITRs for last 3 years.
Solution:
Total withdrawal upto 31.03.2024 are ₹160 lakhs hence TDS shall be at a rate of 2% on
the entire amount of ₹60 lakhs. Amount of TDS shall be ₹60 lakhs X 2% = ₹1,20,000.
If return has not been filed upto 3 years, TDS shall be on ₹80 lakhs at a rate of 2% =
₹1,60,000 and on ₹60 lakhs at a rate of 5% = ₹3 lakhs.
Illustration: 16
An amount of ₹40,000 was paid to Mr. X on 01.07.2023 towards fees for professional
services without deduction of tax at source. Subsequently, another payment of ₹50,000
was due to Mr. X on 28.02.2024, from which tax @ 10% (amounting to ₹9,000) on the
entire amount of ₹ 90,000 was deducted. However, this tax of ₹ 9,000 was deposited only
on 22.06.2024.
Compute the interest chargeable under section 201.
Solution:
Illustration: 17
Mrs. X has received incomes as given below during the previous year 2023-24:
1. Interest on savings bank account with State Bank ₹50,000 (gross).
2. Interest from Government securities ₹1,00,000 on 01.01.2024 (collection charge paid to
the bank @ 1.5%).
3. Interest from ABC Ltd on non-listed debentures ₹3,60,000 (after TDS) on 01.03.2024
(collection charge paid to the bank ₹30).
4. Interest credited to post office savings bank account during the year ₹10,000.
5. Interest credited to public provident fund during the year ₹15,000.
6. Interest received from XYZ Ltd on listed debentures ₹1,35,000 (Net). (Collection
charge ₹30) The amount was invested by taking a loan of ₹15,00,000 @ 12% p.a.
7. Mrs. X received rent of house property ₹ 72,000 per month after TDS.
8. Winnings from a lottery ₹70,000 (after TDS)
Compute her tax liability and also tax payable for the assessment year 2024-25 under old
tax provision.
Solution:
Question 1: Whether TCS @ 1% is on sale of motor vehicle at retail level or also on sale
of motor vehicles by manufacturers to dealers/ distributors?
Solution:
To bring high value transactions within the tax net, section 206C has been amended to
provide that the seller shall collect the tax @ 1% from the purchaser on sale of motor
vehicle of the value exceeding ₹ 10 lakhs. This is brought to cover all transactions of retail
sales and accordingly, it will not apply on sale of motor vehicles by manufacturers to
dealers/distributors.
Question 2: Whether TCS @ 1% on sale of motor vehicle is applicable only to luxury cars?
Solution:
No, as per section 206C(1F), the seller shall collect tax @1% from the purchaser on sale of
any motor vehicle of the value exceeding ₹ 10 lakhs.
Solution: An individual who is liable to audit as per the provisions of section 44AB during
the financial year immediately preceding the financial year in which the motor vehicle is
sold shall be liable for collection of tax at source on sale of motor vehicle by him.
Question 6: How would the provisions of TCS on sale of motor vehicle be applicable in a
case where part of the payment is made in cash and part is made by cheque?
Solution:
The provisions of TCS on sale of motor vehicle exceeding ₹ 10 lakhs is not dependent on
mode of payment. Any sale of motor vehicle exceeding ₹ 10 lakhs would attract TCS @ 1%.
HW Question: 1
What are the clarifications made by CBDT with respect to Section 206C(1F) relating to
following issues:
(i) Whether TCS on sale of motor vehicle is applicable only to luxury car?
(ii) Whether TCS is applicable on each sale or aggregate value of sale motor vehicle,
exceeding ₹ 10 lakhs?
(iii) Whether TCS is applicable in case of an individual?
(iv) Whether TCS on sale of motor vehicle is at retail level or only by manufacturer to
distributor or dealer?
HW Question: 2
Rahil & Co., a partnership firm is having a car dealership show-room. They have purchased
cars for ₹2 crores from XYZ Ltd., car manufacturers, the cost of each car being more
than ₹12 lakhs. They sell the cars to individual buyers at a price yielding 10% margin on
cost. State whether there will be any obligation to collect tax in the above two situations.
Illustration: 18
Mr. Gupta, a resident Indian, is in retail business and his turnover for F.Y. 2022-23 was
₹12 crores. He regularly purchases goods from another resident, Mr. Agarwal, a
wholesaler, and the aggregate payments during the F.Y. 2023-24 was ₹95 lakh (₹20 lakh on
01.06.2023, ₹ 25 lakh on 12.8.2023, ₹ 22 lakh on 23.11.2023 and ₹ 28 lakh on 25.3.2024).
Assume that the said amounts were credited to Mr. Agarwal’s account in the books of Mr.
Gupta on the same date. Mr. Agarwal’s turnover for F.Y. 2022-23 was ₹ 15 crores.
(1) Based on the above facts, examine the TDS/TCS implications, if any, under the
Income tax Act, 1961.
(2) Would your answer be different if Mr. Gupta’s turnover for F.Y. 2022-23 was ₹8
crores, all other facts remaining the same?
(3) Would your answer to (1) and (2) change, if PAN has not been furnished by the buyer
or seller, as required?
Solution:
1) Since Mr. Gupta’s turnover for F.Y. 2022-23 exceeds 10 crores, and payments made by
him to Mr. Agarwal, a resident seller exceed ₹ 50 lakhs in the P.Y. 2023-24, he is
liable to deduct tax @ 0.1% of ₹ 45 lakhs (being the sum exceeding ₹ 50 lakhs) in the
following manner-
No tax is to be deducted u/s 194Q on the payments made on 1.6.2023 and 12.8.2023,
since the aggregate payments till that date i.e. 45 lakhs, has not exceeded the
threshold of ₹ 50 lakhs.
Tax of ₹ 1,700 (i.e., 0.1% of ₹ 17 lakhs) has to be deducted u/s 194Q from the
payment/ credit of ₹22 lakh on 23.11.2023 [₹ 22 lakh - ₹ 5 lakhs, being the balance
unexhausted threshold limit].
Tax of ₹ 2,800 (i.e., 0.1% of 28 lakhs) has to be deducted u/s 194Q from the
payment/credit of ₹ 28 lakh on 25.3.2024.
Note- In this case, since both section 194 Q and 206C(1H) applies, tax has to be
deducted u/s 194Q.
2) If Mr. Gupta’s turnover for the F.Y. 2022-23 was only ₹8 crores, TDS provisions
under section 194Q would not be attracted. However, TCS provisions under section
206C (1H) would be attracted in the hands of Mr. Agarwal, since his turnover exceeds
₹ 10 Crores in the F.Y. 2022-23 and his receipts from Mr. Gupta exceed ₹ 50 lakhs.
No tax is to be collected u/s 206C(1H) on 1.6.2023 and 12.8.2023 since the aggregate
receipts till that date i.e. 45 lakhs, has not exceeded the threshold of ₹ 50 lakhs.
Tax of ₹ 1,700 (i.e., 0.1% of 17 lakhs ) has to be collected u/s 206C(1H) on 23.11.2023
(₹22 lakh - ₹5 lakhs, being the balance unexhausted threshold limit).
Tax of ₹ 2,800 (i.e., 0.1% of ₹ 28 lakhs) has to be collected u/s 206C(1H) on
25.3.2024.
3) In case (1), if PAN is not furnished by Mr. Agarwal to Mr. Gupta, then, Mr. Gupta has
to deduct tax @5%, instead of 0.1%. Accordingly, tax of ₹ 85,000 (i.e., 5% of ₹ 17
lakhs) and ₹ 1,40,000 (5% of 28 lakhs) has to be deducted by Mr. Gupta u/s 194Q on
23.11.2023 and 25.3.2024, respectively.
In case (2), if PAN is not furnished by Mr. Gupta to Mr. Agarwal, then, Mr. Agarwal
has to collect tax @ 1% instead of 0.1%. Accordingly, tax of ₹ 17,000 (i.e., 1% of ₹ 17
lakhs) and ₹ 28,000 (1% of ₹ 28 lakhs) has to be collected by Mr. Agarwal u/s
206C(1H) on 23.11.2023 and 25.3.2024, respectively.
Illustration: 19
ABC Ltd. has estimated its tax liability for assessment year 2024-25 as ₹4,40,000 and
has paid advance tax accordingly but actual tax liability was found to be ₹10,00,000.
The company has paid balance amount on 02.12.2024 and filed return of income on the
same date.
Compute interest payable under section 234A, 234B, and 234C.
Solution:
Interest under section 234C shall be computed in the manner given below:
Due date Tax Payable Tax Paid Default Rounded off Interest u/s. 234C
15.06.2023 1,50,000 66,000 84,000 84,000 2520
[84000 x 1% x 3]
15.09.2023 4,50,000 1,98,000 2,52,000 2,52,000 7,560
[2,52,000 x 1% x 3]
15.12.2023 7,50,000 3,30,000 4,20,000 4,20,000 12,600
[4,20,000 x 1% x 3]
15.03.2024 10,00,000 4,40,000 5,60,000 5,60,000 5,600
[5,60,000 x 1% x 1]
Total interest payable u/s 234C 28,280
Interest under section 234B shall be computed from 01.04.2023 to 02.12.2024 and is as
given below:
10,00,000 – 4,40,000 = 5,60,000 x 1% x 9 = 50,400
Interest under section 234A shall be computed from 01.11.2023 to 02.12.2024 and is as
given below:
5,60,000 x 1% x 2 = 11,200
Illustration: 20
ABC Ltd. has tax liability of ₹7,00,000 for the previous year 2023-24 and the company
has not paid any advance tax and entire tax amount was paid by the company on
31.12.2024. In this case, interest shall be calculated in the manner given below:
Illustration: 21
ABC Ltd. has estimated its tax payable to be ₹5,00,000 for previous year 2023-24 and
has paid advance tax accordingly but actual tax liability of the company was found to be
₹5,50,000 and difference of tax amount was paid on 10.12.2024. Compute interest under
section 234A, 234B and 234C.
Solution:
Interest under section 234C shall be computed in the manner given below:
Due date Tax Payable Tax Paid Default Rounded off Interest u/s. 234C
15.06.2023 82,500 75,000 7,500 7,500 Nil
Interest u/s 234C = Nil (because advance tax paid is at least 12%)
[50,000 x 1% x 1]
Total interest payable u/s 234C 1,625
Interest under section 234A shall be computed from 01.11.2024 to 10.12.2024 and is as
given below:
50,000 x 1% x 2 = 1,000
Illustration: 22
He had long term capital gains of ₹3,00,000 on 01.01.2024 and his income under the head
business/Profession is ₹11,00,000
He has filed return of income on 10.12.2024 and has paid difference of the tax on
10.12.2024.
Last date for filing of return is 31.07.2024.
Compute interest payable under section 234A, 234B and 234C.
Solution:
Note: Installment for 15th March shall be including tax on capital gains
Illustration: 23
A partnership firm made the following payments of advance tax during the financial year
2023-24:
Upto June 15, 2023 4,15,000
Upto September 15, 2023 8,25,000
Upto December 15, 2023 16,64,000
Upto March 15, 2024 26,23,000
Return of income filed by the firm is ₹88,00,000 under the head “profits and gains of
business or profession” and ₹9,50,000 by way of long term capital gains on sale of a
property effected on December 1, 2023. What is the interest payable by the assessee
under section 234B and section 234C for assessment year 2024-25?
Assume that the return of income was filed on 31.10.2024 i.e. the due date and tax was
fully paid on self assessment.
Solution:
Note: Installments for 15th December and 15th March shall be including tax on capital
gains
HW Question: 3
ABC Ltd. has paid advance tax for the previous year 2023-24 as given below:
1. Upto 15.06.2023 ₹50,000
2. Upto 15.09.2023 ₹1,50,000
3. Upto 15.12.2023 ₹3,00,000
4. Upto 15.03.2024 ₹6,00,000
Actual tax liability was found to be ₹7,00,000 and balance tax was paid on 10.12.2024.
Compute interest payable under section 234A, 234B, 234C.
Comprehensive Questions
Examine whether TDS provisions would be attracted in the following cases, and if so,
under which section. Also specify the rate of TDS and amount required to be deducted at
source as applicable in each case. Assume that all payments are made to residents.
(i) On 1.5.2023, Mr. Brijesh made three fixed deposits of nine months each of ₹3 lakh
each, carrying interest @ 9% p.a. with Mumbai Branch, Delhi Branch and Chandigarh
Branch of CBZ Bank, a bank which had adopted CBS. These Fixed Deposits mature on
31.01.2024.
(ii) Mr. Marwah, aged 80 years, holds 6½% Gold Bonds, 1977 of ₹2,00,000 and 7% Gold
Bonds 1980 of ₹3,00,000. He received yearly interest on these bonds on 28.02.2024.
(iii) M/s AG Pvt. Ltd. took a loan of ₹ 50,00,000 from Mr. Haridas. It credited interest of
₹79,000 payable to Mr. Haridas during the previous year 2023-24. M/s AG Pvt. Ltd. is not
liable for tax audit during previous years 2022-23 and 2023-24.
(iv) Mr. Prabhakar is due to receive ₹6 lakh on 31.3.2024 towards maturity proceeds of
LIC policy taken on 01.04.2019, for which the sum assured is ₹5 lakhs and the annual
premium is ₹1,40,000.
Examine & explain the TDS implications in the following cases along with reasons thereof,
assuming that the deductees are residents and having a PAN which they have duly
furnished to the respective deductors.
(i) Mr. Tandon received a sum of ₹1,75,000 as pre-mature withdrawal from Employees
Provident Fund Scheme before continuous service of 5 years on account of termination of
employment due to ill-health.
(ii) A sum of ₹42,000 has been credited as interest on recurring deposit by a banking
company to the account of Mr. Hasan (aged 63 years).
(iii) Ms. Kaul won a lucky draw prize of ₹21,000. The lucky draw was organized by M/s.
Maximus Retail Ltd. for its customer.
(iv) Finance Bank Ltd. sanctioned and disbursed a loan of ₹10 crores to Borrower Ltd. on
31.03.2024. Borrower Ltd. paid a sum of ₹1,00,000 as service fee to Finance Bank Ltd. for
processing the loan application.
(v) Mr. Ashok, working in a private company, is on deputation for 3 months (from
December, 2023 to February, 2024) at Hyderabad where he pays a monthly house rent of
₹52,000 for those three months, totalling to ₹1,56,000. Rent is paid by him on the first
day of the relevant month.
Examine the TDS implications in the following cases along-with reasons thereof;
(i) Ms. Varsha received a sum of ₹ 95,000 on 31st December 2023 towards maturity
proceeds of LIC taken on 1st October 2015 for which sum assured was ₹ 80,000 and
annual premium was ₹ 10,000.
(ii) Mr. Deepak transferred a residential house property to Mr. Karan for ₹ 45 lacs. The
stamp duty value of such property is ₹ 55 lacs.
(iii) XYZ Private Limited pays the following amounts to Mr. Narayan during previous year
2023-24:
(1) ₹22,000 towards fee for professional services
(2) ₹18,000 towards royalty
(iv) Payment of ₹ 1,75,000 made to Mr. Vaibhav for purchase of calendar according to
specification of M/s. ABC Limited. However, no material was supplied for such calendar by
ABC Limited to Mr. Vaibhav.
(v) Talent Private Limited pays ₹ 12,000 to Ms. Sudha, its director, towards sitting fee
which is not taxable u/s 192.
(vi) Radha Limited is engaged for Shyam Limited only in the business of operation of call
centre. On 18.03.2024, the total amount credited by Shyam Limited in the ledger account
of Radha Limited is ₹70,000 regarding service charges of call centre. The amount is paid
through cheque on 28.03.2024 by Shyam Limited.
The following issues arise in connection with the deduction of tax at sources under
chapter XVII-B. Discuss the liability for tax deduction in these cases:
(i) An employee of the Central Government receives arrears of salary for the earlier 3
years. He inquires whether amount will be received after deduction of tax at source during
the current year.
(ii) A T.V. channel pays ₹ 10 lacs as prize money to the winner of a quiz programme.
(iii) A Nationalized bank pays ₹ 50,000 per month as rent to ABC limited for a building in
which one of its branch is situated.
(iv) A television company pays ₹ 50,000 to a cameraman for shooting of a documentary
film.
Mr. Dhanapal wishes to purchase a residential house costing ₹60 lakhs from Ms. Saipriya.
The house is situated at Chennai. He also wants to purchase agricultural lands in a rural
area for ₹65 lakhs. He wants to know whether there will be any obligation to deduct tax at
source in these two situations. Both the buyer as well as the sellers are residents in India.
Advise Mr. Dhanapal suitably.
Examine the applicability of the provisions for tax deduction at source under Income Tax
Act, 1961.
(i) Mr. Z, a resident, is due to receive, ₹95,000 on 01.10.2023 towards maturity proceeds
of LIC policy taken on 01.10.2012 for which sum assured was ₹90,000 and the annual
premium was ₹15,000.
(ii) Mudra Ltd., an advertising Company, has retained a sum of ₹15 Lakhs, towards charges
for procuring and canvassing advertisement, from payment of 1 crore due to Cloud TV, A
television channel and remitting the balance amount of ₹85 Lakhs to the television channel.
Would the provisions of Tax deduction at source under section 194H be attracted on the
sum of ₹15 Lakhs retained by the advertising company?
(iii) Mr. X is salaried Individual pays rent of 55,000 per month to Mr. Y (does not have
PAN) from June 2023. ls he required to deduct TDS? If so, when is he required to deduct
tax? Mr. X vacated the premises on 31st December, 2023.
The following details are provided by Mr. Pinto, an individual, for the assessment year
2024-2025.
Amount (₹)
Total estimated tax payable 2,00,000
TDS (estimated but not deducted) 55,000
Determine the advance tax payable with their due dates for the assessment year 2024-
25.
Question: 9 [TDS]
Discuss the applicability of provisions of Tax Deduction at source, the rate and amount of
tax deduction to be made in the following cases for the financial year 2023-24.
(A) Mr. Bobby, a resident whose turnover during the previous financial year is ₹205 Lakhs
and for the current year 2023-24 it is ₹80 lakhs.
(i) Shop rent paid to Mr. Rajasekharan, a resident ₹25,000 per month.
(ii) On 1-11-2023 paid towards fee for technical services ₹25,000 and royalty of ₹20,000
to Mr. Swamy, a resident who is having PAN. No other payment made to Mr. Swamy.
(iii) On 01-10-2023 payment of ₹2,00,000 made to Mr. A for purchase of diaries according
to specifications. However, no material was supplied for such diaries.
(iv) Contract payments made to Mr. Satheesan on 01-05-2023 for painting ₹25,000 and
another contract for interior furnishing on 22-03-2024 for ₹20,000.
(B) Mr. Thrilok an individual not assessed to tax pays towards rent ₹60,000 per month.
Pallavi Bank Ltd., has paid interest of ₹29,000 to Mr. A, a resident Indian, from its
Chennai branch and ₹28,000 from Bangalore branch. If there is no core banking services in
the bank, is tax required to be deducted at source from such interest payments made on
31.03.2024? Will your answer be different if there is core banking service present in the
bank? Also, explain the provisions of the Income-tax Act, 1961 in this regard.
Mr. Sachal, a resident individual aged 54, furnishes income details as under:
(i) Wholesale Cloth business, whose turnover is ₹150 lakhs, for which accounts are audited
u/s 44AB.
Income from such business is ₹8,10,000.
(ii) Income from other sources is ₹2,70,000.
(iii) Tax deducted at source is ₹25,000.
(iv) Advance tax paid of ₹1,03,000 on 14-3-2024.
Return of income will be filed on 11-12-2024. The assesse is willing to pay the requisite
self-assessment tax.
Calculate the interest payable under section 234C and 234B of the income-tax Act, 1961.
Assume that the return of income would be processed on the same day of filing of return.
Question: 12 [TDS]
Ashwin a resident Individual carrying on business, furnishes you the following information:
Particulars ₹
Commission paid to Babloo 18,500
Payment to Vijay for repair of office building 23,000
Payment of fees for Technical Services, to Vivek 35,000
Question: 13 [TDS]
Mr. Madan sold his house property in Surat as well as his rural agricultural land for a
consideration of ₹65 lakhs and ₹20 lakhs respectively, to Mr. Raman on 01.10.2023. He has
purchased the house property for ₹40 lakhs and the land for ₹15 lakhs, in the year 2015.
There was no difference in the stamp valuation. You are required to determine TDS
implications, if any, assuming both persons are resident Indians.
State in brief the applicability of tax deduction at source provisions, the rate and amount
of tax deduction in the following cases for the financial year 2023-24
(1) Payment of ₹27,000 made to a South African cricketer, by an Indian newspaper agency
on 02.07.2023 for contribution of articles in relation to the sport of cricket.
(2) Rent of ₹1,70,000 paid by a partnership firm for use of plant and machinery.
(3) Winning from horse race ₹1,50,000.
(4) ₹2,00,000 paid to Mr. X, a resident individual on 22.02.2024 by the State of Uttar
Pradesh on compulsory acquisition of his urban land.
What are the provisions relating to tax deduction at source in respect of:
(A) ABC and Co. Ltd. Paid ₹19,000 to one of its Directors as sitting fees on 01.01.2024.
(B) Mr. X sold his House to Mr. Y on 01.02.2024 for ₹60 lacs?
(i) Mr. X doing textiles business furnishes you the following information:
State whether the provisions of tax deduction at source are attracted for the following
expenses incurred during the financial year 2023-24;
State the applicability of TDS provisions and TDS amount in the following cases:
(a) Rent paid for hire of machinery by ABC Ltd. to Mr. X ₹3,10,000.
(b) Fee paid to Dr. Y by X (HUF) ₹35,000 for surgery performed to a member of the
family.
Compute amount of tax to be deducted at source on the following payments made by M/s
ABC Ltd. during the financial year 2023-24 as per the provisions of the Income-Tax Act,
1961.
Sr. No. Date Nature of Payment
(i) 01.10.2023 Payment of ₹2,00,000 to Mr. “X” a transporter who is
having PAN and who do not have more than 10 goods
carriages.
(ii) 01.11.2023 Payment of fee for technical services of ₹45,000 to
Mr. X who is having PAN.
(iii) 30.12.2023 Payment of ₹25,000 to M/s X Ltd. for repair of
building.
(iv) 01.01.2024 Payment of ₹2,00,000 made to Mr. Y for purchase of
diaries made according to specifications of M/s ABC
Ltd. However, no material was supplied for such diaries
to Mr. Y by M/s ABC Ltd.
(v) 01.01.2024 Payment made ₹2,20,000 to Mr. Z for compulsory
acquisition of his house as per Law of the State
Government.
(vi) 01.02.2024 Payment of commission of 25,000 to Mr. A.
Mr. X doing manufacture and wholesale trade furnishes you the following information:
Total turnover for the financial year ₹
2022-23 215,00,000
2023-24 105,00,000
State whether tax deduction at source provisions are attracted for the below said
expenses incurred during the financial year 2023-24: ₹
Interest paid to UCO Bank 41,000
Contract payment to Mr. Y (2 contracts of ₹12,000 each) 24,000
Shop rent paid (one payee) 2,90,000
Commission paid to Mr. Z 18,000
State with reasons, whether tax deduction at source provisions are applicable to the
following transactions and if so, the rate of tax deduction:
(i) X & Co. (Firm) engaged in wholesale business assigned a contract for construction of its
godown building to Mr. X, a contractor. It paid ₹25,00,000 to Mr. X as contract payment.
(ii) Y & Co. engaged in real estate business conducted a lucky dip and gave Maruti car to a
prize winner.
(iii) An Insurance Company paid ₹45,000 as Insurance Commission to its agent Mr. Y.
(iv) AB Ltd. allowed a discount of ₹50,000 to XY & Co. (a firm) on prompt (immediate)
payment towards supply of automobile parts.
From the following particulars of Mr. X for the previous year ended 31st March, 2024
compute total income and tax liability for assessment year 2024-25.
₹
(i) Directors Fee from a company 3,00,000
(ii) Interest on saving bank deposits 23,000
(iii) Winnings from Lotteries (Net) 35,000
(iv) Royalty on a book written by him 2,09,000
(v) Lectures in Seminars 5,000
(vi) Interest on loan given to a relative 7,000
(vii) Interest on debentures of a company (listed in a Recognised Stock Exchange) net of
TDS 36,000
(viii) Interest on Post Office Savings Bank Account 500
(ix) Interest on Government Securities 2,200
(x) Interest on Monthly Income Scheme of Post office 33,000
He paid ₹10,000 for typing the manuscript of book written by him.
Illustration: 1
Solution: ₹
Mrs. Rohini, aged 62 years, was born and brought up in New Delhi. She got married in
Russia in 1996 and settled there since then. Since her marriage, she visits India for 60
days each year during her summer break. The following are the details of her income for
the previous year ended 31.03.2024:
You are required to ascertain the residential status of Mrs. Rohini and compute her total
income and tax liability in India for Assessment year 2024-25.
Solution:
An Indian citizen or a person of Indian origin who, being outside India, comes on a visit to
India (and whose total income, other than from foreign sources, does not exceed
₹15,00,000) would be resident in India only if he or she stays in India for a period of 182
days or more during the previous year.
Since Mrs. Rohini is a person of Indian origin who comes on a visit to India only for 60 days
in the P.Y. 2023-24 and her income other than from foreign sources does not exceed
₹15,00,000, she is non-resident for the A.Y. 2024-25.
in India. Accordingly, her total income and tax liability would be determined in the
following manner:
Computation of total income and tax liability of Mrs. Rohini for A.Y. 2024-25
Capital Gains
Long-term capital gains on sale of land at New Delhi [Taxable, since it is 3,00,000
deemed to accrue or arise in India as it is arising from transfer of land
situated in India]
Short-term capital gains on sale of shares of Indian listed companies in
respect of which STT was paid [Taxable, since it is deemed to accrue or arise 60,000
in India, as such income arises on transfer of shares of Indian listed
companies]
Gross Total Income 4,23,000
Less: Deduction under Chapter VI-A
Deduction under section 80C (63,000)
- Life insurance premium of ₹ 75,000 [Premium paid to Russian Life Insurance
Corporation allowable as deduction. However, the same has to be restricted to
gross total income excluding LTCG and STCG, as Chapter VI-A deductions are
not allowable against such income chargeable to tax u/s 112 and 111A,
respectively] 3,60,000
Total Income
Note –
(i) Even if her total income exceeds ₹ 15 lakh, still, she would be non-resident since the
minimum period of stay required in the current year for being a resident is 120 days.
(ii) The benefit of adjustment of unexhausted basic exemption limit against long-term
capital gains taxable u/s 112 and short-term capital gains taxable u/s 111A is not available
in case of non-resident.
Further, rebate u/s 87A is not allowable to a non-resident, even if his income does not
exceed ₹ 5 lakh.
(iii) It is assumed that such premium is paid for self or spouse or any child of Mrs. Rohini.
HW Question: 1
Compute the eligible deduction under section 80C for A.Y. 2024-25 in respect of life
insurance premium paid by Mr. Ajeet during the P.Y. 2023-24, the details of which are
given hereunder –
Compute the eligible deduction under section 80C for A.Y. 2024-25 in respect of life
insurance premium paid by Mr. Mukesh during the P.Y. 2023-24, the details of which are
given hereunder –
Illustration: 3
Mr. Vijay, aged about 61 years, has earned a lottery income of ₹1,20,000 (gross) during
the P.Y. 2023-24. He also has a business income of ₹6,00,000. He invested an amount of
₹10,000 in Public Provident Fund account and ₹24,000 in National Saving Certificates and
₹60,000 in eligible mutual funds. He has paid premium of ₹30,000 for a life policy having
sum assured ₹2,00,000 and policy was taken after 01.04.2012. Compute his tax liability for
assessment year 2024-25.
Solution:
The particulars of income of Mrs. Nikita, aged 55 years for the financial year 2023-24 are
given below:
(1) Income under the head salary received from M/s ABC Ltd. for the year is 4,00,000
(2) Rental income received from a commercial complex is 12,000 p.m.
(3) Arrears of rent received from the complex, which were not charged to tax in any
earlier years is 30,000
(4) Interest paid on loan taken for the purchase of a house from a scheduled bank for use
as own residence is 1,20,000
(5) Repayment of instalments of loan taken from the bank for the purchase of the above
property is 60,000
(6) Deposits in public provident fund account:
(i) Towards loan taken from public provident account is 20,000
(ii) Out of current year’s income is 40,000
(7) Investment made in units of a mutual fund approved by the board under section 80C of
the Income-Tax Act is 40,000
Compute the total income of Mrs. Nikita and the tax payable thereon in respect of
assessment year 2024-25.
HW Question: 4
(x) Payment of Tuition fees of his son to a private coaching center for coaching in taxation
is ₹5,000.
Compute his income and tax liability for assessment year 2024-25.
Illustration: 4
Solution:
HW Question: 5
The gross total income of Mr. Dulichand for the A.Y. 2024-25 is ₹5,00,000. He has made
the following investments/ payments during the F.Y. 2023-24
(1) Contribution to PPF of ₹90,000
(2) Payment of tuition fees to Apeejay School, New Delhi, for education of his son
Illustration: 5
The basic salary of Mr. A is ₹1,00,000 p.m. He is entitled to dearness allowance, which
is 40% of basic salary. 50% of dearness allowance forms part of pay for retirement
benefits. Both Mr. A and his employer, ABC Ltd., contribute 15% of basic salary to
the pension scheme referred to in section 80CCD. Explain the tax treatment in respect
of such contribution in the hands of Mr. A if he has exercised the option of shifting
out of the default tax regime provided under section 115BAC(1A).
What would be your answer if Mr. A pays tax under the default tax regime under
section 115BAC?
Solution:
(i) Tax treatment in the hands of Mr. A in respect of employer’s and own contribution
to pension scheme referred to in section 80CCD, where Mr. A has exercised the
option of shifting out of the default tax regime provided under section 115BAC(1A)
[i.e., where Mr. A pays tax under the normal provisions of the Act]
(a) Employer’s contribution to such pension scheme would be treated as salary since it is
specifically included in the definition of “salary” under section 17(1)(viii). Therefore,
₹1,80,000, being 15% of basic salary of ₹ 12,00,000, will be included in Mr. A’s salary.
(b) Mr. A’s contribution to pension scheme is allowable as deduction under section
80CCD(1). However, the deduction is restricted to 10% of salary. Salary, for this purpose,
means basic pay plus dearness allowance, if it forms part of pay.
Therefore, “salary” for the purpose of deduction under section 80CCD for Mr. A would be–
Particulars ₹
Basic salary = ₹ 1,00,000 × 12 = 12,00,000
Dearness allowance = 40% of ₹ 12,00,000 = ₹ 4,80,000
50% of Dearness Allowance forms part of pay = 50% of ₹4,80,000 2,40,000
Salary for the purpose of deduction under section 80CCD 14,40,000
₹1,44,000 is allowable as deduction under section 80CCD(1). This would be taken into
consideration and be subject to the overall limit of ₹1,50,000 under section 80CCE.
₹36,000 allowable as deduction under section 80CCD(1B) is outside the overall limit of
₹1,50,000 under section 80CCE.
In the alternative, ₹ 50,000 can be claimed as deduction under section 80CCD(1B). The
balance ₹ 1,30,000 (₹ 1,80,000- ₹ 50,000) can be claimed as deduction under section
80CCD(1).
(ii) Where Mr. A pays tax under the default tax regime under section 115BAC
Mr. A would not be eligible for deduction under section 80CCD(1)/(1B) in respect of his
contribution to pension scheme under the default tax regime under section 115BAC.
However, he would be allowed deduction of ₹ 1,44,000 under section 80CCD(2) in respect
of employer’s contribution to pension scheme.
Illustration: 6
The gross total income of Mr. X for the A.Y. 2024-25 is ₹8,00,000. He has made the
following investments/payments during the F.Y. 2023-24 –
Particulars ₹
(1) Contribution to PPF 1,10,000
(2) Payment of tuition fees to Apeejay School, New Delhi, for 45,000
education of his son studying in Class XI
(3) Repayment of housing loan taken from Standard Chartered Bank 25,000
(4) Contribution to approved pension fund of LIC 1,05,000
Compute the eligible deduction under Chapter VI-A for the A.Y. 2024-25 if Mr. X
exercises the option of shifting out of the default tax regime provided under section
115BAC(1A).
Solution:
Particulars ₹
Deduction under section 80C
- Contribution to PPF 1,10,000
- Payment of tuition fees to Apeejay School, New Delhi, for education of
his son studying in Class XI 45,000
- Repayment of housing loan 25,000
1,80,000
Restricted to ₹ 1,50,000, being the maximum permissible deduction 1,50,000
u/s 80C
Deduction under section 80CCC
Contribution to approved pension fund of LIC 1,05,000
2,55,000
As per section 80CCE, the aggregate deduction under section 80C, 80CCC
and 80CCD(1) has to be restricted to ₹ 1,50,000
Deduction allowable under Chapter VIA for the A.Y. 2024-25 1,50,000
Illustration: 7
Mr. Arjun (52 years old) furnishes the following particulars in respect of the following
payments:
Compute the deduction available to Mr. Arjun under section 80D for the A.Y. 2024-25.
Solution:
Note: Irrespective of the fact that the mother of Arjun is a very senior citizen the
deduction under section 80D would not be available to him in respect of the medical
expenditure incurred for his mother, since Mr. Arjun has taken a health insurance policy
for his mother.
Illustration: 8
Mr. Rahul, aged 40 years, paid medical insurance premium of ₹18,000 by cheque during the
P.Y. 2023-24 to insure his health as well as the health of his spouse. He also paid medical
insurance premium of ₹26,000 by cheque during the year to insure the health of his
father, aged 63 years, who is not dependent on him. He contributed ₹5,000 by cheque to
Central Government Health Scheme during the year. He has incurred ₹3,000 in cash on
preventive health check-up of himself and his spouse and ₹4,000 by cheque on
preventive health check-up of his father.
Compute the deduction allowable under section 80D for the A.Y. 2024-25.
Solution:
Particulars ₹ ₹
Actual Maximum
Payment deduction
allowable
HW Question: 6
Mr. Krishan, aged 40 years, paid medical insurance premium of ₹20,000 by cheque during
the P.Y. 2023-24 to insure his health as well as the health of his spouse and dependent
children. He also paid medical insurance premium of ₹51,000 by cheque during the year to
insure the health of his father, aged 67 years, who is not dependent on him. He
contributed ₹6,000 by cheque to Central Government Health Scheme during the year.
Compute the deduction allowable under section 80D for the A.Y. 2024-25.
Compute the eligible deduction under Chapter VI-A for the Assessment year 2024-25 of
Ms. Roma, who has a gross total income of ₹15,00,000 for the assessment year 2024-25
and provide the following information about his investments/payments during the financial
year 2023-24:
HW Question: 8
The gross total income of Mr. Tarun for the Assessment Year 2024-25 was ₹12,00,000.
He has made the following investments/ payments during the financial year 2023-24:
1. L.I.C. Premium paid (Policy value ₹1,00,000) (Policy taken after 1.4.2012) 25,000
2. P.P.F. amount paid 70,000
3. Repayment of housing Loan to Indian Bank 50,000
4. Payment made to L.I.C. pension fund 25,000
5. Medical insurance premium for self, wife and dependent Children. 18,000
6. Mediclaim premium for parents (aged over 80 years) 30,000
Compute eligible deduction under Chapter VI-A for the Assessment Year 2024-25 and also
compute his tax liability under old tax regime.
HW Question: 9
Mr. Kartik, an individual, made payment of health insurance premium to General Insurance
Corporation in an approved scheme. Premium paid on his health is ₹10,000 and his spouse’s
health is ₹15,000 during the year 2023-24. He also paid health insurance premium of
₹25,000 on his father’s health who is a senior citizen and not dependent on him. The
payments have not been made by cash. Compute the amount of deduction under Chapter
VI-A of the Act, available to Mr. Kartik from his gross total income for the assessment
year 2024-25.
Illustration: 9
Mr. Mohan is a resident individual. He deposits a sum of ₹25,000 with Life Insurance
Corporation every year for the maintenance of his handicapped grandfather. A copy of the
certificate from the medical authority is submitted. Compute the amount of deduction
available under section 80DD for the A.Y. 2024-25.
Solution:
Since the amount deposited by Mr. Mohan was for his grandfather, he will not be allowed
any deduction under section 80DD. The deduction is available if the individual assessee
incurs any expense for a dependent disabled relative. Grandfather does not come within
the definition of dependent relative.
Illustration: 10
What will be the deduction if Mr. Mohan had made this deposit for his dependent father?
Solution:
Since the expense was incurred for a dependent disabled relative, Mr. Mohan will be
entitled to claim a deduction of ₹75,000 under section 80DD, irrespective of the amount
deposited. In case his father has severe disability, the deduction would be ₹1,25,000.
HW Question: 10
Illustration: 11
Mr. Jatin has taken three education loans on April 1st, 2023, the details of which are given
below:
Compute the amount deductible under section 80E for the A.Y. 2024-25.
Solution:
Illustration: 12
Solution:
Particulars ₹
Interest deduction for A.Y. 2024-25
(i) Deduction allowable while computing income under the
head “Income from house property”
Deduction under section 24(b) ₹ 3,85,000
[₹ 35,00,000 × 11%]
Restricted to 2,00,000
(ii) Deduction under Chapter VI-A from Gross Total Income
Deduction under section 80EE ₹ 1,85,000
(₹ 3,85,000 – ₹ 2,00,000)
Restricted to 50,000
Illustration: 13
The following are the particulars relating to Mr. Anand and Mr. Raju, salaried individuals
for A.Y. 2024-25 –
Compute the amount of deduction, if any, allowable under the provisions of the Income-tax
Act, 1961 for A.Y. 2024-25 in the hands of Mr. Anand, Mr. Raju. Assume that there has
been no principal repayment during the P.Y. 2023-24.
Solution:
Particulars ₹
Mr. Anand
Deduction under Chapter VI-A
Deduction u/s 80EEB for interest payable on loan taken for purchase of
electric vehicle [₹ 20 lakhs X 10% X 11/12 = ₹ 1,83,333, restricted to ₹
1,50,000, being the maximum permissible deduction] 1,50,000
Mr. Raju
Deduction under Chapter VI-A
Deduction u/s 80EEB is not permissible since loan was sanctioned before
01.04.2019 Nil
HW Question: 11
The following are the particulars relating to Mr. A, Mr. B, Mr. C and Mr. D, salaried
individuals, for A.Y. 2024-25 –
Compute the amount of deduction, if any, allowable under the provisions of the Income-tax
Act, 1961 for A.Y. 2024-25 in the hands of Mr. A, Mr. B, Mr. C and Mr. D if they have
exercised the option of shifting out of the default tax regime provided under section
115BAC(1A). Assume that there has been no principal repayment in respect of any of the
above loans upto 31.3.2024.
Illustration: 14
Mr. X has income from business/ profession of ₹6,00,000 and long-term capital gain of
₹4,00,000 and short-term capital gain u/s 111A of ₹2,00,000 and casual income of
₹1,00,000.
He has paid premium of a mediclaim policy amounting to ₹20,000 taken in the name of his
dependent grandfather who is senior citizen and payment was made by a cheque on
09.01.2024.
He has given premium of Jeevan Suraksha policy of ₹7,000, has donated ₹12,000 to the
National Defence Fund, ₹4,000 to Prime Minister’s Drought Relief Fund and ₹3,00,000 to
a charitable institution and ₹1,00,000 to a social organization and ₹4,00,000 to religious
organization and all such organization are notified under section 80G. (all the donations
were made by account payee cheque)
Compute his total income and tax liability for A.Y. 2024-25.
(b) Presume in the above question the assessee has given donation of ₹10,000 by cheque
for promotion of family planning also to the Government.
Compute his total income and tax liability for the assessment year 2024-25.
(c) Presume in part (b), donation to government for family planning is ₹2,00,000 by cheque.
Solution:
Working Note:
Charitable Institution 3,00,000
Social organization 1,00,000
Religious organization 4,00,000 8,00,000
AGTI = GTI – LTCG – STCG u/s 111A – Deduction u/s 80C to 80U (except 80G)
= 13,00,000 – 4,00,000 – 2,00,000 – 7,000 = 6,93,000
Qualifying amount = 10% of AGTI or donation whichever is less
= 69,300 or 8,00,000 whichever is less = 69,300
50% of the qualifying amount = 34,650
Solution (b):
Working Note:
Charitable Institution 3,00,000
Social organization 1,00,000
religious organization 4,00,000
Family planning 10,000
8,10,000
AGTI = GTI – LTCG – STCG u/s 111A – Deduction u/s 80C to 80U (except 80G)
= 13,00,000 – 4,00,000 – 2,00,000 – 7,000 = 6,93,000
Qualifying amount = 10% of AGTI or donation whichever is less
= 69,300 or 8,10,000 whichever is less = 69,300
Deduction of donation for family planning = ₹10,000
50% of balance amount (i.e. ₹59,300) = ₹29,650
Total deduction allowed 10,000 + 29,650 = 39,650
Solution (c):
Working Note: ₹
Charitable Institution 3,00,000
Social organization 1,00,000
religious organization 4,00,000
Family planning 2,00,000
10,00,000
AGTI = GTI – LTCG – STCG u/s 111A – Deduction u/s 80C to 80U (except 80G)
= 13,00,000 – 4,00,000 – 2,00,000 – 7,000 = 6,93,000
Qualifying amount = 10% of AGTI or donation whichever is less
= 69,300 or 10,00,000 whichever is less = 69,300
Donation for family planning is ₹2,00,000 but maximum deduction allowed shall be ₹69,300
Mr. Rohan, a resident individual has Gross Total Income of ₹7,50,000 comprising of
Income from Salary and income from house property for the assessment year 2024-25.
He provides the following information:
Paid ₹70,000 towards premium on life insurance policy of his Handicapped Son (Section
80U disability).
Sum assured of ₹4,00,000; and date of issue of policy is 01-08-2020.
Deposited ₹90,000 in tax saver deposit in the name of his major son in State Bank of
India.
Contributed by cheque ₹25,000 to The Clean Ganga Fund, set up by the Central
Government.
Compute the Total Income and deduction under Chapter VI-A for the Assessment year
2024-25.
Mr. X declares gross total income ₹4,00,000 for the assessment year 2024-25. The gross
total income includes taxable long term capital gain of ₹ 65,000 and short-term capital
gain of ₹35,000 which is taxable @ 15% under section 111A of the Income-tax Act, 1961.
The details of fund investment made during the year 2023-24 are: ₹
(i) Medical insurance premium paid by cheque –
(a) in the name of Mr. X 4,000
(b) in name of Mrs. X 5,000
(ii) Contribution made to –
(a) Prime minister Drought Relief Fund 7,000
(b) Delhi University (declared as an institution of national eminence) by cheque 3,000
(c) Zila Saksharta Samiti by cheque 5,000
(d) An approved charitable institute by cheque 30,000
(e) Government by cheque for the purpose of promoting family planning 10,000
For the assessment year 2024-25, the gross total income of Mr. X was ₹4,50,240 which
includes long term capital gain of ₹45,000 and short-term capital gain of ₹80,000. The
gross total income also includes interest income (fixed deposits) from banks of ₹12,000.
Mr. X has invested in public provident fund ₹60,000 and also paid medical insurance
premium ₹11,000 by cheque. Mr. X also contributed ₹15,000 by cheque to public charitable
trust eligible for deduction u/s 80G.
Compute the total income and tax thereon of Mr. X, who is 50 years old as on 31.03.2024.
Illustration: 15
Mr. X has income under the head Business/Profession of ₹5,00,000 and LTCG of
₹2,00,000, STCG u/s 111A of ₹3,00,000 and casual income of ₹1,00,000.
He is paying rent for a house of ₹40,000 p.m. He has deposited ₹30,000 in the home loan
account scheme of National Housing Bank.
He has complied with all the conditions of section 80GG.
Compute income tax liability for A.Y. 2024-25.
Solution:
Working Note:
Least of the following:
1. ₹4,80,000 – 57,000 = 4,23,000
2. 60,000
3. 25% x 5,70,000 = 1,42,500
AGTI = GTI – LTCG – STCG 111A – Deduction u/s 80C to 80U (except 80GG)
= 11,00,000 – 2,00,000 – 3,00,000 – 30,000
= 5,70,000
HW Question: 15
Mr. X is a retired Government officer aged 40 years, who derived the following income in
respect of the financial year 2023-24. He resides in Cochin: ₹
Salary 6,00,000
Interest from bank deposits (fixed deposits) 2,00,000
He has paid ₹28,000 as premium to affect an insurance on his health and it was paid by a
cheque. He pays a rent of ₹10,000 per month in respect of furnished accommodation. He is
eligible for deduction under Section 80GG. Compute his total income and tax liability for
assessment year 2024-25.
What are the conditions to be satisfied by him to qualify for the deduction?
Illustration: 16
Mr. X has income under the head salary of ₹6,00,000 and income under the head house
property of ₹7,00,000 and he has submitted information as given below:
(i) Paid premium of life policy ₹40,000 (sum assured ₹1,50,000) and policy has taken
before 01.04.2012 in the name of Mr. X
(ii) Paid premium of life policy ₹40,000 (sum assured ₹1,50,000) and policy has taken after
01.04.2012 in the name of Mrs. X
(iii) Paid premium of life policy ₹40,000 (sum assured ₹1,50,000) and policy has taken
before 01.04.2012 in the name of father of Mr. X who is dependent on Mr. X.
(iv) Paid premium of life policy ₹40,000 (sum assured ₹1,50,000) and policy has taken
before 01.04.2012 in the name of son of Mr. X who is not dependent on Mr. X.
(v) He has donated ₹1,00,000 by cheque in rural development fund setup by government.
(vi) He has paid premium of Jeevan Suraksha Policy ₹10,000 by cheque in the name of Mrs.
X.
(vii) He has paid ₹15,000 in cash in connection with preventive health checkup for his
father.
(viii) He has donated ₹60,000 by cheque to a charitable institution notified under section
80G
Compute his tax liability assessment year 2024-25.
Solution:
Working Note:
AGTI = GTI – Deductions u/s 80C to 80U (Except 80G)
= 13,00,000 – 30,000 – 15,000 – 30,000 – 1,00,000 – 5,000
= ₹11,20,000
Qualifying Amount = 10% of AGTI or Donation given whichever is less
= ₹1,12,000 or 60,000
= 60,000
50% of qualifying amount = ₹30,000
Mr. X having gross total income of ₹6,35,000 for the financial year 2023-24 furnishes you
the following information:
(i) Paid ₹25,000 towards premium on life insurance policy of his married daughter.
(ii) Contributed ₹10,000 by cheque to Prime Minister’s National Relief Fund.
(iii) Donated ₹20,000 to a Government recognized institution (by cheque) for scientific
research.
Compute the total income of Mr. X for the assessment year 2024-25.
Note: Assume that the gross total income of Mr. X does not include any income under the
head ‘profits and gains of business or profession’.
Illustration: 17
Solution:
An Indian company is eligible for deduction under section 80GGB in respect of any sum
contributed by it in the previous year to any political party or an electoral trust. Further,
the word “contribute” in section 80GGB has the meaning assigned to it in section 182 of
the Companies Act, 2013, and accordingly, it includes the amount of expenditure incurred
on advertisement in a brochure of a political party.
Therefore, ABC Ltd. is eligible for a deduction of ₹2,25,000 under section 80GGB in
respect of sum of ₹2 lakh contributed to an electoral trust and ₹25,000 incurred by it on
advertisement in a brochure of a political party.
It may be noted that there is a specific disallowance under section 37(2B) in respect of
expenditure incurred on advertisement in a brochure of a political party. Therefore, the
Mr. Satya is a manufacturer of household goods in a factory located in Navi Mumbai and
commenced his business on 1st April 2023 and he employed 120 new work men during the
previous year 2023-24 which included:
(a) 20 employee whose total emoluments paid @ ₹30,000 p.m. per employee.
(b) 40 workers employed on 01st April, 2023
(c) 35 workers employed on 1st May, 2023
(d) 25 workers employed on 5th October, 2023
Compute the Deduction under Section 80JJAA, if available to Mr. Satya for Assessment
year 2024-25, if wages are paid to each worker @ ₹3,000 per month. His profit from the
manufacture of goods for Assessment year 2024-25 is ₹9.50 lakhs. (after debiting all the
wages paid)
The Assessee is liable to Audit his accounts.
If in the previous year 2024-25, he has given employment to 35 workmen on 01-05-2024
and each getting salary 15,000 per month. Compute deduction allowed in previous year
2024-25.
If in the previous year 2025-26, he has given employment to 40 workmen on 01-05-2025
and each getting salary 12,000 per month. Compute deduction allowed in previous year
2025-26.
Solution:
Mr. Satya is eligible for deduction under section 80JJAA since he is liable to Audit of his
accounts in the previous year 2023-24. Deduction allowed shall be as given below:
(a) 20 employee whose total emoluments paid @ ₹30,000 p.m. per employee - Not eligible
(b) 40 workers employed on 01st April, 2023 - Eligible – 3,000 x 12 x 40 = 14,40,000
(c) 35 workers employed on 1st May, 2023 - Eligible – 3,000 x 11 x 35 = 11,55,000
(d) 25 workers employed on 5th October, 2023 - will be considered next year
Total 25,95,000
Deduction allowed @ 30% 7,78,500
Total 66,75,000
Deduction allowed @ 30% 20,02,500
+ Deduction related to P.Y. 2023-24 7,78,500
Illustration: 19
The regular employees participate in recognized provident fund while the casual employees
do not. Compute the deduction, if any, available to Mr. A for A.Y. 2024-25, if the profits
and gains derived from manufacture of computers that year is ₹ 75 lakhs and his total
turnover is ₹ 10.16 crores.
What would be your answer if Mr. A commenced the business of manufacture of footwear
on 1.4.2023?
Solution:
Mr. A is eligible for deduction under section 80JJAA since he is subject to tax audit
under section 44AB for A.Y. 2024-25 and he has employed “additional employees” during
the P.Y. 2023-24.
Working Note:
Particulars No. of
workmen
Total number of employees employed during the year 350
Less: Casual employees employed on 1.8.2023 who do not participate in 50 275
recognized provident fund
Regular employees employed on 1.5.2023, since their total monthly emoluments 125
exceed ₹ 25,000
Regular employees employed on 1.9.2023 since they have been employed for
less than 240 days in the P.Y.2023-24. 100
Number of “additional employees” 75
Notes –
(i) Since casual employees do not participate in recognized provident fund, they do not
qualify as additional employees. Further, 125 regular employees employed on 1.5.2023 also
do not qualify as additional employees since their monthly emoluments exceed ₹ 25,000.
Also, 100 regular employees employed on 1.9.2023 do not qualify as additional employees
for the P.Y.2023-24, since they are employed for less than 240 days in that year.
Therefore, only 75 employees employed on 1.4.2023 qualify as additional employees, and
the total emoluments paid or payable to them during the P.Y.2023-24 is deemed to be the
additional employee cost.
(ii) As regards 100 regular employees employed on 1.9.2023, they would be treated as
additional employees for previous year 2024-25, if they continue to be employees in that
year for a minimum period of 240 days.
Accordingly, 30% of additional employee cost in respect of such employees would be
allowable as deduction under section 80JJAA in the hands of Mr. A for the A.Y. 2025-26.
Illustration: 20
Mrs. X is author of one book of scientific nature and its print price is ₹500 and total
copies sold are 2000 and she has received royalty @ 50%.
She has taken a loan from State Bank in 2015 for pursuing bachelor’s degree in
Engineering and she has given repayment of principal amount ₹80,000 and interest
₹20,000 to State Bank. (payment of interest was given for the first time in financial year
2020-21)
She has paid tuition fee of her son for whole time education ₹ 3,000 in India.
Compute Income Tax liability A.Y. 2024-25.
Solution: ₹
Mr. X received royalty of ₹2,88,000 from abroad for a book authored by him which is a
work of artistic nature. The rate of royalty is 18% of value of books and expenditure made
for earning this royalty was ₹40,000. The amount remitted to India till 30th September,
2024 is ₹2,30,000. Compute deduction u/s 80QQB. Also compute tax liability if he has
income under the head House Property ₹7,00,000.
Solution:
HW Question: 17
Mr. X received royalty of ₹2,00,000 from abroad for a book authored by him which is a
work of artistic nature. The rate of royalty is 20% of value of books and expenditure
made for earning this royalty was ₹50,000. The amount remitted to India till 30th
September, 2023 is ₹1,20,000. Compute deduction u/s 80QQB and also compute income to
be added in Gross Total Income.
HW Question: 18
Mr. X received royalty of ₹6,00,000 from abroad for a book authored by him which is a
work of artistic nature. The rate of royalty is 25% of value of books and expenditure
made for earning this royalty was ₹2,50,000. The amount remitted to India till 30th
September, 2024 is ₹3,80,000. Compute deduction u/s 80QQB and also compute income to
be added in Gross Total Income.
HW Question: 19
Mr. X received royalty of ₹10,00,000 from ABC Ltd. situated in India for a book authored
by him which is a work of artistic nature. The rate of royalty is 10% of value of books and
expenditure made for earning this royalty was ₹5,00,000. Compute deduction u/s 80QQB
and also compute income to be added in Gross Total Income.
HW Question: 20
Mr. X received royalty (lumpsum) of ₹6,00,000 from abroad for a book authored by him
which is a work of artistic nature. Expenditure made for earning this royalty was
₹2,50,000. The amount remitted to India till 30th September, 2024 is ₹4,00,000.
Compute deduction u/s 80QQB and also compute income to be added in Gross Total
Income.
Mr. X, a writer and a professional furnishes the following particulars for the previous year
ended 31.03.2023: ₹
(a) Royalty on books (eligible for deduction u/s 80QQB) 42,000
(b) Expenditure on books 8,000
(c) Income from profession 3,80,000
(d) Deposited in public provident fund (15.03.2023) 70,000
You are required to compute
(i) Taxable income,
(ii) Tax payable for assessment year 2023-24.
Illustration: 22
For the Assessment year 2024-25, the Gross Total Income of Mr. Chaturvedi, a resident
in India, was ₹8,18,240 which includes long-term capital gain of ₹2,45,000 taxable under
section 112 and Short-term capital gain of ₹58,000. The Gross Total Income also includes
interest income of ₹12,000 from savings bank deposits with banks and ₹40,000 interest on
fixed deposits with banks. Mr. Chaturvedi has invested in PPF ₹1,20,000 and also paid a
medical insurance premium ₹51,000. Mr. Chaturvedi also contributed ₹50,000 to Public
Charitable Trust eligible for deduction under section 80G by way of an account payee
cheque. Compute the total income and tax thereon of Mr. Chaturvedi, who is 70 years old
as on 31.3.2024.
Solution:
Computation of total income and tax payable by Mr. Chaturvedi for the A.Y. 2024-
25
Particulars ₹ ₹
Gross total income including capital gain 8,18,240
Less: Long term capital gain u/s 112 (2,45,000)
5,73,240
Less: Deductions under Chapter VI-A:
Under section 80C in respect of PPF deposit 1,20,000 1,20,000
Under section 80D (it is assumed that premium of ₹51,000 is 50,000
paid by otherwise than by cash. The deduction would be
restricted to ₹50,000, since
Mr. Chaturvedi is a senior citizen)
Under section 80G (See Notes 1 & 2 below) 17,662
Under section 80TTB (See Note 3 below) 50,000 (2,37,662)
Total income (excluding long term capital gains) 3,35,578
Total income (including long term capital gains) 5,80,578
Total income rounded off u/s 288A 5,80,580
Notes:
1. Computation of deduction under section 80G:
Particulars ₹
Gross total income (excluding long term capital gains) 5,73,240
Less: Deduction under section 80C, 80D & 80TTB 2,20,000
Adjusted Gross Total Income 3,53,240
10% of the above 35,324
Contribution made 50,000
Lower of the two eligible for deduction under section 80G 35,324
2. Deduction under section 80G is allowed only if amount is paid by any mode other than
cash, in case of amount exceeding ₹2,000. Therefore, the contribution made to public
charitable trust is eligible for deduction since it is made by way of an account payee
cheque.
3. Deduction of upto ₹50,000 under section 80TTB is allowed to a senior citizen if gross
total income includes interest income on bank deposits, both fixed deposits and savings
account.
HW Question: 22
Mr. Rajmohan whose gross total income was ₹6,40,000 for the financial year 2023–24,
furnishes you the following information:
(i) Stamp duty paid on acquisition of residential house (self-occupied) - ₹50,000.
(ii) Five year post office time deposit - ₹20,000.
(iii) Donation to a recognized charitable trust ₹25,000 which is eligible for deduction
under section 80G at the applicable rate.
(iv) Interest on loan taken for higher education of spouse paid during the year - ₹10,000.
Compute the total income of Mr. Rajmohan for the Assessment year 2024-25.
(i) Prakash is retired Government Officer aged 65 years, resides in Cochin, derived
following income:
Particulars ₹
Pension 6,60,000
Interest from bank on fixed deposits (Gross) 55,000
Compute the total income of Mr. Prakash for the assessment year 2024-25 from the
following particulars:
(i) Life insurance premium paid by cheque ₹22,500 for insurance of his life. The insurance
policy was taken on 08-09-2020 and the sum assured is ₹2,00,000.
(ii) Premium of ₹ 26,000 paid by cheque for health insurance of self.
(iii) ₹ 1,500 paid in cash and ₹ 4,500 paid through cheque for preventive health check-up
of his parents, who are senior citizens.
(iv) Paid interest ₹ 6,500 on loan taken from bank for MBA course pursued by his
daughter.
(v) A sum of ₹ 15,000 donated in cash to an institution approved for purpose of section
80G for promoting family planning.
Krishna furnishes the following particulars for the previous year 2022-23 and 2023-24 in
respect of an industrial undertaking established in “Special Economic Zone” during the
financial year 2018-19.
Total Sales of F.Y. of 2023-24 includes freight of ₹ 5 lacs for delivery of goods outside
India. Compute the amount of deduction available to Mr. Krishna under section 10AA.
Solution:
Working Note:
Export Turnover
Sale proceeds received in India 40,00,000
Mrs. Vibha Gupta, a resident individual, is running a SEZ unit, as well as a unit in Domestic
Tariff Area (DTA). She furnishes the following details relating to the year ended 31-3-
2024, pertaining to these two units (₹ in lakhs)
Solution:
(i) 50% of the profit derived from export of articles or things or services is eligible for
deduction under section 10AA, F.Y.2023-24 falls in the next five year period commencing
from the year of manufacture or production of articles or things or provision of services
by the Unit in SEZ. As per section 10AA, the profit derived from export of articles or
things or services shall be
= Profit of the business of Unit in SEZ x Export Turnover of Unit in SEZ
Total Turnover of Unit in SEZ
= 50% of ₹ 220 lakhs x 1000 Lakhs
1100 Lakhs
= 50% x ₹200 lakhs
= ₹100 lakhs
(ii) 100% of the profit derived from export of articles or things or services is eligible for
deduction under section 10AA, F.Y. 2023-24 falls in the first five year period commencing
from the year of manufacture or production of articles or things or provision of services
by the Unit in SEZ. As per section 10AA, the profit derived from export of articles or
things or services shall be
= Profit of the business of Unit in SEZ x Export Turnover of Unit in SEZ
Total Turnover of Unit in SEZ
= 100% of ₹ 220 lakhs x 1000 Lakhs
1100 Lakhs
= 100% x ₹200 lakhs
= ₹200 lakhs
Mr. Suresh has set up an undertaking in SEZ (Unit A) and another undertaking in DTA
(Unit B) in the financial year 2018-19. In the Previous year 2023-24, total turnover of the
unit A is ₹180 lacs and total turnover of Unit B is ₹120 lacs. Export Turnover of Unit A for
the year is ₹150 lacs and Profit for the unit A is ₹60 lacs.
Calculate the deduction available, if any, to Mr. Suresh under section 10AA of the Income-
tax Act, 1961, for the Assessment Year 2024-25, If the manufacturing started in Unit A
in the Financial year 2018-19.
Rudra Ltd. has one unit at Special Economic Zone (SEZ) and other unit at Domestic Tariff
Area (DTA), the company provides the following details for the previous year 2023-24.
Calculate the eligible deduction under section 10AA of the Income-tax Act, 1961, for the
Assessment Year 2024-25, in the following situations:
(i) If both the units were set up and start manufacturing from 22.05.2017.
(ii) If both the units were set up and start manufacturing from 14.05.2021.
Mr. X is running two industrial undertakings, one in a SEZ (Unit A) and another in a
Domestic Tariff Area (Unit B). The brief details for the year ended 31.03.2024 are as
under:
Particulars Amount (₹ in lacs)
Unit A Unit B
Domestic turnover 10 100
Export turnover 120 Nil
Gross Profit 20 10
Less: Expenses and depreciation (07) (05)
Profits derived from the units 13 05
The brought forward business loss pertaining to assessment year 2021-22 for Unit B is
₹3.2 lacs. Briefly compute the business income of the assessee.
Comprehensive Questions
Mr. X has taken a loan of ₹10,00,000 from S.B.I @ 10 % p.a. on 01.07.2019 for
construction of one residential house which was completed on 01.07.2021. It was let out @
₹80,000 p.m. w.e.f. 01.04.2023 and Mr. X has paid Municipal tax of ₹20,000 though the
amount due is ₹30,000. He has repaid Principal amount of ₹70,000 on 01.07.2023. He has
Agricultural income of ₹ 3,00,000 and unadjusted loss of house property of P.Y. 2014-15
₹10,000 and P.Y. 2015-16 ₹21,000.
He has invested ₹10,000 in NSC and ₹5,000 in Public Provident Fund and ₹5,000 in Post
Office 5 Year Time Deposit.
Compute his Income Tax Liability for the A.Y. 2024-25 under old tax regime.
Mr. X took a loan of ₹12,00,000 @ 10 % p.a. on 01.07.2019 and the house completed on
01.05.2023. It was let out @ ₹ 30,000 p.m. w.e.f. 01.08.2023 and the loan was repaid in
annual installment of ₹40,000 starting from 01.01.2021.
Mr. X has STCG 111A ₹10,00,000.
He has paid premium of life policy ₹40,000 on 12.12.2023 and sum assured is ₹1,00,000.
He has paid premium of Jeevan Suraksha Policy ₹20,000.
Compute his Total Income and Tax Liability for the A.Y. 2024-25 under old tax regime.
Mr. X has income from business ₹4,10,000 and he has received income of ₹3,80,000 from
subletting also. He has paid premium of medi-claim policy of ₹31,000. It was paid by
cheque and the policy was taken on 02.01.2024 in the name of his father. Compute his total
income and tax liability for assessment year 2024-25 under old tax regime.
Question: 4 [Deductions]
Mr. X has income from business ₹7,00,000. Mr. X has incurred ₹65,000 on the treatment
of his dependent brother who is suffering from a disease notified under Rule 11DD and he
has received a claim under medi-claim policy ₹35,000.
Compute his income and tax liability for assessment year 2024-25.
(b) Presume assessee incurred ₹65,000 on the treatment of his independent brother.
Mrs. X has let out one residential house property @ ₹1,00,000 p.m. and she has paid
municipal tax of ₹1,00,000.
She has taken a Medi-claim policy on 17.07.2023 in the name of Mr. X and paid premium of
₹18,000 by cheque.
She has also taken a Medi-claim policy on 15.10.2023 in the name of her Father in law who
is aged 66 years and paid premium of ₹16,000 by cheque.
She has incurred ₹21,000 on the treatment of her brother who is dependent on her and
suffering from severe disability.
She has purchased N.S.C. in P.Y. 2021-22 and there is accrued interest of ₹30,000 and
also there is accrued interest on PPF of ₹10,000.
She has taken Jeevan Suraksha Policy in the name of Mr. X and paid premium of ₹19,000.
She has taken a loan in 2017-18 from SBI for the education of his son who is studying in
B.com (Hons) in SRCC and she had paid principal amount of ₹60,000 and interest ₹10,000
in P.Y. 2023-24.
She has Agricultural Income ₹1,00,000
Compute her Income Tax Liability for the A.Y. 2024-25 under old tax regime.
Compute his total income and tax liability for the assessment year 2024-25 under old tax
regime.
Mr. X is a Practicing Chartered Accountant, and he started his practice from 01.04.2023
and he has income from profession ₹8,00,000.
He has LTCG of ₹3,00,000, STCG 111A of ₹1,00,000, casual income ₹2,00,000.
Investment and donations are as given below:-
• NSC ₹10,000.
• Medi-claim premium (by cheque) of ₹15,000.
• Prime Minister’s National Relief Fund ₹10,000. (Paid by cheque)
• Prime minister drought relief fund ₹8,000. (Paid by cheque)
• Donation to Birla Temple (Notified u/s 80G) ₹1,60,000. (Paid by cheque)
• Charitable institution (Notified u/s 80G) ₹40,000. (Paid by cheque)
• Social organization (Notified u/s 80G) ₹20,000. (Paid by cheque)
• Municipal Corporation of Delhi (notified under section 80G) ₹10,000. (Paid by cheque)
Compute income tax liability for A.Y. 2024-25 under old tax regime.
(b) Presume in the above question the assessee has given donation to the Government also
for family planning is ₹20,000.
(c) Presume in the above question the assessee has given donation to the Government also
for family planning is ₹3,00,000.
Question: 8 [Deductions]
Mr. X has short term capital gain of ₹6 lakhs, and he has donated ₹20,000 by cheque to a
charitable institution which is notified under section 80G and he has spent ₹25,000 on the
treatment of his handicapped dependent brother.
Compute his total income and tax liability for the assessment year 2024-25 under old tax
regime.
Question: 9 [Deductions]
3. He has donated ₹10,000 by cheque to MCD for family planning and has donated ₹3,000
by cheque to a charitable institution notified under section 80G.
Compute his total income and tax liability for the assessment year 2024-25 under old tax
regime.
Question: 10 [Deductions]
Question: 11 [Deductions]
Question: 12 [Deductions]
Question: 13 [Deductions]
Mr. X is engaged in the business of manufacturing chemicals and has income under the
head business/profession of ₹5,00,000 and has paid rent of ₹10,000 p.m. for taking a
house on rent because he did not have any house in his name or in the name of his spouse
or minor child or the Hindu Undivided Family of which he is a member. He has invested
₹75,000 in NSC which were taken in the name of his spouse.
Compute his total income and tax liability for assessment year 2024-25 under old tax
regime.
Question: 14 [Deductions]
During the previous year 2023-24, Mr. X has income under the head house property
₹4,00,000. He has donated ₹12,000 by cheque to a notified institution for the purpose of
scientific research. Compute his total income and tax liability for the assessment year
2024-25 under old tax regime.
For the assessment year 2024-25, Mr. X submits the following information:
Income from business 9,800
Property income House I House II
Fair Rent 75,000 82,000
Rent Received/Receivable 78,000 85,000
Municipal Valuation 76,000 75,000
Municipal Taxes (due but outstanding) 13,000 14,000
Repairs 3,500 47,000
Insurance 2,000 3,000
Land Revenue (Paid) 2,500 4,000
Ground Rent (due but outstanding) 1,600 6,000
Interest on capital borrowed by mortgaging house I
(Funds are used for construction of house II) 14,000 ———
Nature of Occupation Let out for Residence Let out for Business
Date of completion of construction 30.04.2019 07.04.2021
Assume that standard rent is less than rent actually received.
Question: 16 [Deductions]
Question: 17 [Deductions]
Mr. X has received royalty of ₹8,00,000 in connection with a patent right registered in his
name.
Other information are as given below:
1. He has donated ₹30,000 to a political party by cheque.
2. He donated ₹10,000 by cheque to Delhi University notified under section 80G
3. He donated ₹10,000 by cheque to the Government for the purpose of promoting family
planning.
4. He paid the premium of medi-claim policy ₹6,000 by cheque in the name of his major
married independent son.
5. LIC premium paid ₹25,000 on 15.01.2023 (Policy value ₹1,00,000)
6. Repayment of housing loan to Indian Bank ₹50,000
7. Payment made to LIC pension fund notified under section 80CCC ₹20,000
Compute income tax liability for A.Y. 2024-25 under old tax regime.
Illustration: 1
Mr. Vakil has transferred, through a duly registered document, the income arising from a
godown to his friend, without transferring the godown. In whose hands will the rental
income from godown be charged?
Solution:
Section 60 expressly states that where there is transfer of income from an asset without
transfer of the asset itself, such income shall be included in the total income of the
transferor. Hence, the rental income derived from the godown shall be clubbed in the
hands of Mr. Vakil.
Illustration: 2
Mr. Ompal holds shares carrying 25% voting power in Om (P) Ltd. Mrs. Ompal is working as
a computer software programmer in Om (P) Ltd. at a salary of 30,000 p.m. She is, however,
not qualified for the job. The other income of Mr. Ompal & Mrs. Ompal is Rs. 7,00,000 &
Rs. 4,00,000, respectively. Compute the gross total income of Mr. Ompal and Mrs. Ompal
for the A.Y. 2024-25, assuming that they do not opt for section 115BAC.
Solution:
Mr. Ompal holds shares carrying 25% voting power in Om (P) Ltd i.e., a substantial interest
in the company. His wife is working in the same company without any professional
qualifications for the same. Thus, by virtue of the clubbing provisions of the Act, the
salary received by Mrs. Ompal from Om (P) Ltd. will be clubbed in the hands of Mr. Ompal.
Hence, computation of gross total income of Mr. Ompal shall be:
Illustration: 3
Will your answer be different if Mrs. Ompal was qualified for the job?
Solution:
If Mrs. Ompal possesses professional qualifications for the job, then the clubbing
provisions shall not be applicable. Hence, computation of gross total income of Mrs. Ompal
shall be:
Illustration: 4
Mr. Boman holds shares carrying 30% voting power in Bombay (P) Ltd. Mrs. Boman is
working as accountant in Bombay (P) Ltd. getting income under the head salary (computed)
of Rs. 3,44,000 without any qualification in accountancy. Mr. Boman also receives Rs.
30,000 as interest on securities. Mrs. Boman owns a house property which she has let out.
Rent received from tenants is Rs. 6,000 p.m. Compute the gross total income of Mr. and
Mrs. Boman for the A.Y. 2024-25.
Solution:
Since Mrs. Boman is not professionally qualified for the job, the clubbing provisions shall
be applicable. Hence, computation of gross total income of Mr. Boman shall be:
Particulars Rs.
Income under the head salary of Mrs. Boman (Computed) 3,44,000
Income from other sources
- Interest on securities 30,000
Gross Total Income 3,74,000
Particulars Rs Rs
Income from Salary [clubbed in the hands of Mr. B] Nil
Income from house property:
Gross Annual Value [Rs 6,000 × 12] 72,000
Less: Municipal taxes paid 0
Net Annual Value (NAV) 72,000
Less: Deductions under section 24
i) 30% of NAV i.e., 30% of Rs 72,000 21,600
ii) Interest on loan 0 50,400
Gross total income 50,400
Illustration: 5
A proprietary business was started by Mrs. Neelam in the year 2021. As on 01.04.2023 her
capital in business was ₹4,00,000. Her husband gifted ₹3,00,000, on 01.04.2023, which
Mrs. Neelam invested in her business on the same date. Mrs. Neelam earned profits from
her proprietary business for the
Financial year 2023-24 ₹2,00,000
Financial year 2024-25 ₹2,40,000
Financial year 2025-26 ₹2,80,000
Financial year 2026-27 ₹3,00,000
Amount of profit was further invested in the business.
Compute amount to be clubbed in the income of his husband in each of the year.
Solution:
Mr. Manish bought 200 listed shares on 19.04.2022 @ ₹2,000 per share. He gifted these
shares to Mrs. Manish on 21.03.2023. On 01.04.2023, bonus shares were allotted in the
ratio of 1:1. All these shares were sold by Mrs. Manish as under:
Briefly state the income-tax consequences in respect of the sale of the shares by Mrs.
Manish, showing clearly the person in whose hands the same is chargeable, the quantum
and the head of income in respect of the above transactions. Detailed computation of total
income is NOT required. Also Discuss tax consequences in the hands of Mrs. Ramesh.
Solution:
As per the provision of Income tax Act, 1961, where an asset has been transferred by an
individual to his spouse otherwise than for adequate consideration, the income arising from
the sale of the said asset by the spouse will be clubbed in the hands of the individual.
Also, where there is any accretion to the asset transferred, income arising to the
transferee from such accretion will not be clubbed. Hence, the profit from sale of bonus
shares allotted to Mrs. Manish will be chargeable to tax in the hands of Mrs. Manish.
Therefore, the capital gains arising from the sale of the original shares has to be included
in the hands of Mr. Manish, and the capital gains arising from the sale of bonus shares
would be taxable in the hands of Mrs. Manish.
Taxability in the hands of Mrs. Ramesh under the head “Income from other sources”
Mrs. Ramesh has received shares from her friend for ₹1,70,000 but market value is
₹2,50,000 hence ₹80,000 is taxable as gift under the head other sources as per section
56.
Solution:
Nishant gifted ₹ 10 lakhs to his wife, Nisha on her birthday on, 1st January, 2023. Nisha
lent ₹5,00,000 out of the gifted amount to Krish on 1st April, 2023 for six months on
which she received interest of ₹ 50,000. The said sum of ₹ 50,000 was invested in shares
of a listed company on 15th October, 2022, which were sold for ₹ 75,000 on 30th
December, 2023. Securities transaction tax was paid on such sale. The balance amount of
gift was invested as capital by Nisha in a newly business started on 1.4.2023. She suffered
loss of ₹ 15,000 in the business in financial year 2023-24.
In whose hands the above income and loss shall be included in Assessment Year 2024-25?
Support your answer with brief reasons.
Solution:
Interest on loan
As per section 64(1)(iv), in computing the total income of any individual, there shall be
included all such income as arises directly or indirectly, to the spouse of such individual
from assets transferred directly or indirectly, to the spouse by such individual otherwise
than for adequate consideration or in connection with an agreement to live apart.
Accordingly, ₹50,000, being the amount of interest on loan received by Ms. Nisha, wife of
Mr. Nishant, would be includible in the total income of Mr. Nishant, since such loan was
given by her out of the sum of money received by her as gift from her husband.
Illustration: 9
A proprietary business was started by Smt. Kavita in the year 2021. As on 1.4.2022 her
capital in business was ₹3,00,000. Her husband gifted ₹2,00,000 on 10.4.2022 to her and
such sum is invested by Smt. Kavita in her business on the same date. Smt. Kavita earned
profits from her proprietary business for the financial year 2022-23, ₹1,50,000 and
financial year 2023-24 ₹3,90,000. Compute the income, to be clubbed in the hands of
Kavita’s husband for the Assessment year 2024-25 with reasons.
Solution:
Section 64(1) of the Income-tax Act, 1961 provides for the clubbing of income in the
hands of the individual, if the income earned is from the assets transferred directly or
indirectly to the spouse of the individual, otherwise than for adequate consideration. In
this case Smt. Kavita received a gift of ₹2,00,000 from her husband which she invested in
her business. The income to be clubbed in the hands of Smt. Kavita’s husband for A.Y.
2024-25 is computed as under:
Illustration: 10
Rayaan gifted ₹15 lakhs to his wife, Sargam on her birthday on, 23rd February, 2023.
Sargam lent ₹8,00,000 out of the gifted amount to Karuna on 1st April, 2023 for six
months on which she received interest of ₹80,000. The said sum of ₹80,000 was invested
in shares of a listed company on 5th October, 2023, which were sold for ₹96,000 on 28th
March, 2024. Securities transactions tax was paid on purchase and sale of such shares.
The balance amount of gift was invested on 1st April 2023, as capital by Sargam in her new
business. She suffered loss of ₹52,000 in the business in Financial Year 2023-24.
In whose hands the above income and loss shall be included in Assessment Year 2024-25,
assuming that capital invested in the business was entirely out of the funds gifted by her
husband. Support your answer with brief reasons.
Solution:
In computing the total income of any individual, there shall be included all such income as
arises directly or indirectly, to the spouse of such individual from assets transferred
directly or indirectly, to the spouse by such individual otherwise than for adequate
consideration or in connection with an agreement to live apart.
Interest on loan: Accordingly, ₹80,000, being the amount of interest on loan received by
Mrs. Sargam, wife of Mr. Rayaan, would be includible in the total income of Mr. Rayaan,
since such loan was given out of the sum of money received by her as gift from her
husband.
Loss from business: As per Explanation 2 to section 64, income includes loss. Thus,
clubbing provisions would be attracted even if there is loss and not income.
Thus, the entire loss of ₹52,000 from the business carried on by Mrs. Sargam would also
be includible in the total income of Mr. Rayaan, since as on 1st April 2023, the capital
invested was entirely out of the funds gifted by her husband.
Short-term capital gain: Income from the accretion of the transferred asset is not liable
to be included in the hands of the transferor and, therefore, short-term capital gain of
₹16,000 (₹96,000, being the sale consideration less ₹80,000, being the cost of acquisition)
arising in the hands of Mrs. Sargam from sale of shares acquired by investing the interest
income of ₹80,000 earned by her (from the loan given out of the sum gifted by her
husband), would not be included in the hands of Mr. Rayaan. Thus, such income is taxable in
the hands of Mrs. Sargam.
Illustration: 11
Mrs. Kasturi transferred her immovable property to Anand Co. Ltd. subject to a condition
that out of the rental income, a sum of ₹36,000 per annum shall be utilized for the
benefit of her son’s wife. Mrs. Kasturi claims that the amount of ₹36,000 (utilized by her
son’s wife) should not be included in her total income as she no longer owned the property.
Examine with reasons whether the contention of Mrs. Kasturi is valid in law.
Solution:
The clubbing provisions under section 64(1)(viii) are attracted in case of transfer of any
asset, directly or indirectly, otherwise than for adequate consideration, to any person to
the extent to which the income from such asset is for the immediate or deferred benefit
of son’s wife. Such income shall be included in computing the total income of the
transferor individual.
Note - In order to attract the clubbing provisions under section 64(1)(viii), the transfer
should be otherwise than for adequate consideration. In this case, it is presumed that the
transfer is otherwise than for adequate consideration and therefore, the clubbing
provisions are attracted. Moreover, the provisions of section 56(2)(x) would also get
attracted in the hands of Anand Co Ltd., if the conditions specified thereunder are
satisfied. If it is presumed that the transfer was for adequate consideration, the
provisions of section 64(1)(viii) and 56(2)(x) would not be attracted.
Illustration: 12
Mr. Anmol has three minor children – two twin daughters, aged 12 years, and one son, aged
16 years. Income of the twin daughters is Rs. 2,000 p.a. each and that of the son is Rs.
1,200 p.a. Mrs. Anmol has transferred her flat to her minor son on 1.4.2023 out of natural
love and affection. The flat was let out on the same date and the rental income from the
flat is Rs. 10,000 p.m. Compute the income, in respect of minor children, to be included in
the hands of Mr. Anmol and Mrs. Anmol u/s. 64(1A) (assuming that Mr. Anmol’s total
income is higher than Mrs. Anmol’s total income, before including the income of minor
children).
Solution:
Note – As per section 27(i), Mrs. Anmol is the deemed owner of house property
transferred to her minor son. Accordingly, the income from house property of Rs. 84,000
[i.e., Rs. 1,20,000 (-) Rs. 36,000, being 30% of Rs. 1,20,000] would be taxable directly in
her hands as the deemed owner of the said property. Consequently, clubbing provisions
under section 64(1A) would not be attracted in respect of income from house property,
owing to which exemption u/s 10(32) cannot be availed by her.
Illustration: 13
Compute the gross total income of Mr. & Mrs. Babulal from the following information:
Particulars Rs.
(a) Salary income (computed) of Mrs. Babulal 2,30,000
(b) Income from profession of Mr. Babulal 3,90,000
(c) Income of minor son Babu from company deposit 15,000
(d) Income of minor daughter Lali from special talent 32,000
(e) Interest from bank received by Lali on deposit made out of her 3,000
special talent
(f) Gift received by Lali on 30.09.2023 from friend of Mrs. Babulal 2,500
Brief working is sufficient. Detailed computation under various heads of income is not
required.
Solution:
(i) As per the provisions of section 64(1A) of the Income-tax Act, 1961, all the income of a
minor child has to be clubbed in the hands of that parent whose total income (excluding
the income of the minor) is greater. The income of Mr. Babulal is Rs. 3,90,000 and income
of Mrs. Babulal is Rs. 2,30,000. Since the income of Mr. Babulal is greater than that of
Mrs. Babulal, the income of the minor children has to be clubbed in the hands of Mr.
Babulal. It is assumed that this is the first year when clubbing provisions are attracted.
(ii) Moreover, Income derived by a minor child from any activity involving application of
his/her skill, talent, specialized knowledge and experience is not to be clubbed. Hence, the
income of minor child Lali from exercise of special talent will not be clubbed.
(iii) However, interest from bank deposit has to be clubbed even when deposit is made out
of income arising from application of special talent.
The gross total income of Mr. Babulal giving effect to the provisions of section 64(1A) is
as follows:
Particulars Rs Rs
Income from profession 3,90,000
Income of minor son Babu from company deposit 15,000
Less: Exemption under section 10(32) 1,500 13,500
Income of minor daughter Lali
- From special talent – not to be clubbed -
- Interest from bank 3,000
- Gift of Rs. 2,500 received from a non-relative is not -
taxable under section 56(2)(x) being less than the aggregate
limit of Rs.50,000
3,000
Less: Exemption under section 10(32) 1,500 1,500
Gross Total Income 4,05,000
Illustration: 14
(i) Income under the head salary of Mrs. Chunilal (Computed) 4,60,000
(ii) Income of minor son ‘Lal’ who suffers from disability specified in
Section 80U 1,08,000
(iii) Income of minor daughter ‘Chuni' from singing 86,000
(iv) Income from profession of Mr. Chunilal 7,50,000
(v) Cash gift received by 'Lal' on 02.10.2023 from friend of Mrs. Chunilal 48,000
(vi) Income of minor married daughter ‘Chuni’ from company deposit 30,000
Compute the total income of Mr. Chunilal and Mrs. Chunilal for the assessment year 2024-
25.
Solution:
Computation of Total Income of Mr. Chunilal and Mrs. Chunilal for the A.Y. 2024-25
Notes: (i) The income of a minor child suffering from any disability of the nature specified
in section 80U shall not be included in the hands of the parents.
(ii) The income derived by the minor from manual work or from any activity involving
exercise of his skill, talent or specialized knowledge or experience will not be included in
the income of his parent.
(iii) Under section 56(2)(x), cash gifts received from any person/persons exceeding
₹50,000 during the year in aggregate are taxable. Since the cash gift in this case does not
exceed ₹50,000 the same is not taxable.
(iv) The clubbing provisions are attracted even in respect of income of minor married
daughter. The income of the minor will be included in the income of that parent whose
total income is greater. Hence, income of minor married daughter ‘Chuni’ from company
deposit shall be clubbed in the hands of the Mr. Chunilal and exemption under section
10(32) of ₹1,500 per child shall be allowed in respect of such income.
Determine the gross total income of Shri Ram Kumar and Smt. Ram Kumar for the
assessment year 2024-25 from the following:
(i) Salary received by Shri Ram Kumar from a company ₹1,80,000 per annum and Smt. Ram
Kumar also doing job in a company and getting salary of ₹2,40,000 per annum
(ii) Shri Ram Kumar transferred a flat to his wife Smt. Ram Kumar on 1st September,
2023 for adequate consideration. The rent received from this let-out flat is ₹9,000 per
month.
(iii) Shri Ram Kumar and his wife Smt. Ram Kumar both are partners in a firm. Shri Ram
Kumar received ₹36,000 and Smt. Ram Kumar received ₹64,000 as interest from the firm
and also had a share of profit of ₹12,000 and ₹26,000 respectively.
(iv) Smt. Ram Kumar transferred 10% debentures worth ₹3,00,000 to Shri Ram Kumar.
Shri Ram also invested Rs. 90,000 in similar instrument and earned interest income of
₹39,000.
(v) Mother of Shri Ram Kumar transferred a house property to Master Rohit (son of Shri
Ram Kumar) in the year 2022. Master Rohit (Aged 13 years) received of ₹15,000 as income
(computed) from this property on 20th February, 2024.
Solution:
Note:
(i) In case of transfer of debentures date of transfer is not given and whether it is
transferred for adequate consideration or not is not mentioned. Above solution is given on
the assumption that it is transferred for inadequate consideration and clubbing provisions
shall be applicable.
Illustration: 16
Mr. Sharma has four children consisting 2 daughters and 2 sons. The annual income of 2
daughters were Rs. 9,000 and Rs. 4,500 and of sons were Rs. 6,200 and Rs. 4,300,
respectively. The daughter who has income of Rs. 4,500 was suffering from a disability
specified under section 80U. Compute the amount of income earned by minor children to
be clubbed in hands of Mr. Sharma.
Solution:
As per section 64(1A), in computing the total income of an individual, all such income
accruing or arising to a minor child shall be included. However, income of a minor child
suffering from disability specified under section 80U would not be included in the income
of the parent but would be taxable in the hands of the minor child. Therefore, in this case,
the income of daughter suffering from disability specified under section 80U should not
be clubbed with the income of Mr. Sharma.
Under section 10(32), income of each minor child includible in the hands of the parent
under section 64(1A) would be exempt to the extent of the actual income or Rs. 1,500,
whichever is lower. The remaining income would be included in the hands of the parent.
Computation of income earned by minor children to be clubbed with the income of Mr.
Sharma:
Particulars Rs.
(i) Income of one daughter 9,000
Less: Income exempt under section 10(32) 1,500
Total (A) 7,500
(ii) Income of two sons (Rs. 6,200 + Rs. 4,300) 10,500
Less: Income exempt under section 10(32)
(Rs. 1,500 + Rs. 1,500) 3,000
Total (B) 7,500
Total Income to be clubbed as per section 64(1A) (A+B) 15,000
Illustration: 17
Mr. Vasudevan gifted a sum of Rs. 6 lakhs to his brother's wife on 14.6.2023. On
12.7.2023, his brother gifted a sum of Rs. 5 lakhs to Mr. Vasudevan's wife. The gifted
amounts were invested as fixed deposits in banks by Mrs. Vasudevan and wife of Mr.
Vasudevan's brother on 01.8.2023 at 9% interest. Examine the consequences of the above
under the provisions of the Income-tax Act, 1961 in the hands of Mr. Vasudevan and his
brother.
Solution:
In the given case, Mr. Vasudevan gifted a sum of Rs. 6 lakhs to his brother’s wife on
14.06.2023 and simultaneously, his brother gifted a sum of Rs. 5 lakhs to Mr. Vasudevan’s
wife on 12.07.2023. The gifted amounts were invested as fixed deposits in banks by Mrs.
Vasudevan and his brother’s wife. These transfers are in the nature of cross transfers.
Accordingly, the income from the assets transferred would be assessed in the hands of
the deemed transferor because the transfers are so intimately connected to form part of
a single transaction and each transfer constitutes consideration for the other by being
mutual or otherwise.
If two transactions are inter-connected and are part of the same transaction in such a
way that it can be said that the circuitous method was adopted as a device to evade tax,
the implication of clubbing provisions would be attracted. It was so held by the Hon’ble
Apex Court in CIT vs. Keshavji Morarji (1967) 66 ITR 142.
Accordingly, the interest income arising to Mrs. Vasudevan in the form of interest on
fixed deposits would be included in the total income of Mr. Vasudevan and interest income
arising in the hands of his brother’s wife would be taxable in the hands of Mr. Vasudevan’s
brother as per section 64(1), to the extent of amount of cross transfers i.e., Rs. 5 lakhs.
This is because both Mr. Vasudevan and his brother are the indirect transferors of the
income to their respective spouses with an intention to reduce their burden of taxation.
However, the interest income earned by his spouse on fixed deposit of Rs. 5 lakhs alone
would be included in the hands of Mr. Vasudevan’s brother and not the interest income on
the entire fixed deposit of Rs. 6 lakhs, since the cross transfer is only to the extent of
Rs. 5 lakhs.
Illustration: 18
Mr. Karan gifted a sum of ₹ 9 lakhs to his brother’s minor son on 01.05.2023. On the same
date, his brother gifted debentures worth ₹10 lakhs to Mrs. Karan. Son of Mr. Karan’s
brother invested the amount in fixed deposit with Canara Bank @ 9% p.a. interest and
Mrs. Karan received interest of ₹81,000 on these debentures during the previous year
2023-24. Discuss the tax implications under the provisions of the Income- tax Act, 1961.
Solution:
In the given case, Mr. Karan gifted a sum of ₹ 9 lakhs to his brother’s minor son on
01.05.2023 and simultaneously, his brother gifted debentures worth ₹10 lakhs to Mr.
Karan’s wife on the same date. Mr. Karan’s brother’s minor son invested the gifted amount
of ₹9 lakhs in fixed deposit with Canara Bank.
These transfers are in the nature of cross transfers. Accordingly, the income from the
assets transferred would be assessed in the hands of the deemed transferor because the
transfers are so intimately connected to form part of a single transaction and each
transfer constitutes consideration for the other by being mutual or otherwise.
If two transactions are inter-connected and are part of the same transaction in such a
way that it can be said that the circuitous method was adopted as a device to evade tax,
the implication of clubbing provisions would be attracted.
As per section 64(1A), all income of a minor child is includible in the hands of the parent,
whose total income, before including minor’s income is higher. Accordingly, the interest
income arising to Mr. Karan’s brother’s son from fixed deposits would be included in the
total income of Mr. Karan’s brother, assuming that Mr. Karan’s brother’s total income is
higher than his wife’s total income, before including minor’s income. Mr. Karan’s brother
can claim exemption of ₹1,500 under section 10(32). Interest on debentures arising in the
hands of Mrs. Karan would be taxable in the hands of Mr. Karan as per section 64(1)(iv).
This is because both Mr. Karan and his brother are the indirect transferors of the income
to their spouse and minor son, respectively, with an intention to reduce their burden of
taxation.
In the hands of Mr. Karan, interest received by his spouse on debentures of ₹9 lakhs alone
would be included and not the entire interest income on the debentures of ₹10 lakhs, since
the cross transfer is only to the extent of ₹9 lakhs.
Hence, only proportional interest (i.e., 9/10th of interest on debentures received) ₹72,900
would be includible in the hands of Mr. Karan.
The provisions of section 56(2)(x) are not attracted in respect of sum of money
transferred or value of debentures transferred, since in both the cases, the trans fer is
from a relative
Illustration: 19
Mr. Jagmal gifts ₹1 lakh to his wife Mrs. Jagmal on April 1, 2023 which she invests in a
firm on interest rate of 14% per annum. On January 1, 2024, Mrs. Jagmal withdraws the
money and gift it to her son’s wife. She claims that interest which has accrued to the
daughter-in-law, from January 1, 2024 to March 31, 2024 on investment made by her is
not assessable in her hands but in the hands of Mr. Jagmal. Is this correct? What would
be the position, if Mrs. Jagmal has gifted the money to minor grandson, instead of the
daughter-in-law?
Solution:
Section 64(1) provides that in computing the total income of any individual, there shall be
clubbed all such income as arises directly or indirectly to the son’s wife, of such individual,
from assets transferred directly or indirectly to the son’s wife by such individual
otherwise than for adequate consideration. There is an indirect transfer by Mr. Jagmal to
the daughter-in-law and therefore, the interest income shall be clubbed with income of
Mr. Jagmal.
If Mrs. Jagmal had gifted the money to her minor grandson, then the interest income
arising to the minor shall be clubbed under section 64(1A) in the total income of that
parent (son/ daughter-in-law of Mrs. Jagmal) whose total income (before including such
income) is higher.
Comprehensive questions:
Question: 1
During the previous year 2023-24, the following transactions occurred in respect of Mr. A.
(a) Mr. A had a fixed deposit of Rs. 5,00,000 in Bank of India. He instructed the bank to
credit the interest on the deposit @ 9% from 1-4-2023 to 31-3-2024 to the savings bank
account of Mr. B, son of his brother, to help him in his education.
(b) Mr. A holds 75% profit share in a partnership firm. Mrs. A received a commission of
Rs. 25,000 from the firm for promoting the sales of the firm. Mrs. A possesses no
technical or professional qualification.
(c) Mr. A gifted a flat to Mrs. A on April 1, 2023. During the previous year 2023- 24, Mrs.
A’s “Income from house property” (computed) was Rs. 52,000 from such flat.
(d) Mr. A gifted Rs. 2,00,000 to his minor son who invested the same in a business and he
derived income of Rs. 20,000 from the investment.
(e) Mr. A’s minor son derived an income of Rs. 20,000 through a business activity involving
application of his skill and talent. During the year, Mr. A got a monthly pension of Rs.
10,000. He had no other income. Mrs. A received salary of Rs. 20,000 per month from a
part time job.
Examine the tax implications of each transaction and compute the total income of Mr. A,
Mrs. A and their minor child assuming they do not wish to opt for section 115BAC.
Question: 2
Mr. A has gifted a house property valued at Rs. 50 lakhs to his wife, Mrs. B, who in turn
has gifted the same to Mrs. C, their daughter-in-law. The house was let out at Rs. 25,000
per month throughout the year. Compute the total income of Mr. A and Mrs. C.
Will your answer be different if the said property was gifted to his son, husband of Mrs.
C?
Question: 3
Mr. Babu is the Karta of a HUF, whose members derive income as given below:
Particulars Rs.
Examine the tax implications in the hands of Mr. and Mrs. Babu.
Mr. Soham is an employee of Targen Limited and has substantial interest in the company.
His salary is ₹25,000 p.m. Mrs. Soham also is working in that company at a salary of
₹10,000 p.m. without any professional qualification.
Mr. Soham also receives ₹30,000 as income from securities, Mrs. Soham owns a house
property which she has let out. Rent received from such house property is ₹12,000 p.m.
Mr. & Mrs. Soham have three minor children-two twin daughters and one son. The income
of the twin daughters is ₹2,000 p.a. each and that of his son is ₹ 1,200 p.a. Compute the
income of Mr. & Mrs. Soham.
Illustration: 1
During the P.Y. 2023-24, Mr. C has the following income and the brought forward losses:
Particulars ₹
Short term capital gains on sale of shares 1,50,000
Long term capital loss of A.Y. 2022-23 (96,000)
Short term capital loss of A.Y. 2023-24 (37,000)
Long term capital gain u/s 112 75,000
What is the capital gain taxable in the hands of Mr. C for the A.Y. 2024-25?
Solution:
Particulars ₹ ₹
Short term capital gains on sale of shares 1,50,000
Less: Brought forward short-term capital loss of (37,000) 1,13,000
the A.Y. 2023-24
Long term capital gain 75,000
Less: Brought forward long-term capital loss of A.Y.
2022-23 ₹96,000 set off to the extent of ₹75,000 (75,000) Nil
[See Note below]
Taxable short-term capital gains
1,13,000
Note: Long-term capital loss cannot be set off against short-term capital gain. Hence, the
unadjusted long-term capital loss of A.Y. 2022-23 of ₹21,000 (i.e. ₹96,000 – ₹75,000) can
be carried forward to the next year to be set-off against long-term capital gains of that
year.
Illustration: 2
Mr. A, aged 35 years, submits the following particulars pertaining to the A.Y. 2024-25:
Particulars ₹
Income from salary (computed) 4,00,000
Loss from let-out property (-) 2,20,000
Business loss (-)1,00,000
Bank interest (FD) received 80,000
Compute the total income of Mr. A for the A.Y. 2024-25, assuming that
(i) He has exercised the option of shifting out of the default tax regime provided under
section 115BAC(1A).
(ii) He pays tax under the default tax regime.
Solution:
(i) Computation of total income of Mr. A for the A.Y. 2024-25 under normal provisions
of the Act:
Particulars ₹ ₹
Income from salary 4,00,000
Less: Loss from house property of ₹2,20,000 to (-) 2,00,000 2,00,000
be restricted to ₹2 lakhs by virtue of section
71(3A)
Balance loss of ₹20,000 from house property
to be carried forward to next assessment year
Income from other sources (interest on fixed 80,000
deposit with bank)
Less: Business loss of ₹1,00,000 set-off to the (-) 80,000
extent of ₹ 80,000
Business loss of ₹20,000 to be carried
forward for set-off against business income of
Notes:
(a) Gross Total Income includes salary income of ₹2,00,000 after adjusting loss of
₹2,00,000 from house property. The balance loss of ₹20,000 from house property to be
carried forward to next assessment year for set-off against income from house property
of that year.
(b) Business loss of ₹ 1,00,000 is set off against bank interest of ₹ 80,000 and remaining
business loss of ₹ 20,000 will be carried forward as it cannot be set off against salary
income.
(ii) Computation of total income of Mr. A for the A.Y. 2024-25 under default tax
regime
Particulars ₹ ₹
Income from salary 4,00,000
Income from other sources (interest on fixed 80,000
deposit with bank)
Less: Business loss of ₹ 1,00,000 set-off to the (-) 80,000
extent of ₹ 80,000
Business loss of ₹ 20,000 to be carried forward
for set-off against business income of the next
assessment year
Gross total income/ Total Income 4,00,000
Notes:
(i) Under the default tax regime, loss from house property cannot be set off against
income under any other head. Therefore, the loss of ₹ 2,20,000 from house property to be
carried forward to next assessment year for set-off against income from house property
of that year.
(ii) Business loss of ₹ 1,00,000 is set off against bank interest of ₹ 80,000 and remaining
business loss of ₹ 20,000 will be carried forward as it cannot be set off against salary
income.
Illustration: 3
Mr. B, a resident individual, furnishes the following particulars for the P.Y. 2023-24:
Particulars ₹
Income from salary (computed) 45,000
Income from house property (24,000)
Income from non-speculative business (22,000)
Income from speculative business (4,000)
Short-term capital losses (25,000)
Long-term capital gains taxable u/s 112 19,000
What is the total income chargeable to tax for the A.Y. 2024-25, assuming that he pays
tax under section 115BAC?
Solution:
Particulars ₹ ₹
Income from salaries 45,000
Income from house property
Loss from house property to be carried forward [Note (i)] (24,000)
Profits and gains of business and profession
Business loss to be carried forward [Note (ii)] (22,000)
Speculative loss to be carried forward [Note (iii)] (4,000)
Capital Gains
Long term capital gain taxable u/s 112 19,000
Short term capital loss ₹ 25,000 set off against long-term
capital gains to the extent of ₹ 19,000 [Note (iv)] (19,000)
Nil
Balance short term capital loss of ₹ 6,000 to be carried
forward [Note (iv)]
Taxable income 45,000
Notes:
(i) Since Mr. B is paying tax under the default tax regime u/s 115BAC, loss from house
property cannot be set off against income under any other head. Hence, such loss has to
be carried forward to the next year for set-off against income from house property, if
any.
(ii) Business loss cannot be set-off against salary income. Therefore, loss of ₹22,000 from
the non-speculative business cannot be set off against the income from salaries. Hence,
such loss has to be carried forward to the next year for set-off against business profits,
if any.
(iii) Loss of ₹4,000 from the speculative business can be set off only against the income
from the speculative business. Hence, such loss has to be carried forward.
(iv) Short term capital loss can be set off against both short term capital gain and long-
term capital gain. Therefore, short term capital loss of ₹ 25,000 can be set-off against
long-term capital gains to the extent of ₹19,000. The balance short term capital loss of
₹6,000 cannot be set-off against any other income and has to be carried forward to the
next year for set-off against capital gains, if any.
Illustration: 4
Particulars ₹
Income from the activity of owning and maintaining the race horses 75,000
Income from textile business 85,000
Brought forward textile business loss (relating to A.Y. 2023-24) 50,000
Brought forward loss from the activity of owning and maintaining the race 96,000
horses (relating to A.Y. 2021-22)
What is the total income in the hands of Mr. D for the A.Y. 2024-25?
Solution:
Particulars ₹ ₹
Income from the activity of owning and maintaining race 75,000
horses
Less: Brought forward loss of ₹ 96,000 from the activity of
owning and maintaining race horses set-off to the extent of
₹ 75,000 75,000
Nil
Balance loss of ₹ 21,000 (₹ 96,000 – ₹ 75,000) from the
activity of owning and maintaining race horses to be carried
forward to A.Y.2025-26
Income from textile business 85,000
Less: Brought forward business loss from textile business 50,000 35,000
Total income 35,000
Note: Loss from exempt source (for example agricultural loss) cannot be carried
forward and set-off against income under any head of income in subsequent
assessment year.
Illustration: 5
Mr. E has furnished his details for the A.Y. 2024-25 as under:
Particulars ₹
Income from salaries (computed) 1,50,000
Income from speculation business 60,000
Loss from non-speculation business (40,000)
Short term capital gain 80,000
Long term capital loss of A.Y. 2022-23 (30,000)
Winning from lotteries (Gross) 20,000 20,000
Solution:
Particulars ₹ ₹
Income from salaries 1,50,000
Income from speculation business 60,000
Less : Loss from non-speculation business (40,000) 20,000
Short-term capital gain 80,000
Winnings from lotteries 20,000
Taxable income 2,70,000
Note: Long term capital loss can be set off only against long term capital gain. Therefore,
long term capital loss of ₹30,000 has to be carried forward to the next assessment year.
Comprehensive Questions
Question: 1
Mr. Krishan, residing in Indore, provides the following information for the financial year
2023-24:
Particulars ₹
Income from textile business 4,60,000
Income from speculation business 25,000
Loss from gambling 12,000
Loss on maintenance of race horse 15,000
Current year depreciation of textile business not adjusted in the 5,000
income given above.
Unabsorbed depreciation of assessment year 2022-23 10,000
Speculation business loss of assessment year 2023-24 30,000
Compute the Gross Total Income of Mr. Krishan for the Assessment year 2024-25 and
also state the losses eligible for carry forward and period upto which such losses can be
carried forward.
Question: 2
Particulars ₹
Income under the head salary 4,00,000
Loss from self-occupied property (-) 70,00
Business loss (-) 1,00,000
Bank interest (FD) received 80,000
Question: 3
Mr. B, a resident individual, furnishes the following particulars for the P.Y. 2023-24:
Particulars ₹
Income under the head Salary 45,000
What is the total income chargeable to tax for the A.Y. 2024-25?
Question: 4
During the P.Y. 2023-24, Mr. C has the following income and the brought forward losses:
Particulars ₹
Short term capital gains on sale of shares 1,50,000
Long term capital loss of A.Y. 2022-23 (96,000)
Short term capital loss of A.Y. 2023-24 (37,000)
Long term capital gain 75,000
What is the capital gain taxable in the hands of Mr. C for the A.Y. 2024-25?
Question: 5
Particulars ₹
Income from the activity of owning and maintaining the race horses 75,000
Income from textile business 85,000
Brought forward textile business loss 50,000
Brought forward loss from the activity of owning and maintaining the race 96,000
horses (relating to A.Y. 2021-22)
What is the taxable income in the hands of Mr. D for the A.Y. 2024-25?
Question: 6
Mr. E has furnished his details for the A.Y. 2024-25 as under:
Particulars ₹
Income under the head salary 1,50,000
Income from speculation business 60,000
Question: 7
Compute the gross total income of Mr. F for the A.Y. 2024-25 from the information given
below –
Particulars ₹
Net income from house property 1,25,000
Income from business (before provided for depreciation) 1,35,000
Short term capital gains on sales of shares 56,000
Long term capital loss from sale of property (brought forward from (90,000)
A.Y. 2023-24)
Income from growing and manufacturing of tea 1,20,000
Current year depreciation 26,000
Brought forward business loss (loss incurred six years ago) (45,000)
Question: 8 (5 Marks)
Mr. X a resident individual submits the following information, relevant to the previous year
ending March 31, 2024:
Particulars ₹
(i) Income from Salary (Computed) 2,22,000
(ii) Income from House Property
- House in Delhi 22,000
- House in Chennai (–)2,60,000
- House in Mumbai (self-occupied) (–)20,000
(iii) Profit and gains from business of profession
- Textile business 18,000
- Cosmetics business (–)22,000
- Speculative business – 1 (–)74,000
- Speculative business – 2 46,000
(iv) Capital gains
Short term capital loss from sale of property (–)16,000
Determine the gross total income of Mr. X for the assessment year 2024-25 and the
losses to be carried forward assuming that he does not opt to be taxed under section
115BAC.
Question: 9 (5 Marks)
Following are the details of incomes/ losses of Mr. Rishi for the F.Y. 2023-24:
You are required to determine the Gross total income of Mr. Rishi for Assessment year
2024-25.
Mr. Rajeev submits the following information for the previous year 2023-24.
₹
(i) Income under the head salary 6,50,000
(ii) Income from House-I 55,000
(iii) Loss from house-II (self-occupied property) 1,25,000
(iv) Loss from house-III 1,90,000
Question: 11 (4 Marks)
Mr. Shyam, a resident of Chandigarh, provides the following information for the financial
year 2023-24:
Particulars ₹
Income from textile business 4,60,000
Income from speculation business 25,000
Loss from gambling 12,000
Loss on maintenance of race horse 15,000
Eligible current year depreciation of textile business not adjusted in the 5,000
income given above.
Unabsorbed depreciation of Assessment year 2023-24 10,000
brought forward speculation business loss of Assessment year 2023-24 30,000
Compute the Gross total Income of Mr. Shyam for the assessment year 2024-25 and any
other item of expense or loss eligible for carry forward.
Question: 12 (4 Marks)
Mr. X provides the following details for the previous year ending 31.03.2024.
(i) Income under the head salary from XYZ Ltd. ₹6,00,000
(ii) Interest on FD with SBI for the Financial Year 2023-24 ₹72,000 (Net of TDS)
(iii) Determined long term capital loss of A.Y. 2022-23 ₹96,000
(iv) Long term Capital gain ₹75,000
(v) Loss of minor son ₹90,000 computed in accordance with the provisions of Income Tax
Act. Mr. X transferred his own house to his minor son without adequate consideration few
years back and minor son let it out and suffered loss.
(vi) Loss of his wife’s business (₹2,00,000)
She carried business with funds which Mr. X gifted to her.
You are required to compute taxable income of Mr. X for the A.Y. 2024-25.
Question: 13 (4 Marks)
Mr. X, a resident individual, furnishes the following particulars of his income and other
details for the previous year 2023-24. ₹
Question: 14 (4 Marks)
Mr. X an assessee aged 61 years gives the following information for the previous
year 31.03.2024:
Sr. Particulars ₹
a. Loss from profession 1,05,000
b. Capital loss on the sale of property-short term 55,000
c. Capital gains on sale of shares-long term 2,05,000
d. Loss in respect of self occupied property 15,000
e. Loss in respect of let out property 30,000
f. Share of loss from firm 1,60,000
g. Income from card games 55,000
Question: 15 (5 Marks)
Question: 16 (4 Marks)
The following are the details relating to Mr. X, a resident Indian, aged 57, relating to the
year ended 31.03.2024: ₹
Question: 17 (4 Marks)
Mr. X furnishes the following information for the year ended 31.03.2024: ₹
Question: 18 (5 Marks)
Mr. X submits the following details of his income for the assessment year 2024-25.
Question: 19 (5 Marks)
Determine the total income of Mr. X from the following information for the Assessment
Year 2024-25:
(i) Interest received on enhanced compensation (It relates to transfer of land in the
financial year 2018-19. Out of the above ₹65,000 relates to financial year 2023-24 and
the balance relate to preceding years) 4,00,000
(ii) Business loss relating to discontinued business of the assessment year 2018-19 brought
forward and eligible for set off. 1,50,000
(iii) Current year business income (i.e. financial year 2023-24) (Computed) 1,10,000
Question: 20 (6 Marks)
Mr. X furnishes you the following details for the year ended 31.03.2024:
(Note: Assume that the figures given above are computed and arrived at after considering
eligible deductions).
Other sources:
Gift from a Friend (non-relative) on 05.06.2023 60,000
Gift from Maternal Uncle on 25.02.2024 1,00,000
Gift from Grandfather’s Younger Brother on 10.02.2024 1,00,000
Compute the total income of Mr. X for the Assessment Year 2024-25.
Question: 21 (4 Marks)
Mrs. X, a resident individual, provides the following details of her income/ losses for the
year ended 31.03.2024:
Question: 22 (4 Marks)
Mr. X, a resident individual, furnishes the following particulars of his income and other
details for the previous year 2023-24:
₹
(i) Income under the head salary 18,000
(ii) Net annual value of house property 70,000
(iii) Income from business 80,000
(iv) Income from speculative business 12,000
(v) Long term capital gain on sale of land 15,800
(vi) Loss on maintenance of race-horse 9,000
(vii) Loss on gambling 8,000
Depreciation allowable under the Income-tax Act comes to ₹8,000 for which no treatment
is given above.
The other details of unabsorbed depreciation and brought forward losses of previous year
2020-21 are: ₹
(i) Unabsorbed depreciation 9,000
(ii) Loss from speculative business 16,000
(iii) Short term capital loss 7,800
Compute the gross total income of Mr. X, for the Assessment year 2024-25, and the
amount of loss that can or cannot be carried forward.
Question: 23 (4 Marks)
Mr. X, engaged in various types of activities, gives the following particulars of her income
for the year ended 31.03.2024: ₹
Illustration:1
Paras aged 55 years is a resident of India. During the F.Y. 2023-24, interest of ₹
2,88,000 was credited to his Non-resident (External) Account with SBI. ₹ 30,000, being
interest on fixed deposit with SBI, was credited to his saving bank account during this
period. He also earned ₹ 3,000 as interest on this saving account. Is Paras required to file
return of income?
What will be your answer, if he has incurred ₹ 3 lakhs as travel expenditure of self and
spouse to US to stay with his married daughter for some time?
Solution:
An individual is required to furnish a return of income under section 139(1) if his total
income, before giving effect to the deductions under Chapter VI-A or exemption under
section or section 54/54B/54D/54EC or 54F, exceeds the maximum amount not
chargeable to tax i.e. ₹ 3,00,000 under default tax regime u/s 115BAC and ₹ 2,50,000 if
exercises the option of shifting out of the default tax regime provided under section
115BAC(1A) (for A.Y. 2024-25).
Particulars ₹
Income from other sources
Interest earned from Non-resident (External) Account ₹ 2,88,000 Nil
[Exempt under section 10(4)(ii), assuming that Mr. Paras has been
permitted by RBI to maintain the aforesaid account]
Interest on fixed deposit with SBI 30,000
Interest on savings bank account 3,000
Gross Total Income 33,000
Less: Deduction under Chapter VI-A (not available under the default
tax regime under section 115BAC)
Total Income 33,000
In case he exercises the option of shifting out of the default tax regime provided under
section 115BAC(1A), he would be eligible for deduction of ₹ 3,000 under section 80TTA.
Accordingly, his total income would be ₹ 30,000. However, in both regimes, total income of
₹33,000, before giving effect to deductions under Chapter VI-A, would be considered.
Since the total income of Mr. Paras for A.Y.2024-25, before giving effect to the
deductions under Chapter VI-A, is less than the basic exemption limit in both regimes, he
is not required to file return of income for A.Y.2024-25.
Note: In the above solution, interest of ₹ 2,88,000 earned from Non-resident (External)
account has been taken as exempt on the assumption that Mr. Paras, a resident, has been
permitted by RBI to maintain the aforesaid account. However, in case he has not been so
permitted, the said interest would be taxable. In such a case, his total income, before
giving effect to, inter alia, the deductions under Chapter VI-A, would be ₹ 3,21,000
(₹30,000 + ₹ 2,88,000 + ₹ 3,000), which is higher than the basic exemption limit of
₹3,00,000 or ₹2,50,000, as the case may be.
Consequently, he would be required to file return of income for A.Y. 2024-25.
If he has incurred expenditure of ₹3 lakhs on foreign travel of self and spouse, he has to
mandatorily file his return of income on or before the due date under section 139(1), even
if his income is less than the basic exemption limit.
Comprehensive Questions
Question: 1
Enumerate the cases where a return of loss has to be filed on or before the due date
specifies u/s 139(1) for carry forward of the losses. Also enumerate the case where losses
can be carried forward even through the return of loss has not been filed on or before the
due date.
Question: 2
In the following cases relating to P.Y. 2023-24, the total income of the assessee or the
total income of any other person in respect of which he/she is assessable under Income
Tax Act, 1961 does not exceed the basis exemption limit. You are required to state with
reasons, whether the assessee is still required to file the return of income or loss for A.Y
2024-25 in each of the following independent situations:
(i) Manish & Sons (HUF) sold a residential house on which there arose a long-term capital
gain of Rs 12 lakh which was invested in Capital Gain Bonds u/s 54EC so that no long-term
capital gain was taxable.
(ii) Mrs. Archana was born in Germany and married in India. Her residential status under
section 6(6) of the Income Tax Act, 1961 is ‘resident and ordinarily resident’. She owns a
car in Germany which she uses for her personal purposes during her visit to her parents’
place in that country.
Mr. Hari aged 57 years is a resident of India. He provides you with the following details of
his incomes pertaining to F.Y. 2023-24.
- Interest on Non-Resident (External) Account maintained with State Bank of India as
per RBI stipulations - Rs 3,55,000
- Interest on savings bank account maintained with State Bank of India - Rs 8,000
- Interest on Fixed Deposits with Punjab National Bank - Rs 40,000
He seeks your advice on his liability to file return of income as per Income-tax Act, 1961
for the Assessment Year 2024-25.
What will be your answer, if he has incurred Rs 4 lakhs on travel expenses of his newly
married son and daughter in law's honeymoon in Canada?
(1) State whether quoting of PAN in the following transactions is mandatory or not,
as per the provisions of Income Tax Act 1961 for A.Y. 2024-25:
(i) Mr. A makes cash payment to a hotel Radisson Blu, Ahmedabad of Rs 50,000 against the
bill raised by the hotel.
(ii) Mr. Abhishek, in a single transaction, makes contract of Rs 1,20,000 for sale /purchase
of securities (other than shares) as defined in section 2(h) of the Securities Contracts
(Regulation ) Act, 1956.
(iii) Payment to Mutual Funds of Rs 70,000 for purchase of its units.
Mr. Sachin filed return on 30th September, 2024 related to Assessment Year 2024-25.
In the month of October 2024, his tax consultant found that the interest on fixed
deposit was omitted in the tax return.
(i) What is the time limit for filing a belated return?
(ii) Can Mr. Sachin file a revised return?
Justify the above with the relevant provisions under section 139. Assume that the due
date for furnishing return of income was 31st July, 2024 and the assessment was not
completed till the month of October 2024.
By whom should the return of income be signed in the case of following persons:
(i) Political Party;
(ii) Company which is being wound up;
(iii) Hindu Undivided Family, when karta is unable to sign, and
(iv) Scientific research association
(1) Mr. Kamal filed his Return of Income for the Assessment Year 2024-25 on 30-11-2024.
Can he revise such return of income? If so Why?
(2) Mr. Atal, a super senior citizen, has reported a Gross Total Income Rs 5,60,000 and
the deductions eligible under Chapter VI-A amounting to Rs 70,000 for the previous year
2023-24. Is he liable to file his return of income u/s 139(1) for the Assessment year
2024-25? If so, why?
Who are the persons authorized to verify return of income in the case of individual under
section 140 of the Income Tax Act, 1961?
Mr. X, a resident, has provided the following particulars of his income for the P.Y.
2023-24.
(i) Income under the head salary ₹3,40,000
(ii) Income under the head house property ₹3,00,000
(iii) Agricultural income from a land in Jaipur ₹1,80,000
(iv) Expenses incurred for earning agricultural income ₹1,20,000.
Compute his tax liability (under old tax regime) assuming his age is –
(a) 45 years
(b) 70 years
Solution:
Particulars Amount
Income under the head salary 3,40,000
Income under the head house property 3,00,000
Gross Total Income 6,40,000
Less: Deductions under Chapter VI-A Nil
Total Income 6,40,000
Illustration: 2
(a) ABC Ltd. an Indian company has agricultural income ₹350 lakhs and company has
distributed dividend of ₹60 lakhs to its shareholders and one of the shareholders, Mr.
X has received dividend of ₹7,00,000. Compute tax liability of the company and tax
liability of shareholder under old tax regime.
Solution:
Solution:
It is a foreign company; its tax liability shall be nil and tax liability of shareholder shall
be as given below:
Tax on ₹7,00,000 at slab rate 52,500
Add: HEC @ 4% 2,100
Tax Liability 54,600
(c) Presume it is Indian company and income is from business and not from agriculture
and turnover during F.Y. 2021-22 was 500 Crores.
Solution:
(d) Presume it is foreign company and income is from business and not from agriculture
Solution:
Illustration: 3
(a) XY partnership firm has two partners Mr. X and Mr. Y and profit-sharing ratio is 1:1
and the firm have agricultural income ₹300 lakhs without debiting salary or interest to
the partners. The firm has paid salary of ₹8 lakh to each of the partner and interest
of ₹4 lakh to each of the partner. Mr. X has income under the head house property ₹6
lakh and Mr. Y has income under the head house property ₹7 lakh. Compute tax liability
of the firm and also that of partners under old tax regime.
Solution:
Since partnership firm has agricultural income, it is exempt from income tax under
section 10(1).
(b) Presume in the above case partnership firm has income from business and not
agricultural income.
Solution:
Explain with brief reasons, whether the following income can be regarded as
agricultural income, as per the provisions of the Income-tax Act, 1961:
(i) Rent received for letting out agricultural land for a movie shooting.
(ii) Income from sale of seedlings in a nursery adjacent to the agricultural lands owned
by an assessee.
Solution:
(i) Rent received for letting out agricultural land for a movie shooting:
In the present case, rent is being derived from letting out of agricultural land for a
movie shoot, which is not an agricultural purpose and hence, it does not constitute
agricultural income.
As per Explanation 3 to section 2(1A), income derived from saplings or seedlings grown
in a nursery is deemed to be agricultural income, whether or not the basic operations
were carried out on land.
Therefore, the amount received from sale of seedlings in a nursery adjacent to the
agricultural lands owned by the assessee constitutes agricultural income.
Illustration: 5
Mr. X grows sugarcane and uses the same for the purpose of manufacturing sugar in his
factory. 30% of sugarcane produce is sold for ₹10 lacs, and the cost of cultivation of
such sugarcane is ₹5 lacs. The cost of cultivation of the balance sugarcane (70%) is ₹14
lacs and the market value of the same is ₹22 lacs. After incurring ₹1.5 lacs in the
manufacturing process on the balance sugarcane, the sugar was sold for ₹25 lacs.
Compute Mr. X’s business income and agricultural income.
Solution:
Income from sale of sugarcane gives rise to agricultural income and from sale of sugar
gives rise to business income.
Business income = Sales – Market value of 70% of sugarcane produce – Manufacturing
expenses
= ₹25 lacs – ₹22 lacs - ₹1.5 lacs = ₹1.5 lacs.
Agricultural income = Market value of sugarcane produce – Cost of cultivation
= [₹10 lacs + ₹22 lacs] – [₹5 lacs + ₹14 lacs]
= ₹32 lacs – ₹19 lacs
= ₹13 lacs.
Illustration: 6
Mr. X is engaged in growing and manufacturing of rubber. These are then sold in the
market for ₹30 lacs. The cost of growing rubber plants is ₹10 lacs and that of
manufacturing rubber is ₹8 lacs. Compute his total income.
Solution:
The total income of Mr. X comprises of agricultural income and business income.
Total profits from the sale of rubber = ₹30 lacs – ₹10 lacs – ₹8 lacs = ₹12 lacs.
Agricultural income = 65% of ₹12 lacs. = ₹7.8 lacs
Miss Deepika, a citizen of India, got married to Mr. John of Australia and left India
for the first time on 20.8.2023. She has not visited India again during the P.Y. 2023-
24. She has derived the following income for the year ended 31.03.2024:
Particulars ₹
(i) Income from sale of centrifuged latex processed from rubber 1,50,000
plants grown in Kanyakumari.
(ii) Income from sale of coffee grown and cured in Kodagu, 2,00,000
Karnataka
(iii) Income from sale of coffee grown, cured, roasted and grounded 5,00,000
in Colombo. Sale consideration was received in Chennai.
(iv) Income from sale of tea grown and manufactured in West 12,00,000
Bengal.
(v) Income from sapling and seedling grown in a nursery at Cochin. 2,00,000
Basic operations were not carried out by her on land
You are required to determine the residential status of Miss Deepika and compute the
business income and agricultural income of Miss. Deepika for the Assessment Year
2024-25.
Solution:
Miss Deepika is said to be resident if she satisfies any one of the following basic
conditions:
(i) Has been in India during the previous year for a total period of 182 days or more
(or)
(ii) Has been in India during the 4 years immediately preceding the previous year for a
total period of 365 days or more and has been in India for at least 60 days during the
previous year.
Miss Deepika’s stay in India during the P.Y. 2023-24 is 142 days [30+31+30+31+20]
which is less than 182 days. However, her stay in India during the P.Y. 2023-24
exceeds 60 days. Since, she left India for the first time, her stay in India during the
four previous years prior to P.Y. 2023-24 would be more than 365 days. Hence, she is a
resident for P.Y. 2023-24.
Further, Miss Deepika would be “Resident and ordinarily resident” in India in during the
previous year 2023-24, since her stay in India in the last seven previous years prior to
P.Y. 2023-24 is more than 730 days and she must be resident in the preceding ten
years.
Computation of business income and agricultural income of Miss Deepika for A.Y.
2024-25:
Notes:
(1) Since Miss Deepika is resident and ordinarily resident in India for A.Y. 2024-25,
her global income is taxable in India. Entire income from sale of coffee grown, cured,
roasted and grounded in Colombo is taxable as business income since such income is
earned from sale of coffee grown, cured, roasted and grounded outside India i.e., in
Colombo.
(2) As per Explanation 3 to section 2(1A), income derived from sapling or seedlings
grown in a nursery would be deemed to be agricultural income, whether or not the basic
operations were carried out on land. Hence, income of ₹2,00,000 from sapling and
seedling grown in a nursery at Cochin is agricultural income.
Illustration: 8
Mr. X has estates in rubber, tea and coffee. He derives income from them. He has a
nursery wherein he grows and sells the plants. For the previous year ending 31.03.2024,
he furnishes the following particulars of his income from estates and sale of plants.
You are requested to compute the taxable income and tax liability for the assessment
year 2024-25:
(i) Growing and manufacturing of rubber 5,00,000
(ii) Sale of coffee grown and cured 3,50,000
(iii) Growing and manufacturing of tea 7,00,000
(iv) Sale of plants from nursery 1,00,000
He has long term capital gain on the sale of agricultural land in Delhi ₹3,13,500. He has
received rent of ₹7,000 p.m. by letting out one farmhouse near Delhi and he has
incurred ₹20,000 on the repairs of the farm house. He has not paid municipal taxes for
the last ten years in connection with farmhouse and MCD has issued him a notice for
selling of farm house, hence he has paid municipal tax of ₹90,000.
Solution:
Agricultural Business
Income Income
(a) Income from growing and manufacturing of Rubber
{Rule 7A}
[Agricultural income 65% and business income 35%] 3,25,000 1,75,000
(b) Income from Coffee grown and cured {Rule 7B}
[Agricultural income 75% and business income 25%] 2,62,500 87,500
(c) Income from growing and manufacturing of Tea
{Rule 8}
[Agricultural income 60% and business income 40%] 4,20,000 2,80,000
(d) Income from growing and selling of plants 1,00,000 ----------
Option I
Loss under the head house property set off from long term capital gains
Long term capital gains 3,13,500
Loss under the head House Property (6,000)
Long term capital gains after adjusting loss from house property 3,07,500
Income under the head Business/Profession 5,42,500
Gross Total Income 8,50,000
Less: Deduction under section 80C to 80U Nil
Total Income 8,50,000
Option II
Loss under the head house property set off from business income
Income under the head Business/Profession 5,42,500
Loss under the head House Property (6,000)
Income under the head Business/Profession 5,36,500
Income under the head Capital gain (LTCG) 3,13,500
Gross Total Income 8,50,000
Less: Deduction under section 80C to 80U Nil
Total Income 8,50,000
Illustration: 9
Mr. X is employed in MP Agricultural University and getting basic pay ₹20,000 p.m. He
claims that it is his agricultural income. Discuss.
Solution:
Illustration: 10
Mr. X has sold his agricultural land in Delhi and there are long term capital gains of
₹10,00,000. Mr. X claims it to be his agricultural income. Discuss.
Solution:
Illustration: 11
Mr. X holds shares in ABC Ltd., an Indian Company, which is engaged in agricultural
operations. He has received dividends of ₹1,20,000 from ABC Ltd. and claims that it is
his agricultural income. Discuss.
Solution:
Example 1: A rural society has as its principal business the selling on behalf of its
member societies, butter made by these societies from cream sold to them by farmers.
The making of butter was a factory process separated from the farm.
The butter resulting from the factory operations separated from the farm was not an
agricultural product and the society was, therefore, not entitled to exemption under
section 10(1) in respect of such income.
Example 2: X was the managing agent of a company. He was entitled for a commission
at the rate of 10% p.a. on the annual net profits of the company. A part of the
company’s income was agricultural income. X claimed that since his remuneration was
calculated with reference to income of the company, part of which was agricultural
income, such part of the commission as was proportionate to the agricultural income
was exempt from income tax.
Since, X received remuneration under a contract for personal service calculated on the
amount of profits earned by the company; such remuneration does not constitute
agricultural income.
Example 3: Y owned 100 acres of agricultural land, a part of which was used as pasture
for cows. The lands were purely maintained for manuring and other purposes connected
with agriculture and only the surplus milk after satisfying the assessee’s needs was
sold. The question arose whether income from such sale of milk was agricultural income.
The regularity with which the sales of milk were affected and quantity of milk sold
showed that the assessee carried on regular business of producing milk and selling it as
a commercial proposition. Hence, it was not agricultural income.
Example 4: In regard to forest trees of spontaneous growth which grow on the soil
unaided by any human skill and labour there is no cultivation of the soil at all.
Even though operations in the nature of forestry operations performed by the
assessee may have the effect of nursing and fostering the growth of such forest
trees, it cannot constitute agricultural operations. Income from the sale of such forest
trees of spontaneous growth does not, therefore, constitute agricultural income.
Comprehensive Questions
Question: 1
Question: 2
Question: 3
Mrs. X (resident but not ordinarily resident) have incomes as given below:
(i) Income from growing and manufacturing of Tea in India ₹10,00,000
(ii) Income from house property situated outside India ₹3,50,000, received outside
India.
(iii) Income from agriculture in Nepal ₹1,50,000, received in India
(iv) Income from business in Paris and received in Paris ₹ 1,00,000
Compute her total income and tax liability for the assessment year 2023-24 under old
tax regime.
Question: 4
Mr. X (resident but not ordinarily resident) have incomes and losses as given below:
(i) Loss from house I in India ₹80,000
(ii) Income from house II in India ₹1,00,000
(iii) Carried forward loss assessment year 2011-12 from house III in India ₹50,000
(iv) Income under the head Business/Profession in India ₹2,20,000
(v) Royalty received in the UK for use of formula in U.K. ₹30,000
(vi) Long term capital gains in India ₹1,00,000
(vii) Income from agriculture in Indonesia but received in India and subsequently
invested it in Indonesia ₹50,000
(viii) Income from agriculture in India ₹2,00,000
Compute his total income and tax liability for the assessment year 2024-25 under old
tax regime.
Question: 5
A partnership firm XY has agricultural income ₹2,00,000, income under the head
business/profession ₹1,00,000 and long term capital gains ₹10,000.
Compute its tax liability for the assessment year 2024-25.
Question: 6
A partnership firm Z & Co. has agricultural income ₹20,00,000 and its partner Mr. Z
has received ₹5,00,000 being his share in the profits of partnership. Mr. Z has income
under the head house property ₹2,75,000.
Compute tax liability of the partnership firm and also that of Mr. Z.
Illustration: 1
Mr. X has been employed in ABC Ltd. since 01.07.2011 in the pay scale of 11,000 – 500 –
14,500– 1200 – 20,500 – 1,500 – 28,000. The employer has paid dearness allowance @ 20%
of his basic pay from 01.04.2023 to 30.09.2023 and thereafter dearness allowance was
allowed @ 25% of basic pay. Compute employee’s Tax Liability for Assessment Year 2024-
25 under old tax regime.
Solution:
Illustration: 2
Mr. X joins ABC Ltd. on 1st July’ 2014 in the pay scale of 20,000 – 500 – 25,000 – 700 –
30,600 – 1,000 – 37,600. The company has allowed him dearness allowance @ 5% of the
basic pay from 01.04.2023 up to 30.06.2023 and thereafter dearness allowance was
allowed @ 11% of the basic pay but up to 31.12.2023 and after that dearness allowance was
allowed @ 18% of the basic pay. Compute employee’s Tax Liability for the Assessment Year
2024-25.
Solution:
Illustration: 3
Mr. X joined ABC Ltd. on 01.07.2016 in the pay scale of 21,000 – 1,200 – 28,200 – 1,400 –
39,400 – 1,600 – 49,000. The employer has allowed him 3 increments in advance at the
time of joining. The employee’s salary is due on the 1st of next month. Employee was
allowed dearness allowance @ ₹7,000 p.m., during the previous year 2022-23 and @
₹9,000 p.m. in 2023-24. Compute Tax Liability for the Assessment Year 2024-25.
Solution: ₹
Illustration: 4
Mr. X is employed in ABC Ltd. getting basic pay ₹20,000 p.m., dearness allowance ₹7,000
p.m. and half of the dearness allowance forms the part of salary for the purpose of
retirement benefits.
The employer has paid bonus @ ₹500 p.m., commission @ 1% on the sales turnover of ₹20
lakhs. The employer paid him house rent allowance ₹6,000 p.m. Employee has paid rent
₹7,000 p.m. and was posted at Agra. Compute his Tax Liability for the Assessment Year
2024-25.
Solution: ₹
3. ₹72,000
Received = ₹72,000
Exempt = (₹53,800)
Taxable = ₹18,200
Illustration: 5
Situation 1: (Mr. A)
Taxable 60,000
Situation 2: (Mr. B)
1. Rent paid over 10% of retirement benefit salary (₹1,44,000 – ₹24,000) 1,20,000
2. 50% of retirement benefit salary 1,20,000
(Retirement benefit salary = 2,40,000)
3. House Rent allowance received (5,000 x 12) 60,000
Received 60,000
Exempt (60,000)
Taxable Nil
Situation 3: (Mr. C)
1. Rent paid over 10% of retirement benefit salary (₹1,68,000 – ₹24,000) 1,44,000
2. 50% of retirement benefit salary 1,20,000
(Retirement benefit salary = 2,40,000)
3. House Rent allowance received (5,000 x 12) 60,000
Received 60,000
Exempt (60,000)
Taxable Nil
Situation 4: (Mr. D)
Illustration: 6
Mr. X is employed in ABC Ltd. getting basic pay ₹20,000 p.m. but it was increased to
₹30,000 p.m. w.e.f. 01.09.2023. The employer was paying him house rent allowance ₹6,000
p.m. but it was decreased to ₹3,000 p.m. w.e.f. 01.11.2023. The employee was paying rent
₹4,000 p.m. but it was increased to ₹7,000 p.m. w.e.f. 01.12.2023. The employee was
posted at Amritsar but was transferred to Calcutta w.e.f. 01.02.2024. The employee has
resigned w.e.f. 01.03.2024. Compute his Tax Liability for the Assessment Year 2024-25
under old tax regime.
Solution: ₹
Working Note:
From April to August
Least of the following is exempt:
1. ₹20,000 – ₹10,000 = ₹10,000
2. 40% of retirement benefit salary = ₹40,000
(Retirement benefit salary = ₹1,00,000)
3. ₹30,000
Received = ₹30,000
Exempt = (₹10,000)
Taxable = ₹20,000
For November
Least of the following is exempt:
1. ₹4,000 – ₹3,000 = ₹1,000
2. 40% of retirement benefit salary = ₹12,000
(Retirement benefit salary = ₹30,000)
3. ₹3,000
Received = ₹3,000
Exempt = (₹1,000)
Taxable = ₹2,000
For February
Least of the following is exempt:
1. ₹7,000 – ₹3,000 = ₹4,000
2. 50% of retirement benefit salary = ₹15,000
(Retirement benefit salary = ₹30,000)
3. ₹3,000
Received = ₹3,000
Exempt = (₹3,000)
Taxable = Nil
Total = 20,000 + 10,000 + 2,000 + Nil + Nil = ₹32,000
Illustration: 7
Examine with brief reasons, whether the following are chargeable to income tax and· the
amount liable to tax with reference to the provisions of the Income Tax Act,1961:
(i) Allowance received by an employee Mr. Ram working in a transport system at ₹12,000
p.m. which has been granted to meet his personal expenditure while on duty. He is not in
receipt of any daily allowance from his employer.
Solution:
Any allowance granted to an employee working in a transport system to meet his personal
expenditure during his duty is exempt provided he is not in receipt of any daily allowance.
The exemption is 70% of such allowance (i.e., ₹8,400 per month being, 70% of ₹12,000) or
₹10,000 per month, whichever is less.
Hence, 1,00,800 (i.e., ₹8,400 x 12) is exempt. Balance ₹43,200 (₹1,44,000 – ₹1,00,800) is
taxable in the hands of Mr. Ram.
Illustration: 8
Mr. X is employed in Central Government getting basic pay ₹18,000 p.m., dearness
allowance ₹6,000 p.m. Employer has paid children education allowance ₹700 p.m. per child
w.e.f. 01.09.2023 and hostel allowance of ₹1,000 p.m. for one child w.e.f. 01.10.2023.
Employer has paid transport allowance ₹1,700 p.m. w.e.f. 01.11.2023. Employer has paid
house rent allowance ₹5,000 p.m. w.e.f 01.01.2024.
The employee has resigned from 01.02.2024 and has taken up a new job w.e.f. 01.03.2024.
He is getting basic pay ₹27,000 p.m. and house rent allowance ₹4,000 p.m.
Compute his Tax Liability for the Assessment Year 2024-25 under old tax regime.
Solution: ₹
Exempt = Nil
Taxable = ₹5,000
For March
Least of the following is exempt:
1. Nil
2. 40% of retirement benefit salary = ₹10,800
(Retirement benefit salary = ₹27,000)
3. ₹4,000
Received = ₹4,000
Exempt = Nil
Taxable = ₹4,000
Total = ₹5,000 + ₹4,000 = ₹9,000
Illustration: 9
Mr. X has two sons. He is in receipt of children education allowance of ₹150 p.m. for his
elder son and ₹70 p.m. for his younger son. He also receives the following allowances:
Tribal area allowance: ₹500 p.m. Compute his taxable allowances.
Solution:
Illustration: 10
Mr. X is employed in ABC Ltd. getting basic pay ₹22,000 p.m., dearness allowance ₹5,000
p.m. He was retired on 21.12.2023. The employer has allowed him pension of ₹9,000 p.m.
and the employee has requested for commutation of 52% of his pension. The employer has
allowed him such commutation on 01.02.2024 and has paid ₹5,61,600. The employer has
paid him gratuity of ₹6,95,000 and employee has completed service of 20 years and 11
months. Compute Tax Liability for the Assessment Year 2024-25 under old tax regime.
Solution: ₹
Working Note:
For December’ 2023
9,000 x 9/30 = 2,700
For January’ 2024
9,000 x 1 = 9,000
From February to March’ 2024
9,000 x 48% x 2 = 8,640
Total = Rs.2,700 + Rs.9,000 + Rs.8,640 = 20,340
HW Question: 1
TP: 9 Gratuity
Illustration: 11
Mr. X was employed in ABC Ltd. getting basic pay ₹18,000 p.m. but it was increased to
₹24,000 p.m. w.e.f. 01-07-2023, dearness allowance Rs.6,000 p.m. but it was increased to
₹9,000 p.m. w.e.f. 01-07-2023 (50% of DA forms part of salary). The employee was
retired on 10.01.2024 after serving the employer for 20 years and 10 months. The
employer has paid him gratuity of ₹9,10,000 and the employee was covered under Payment
of Gratuity Act, 1972. Compute taxable portion of gratuity and also Tax Liability under old
tax regime.
Solution: ₹
(b) Presume Mr. X is not covered in the Payment of Gratuity Act 1972.
Solution: ₹
Illustration: 12
Mr. X is retired from ABC Ltd. w.e.f. 11.11.2023 after serving the employer for 20 years
and 11 months. The employer has paid him gratuity of ₹5,75,000. At the time of
retirement, employee’s basic pay was ₹18,000 p.m. However, upto 31.07.2023 it was
₹11,000 p.m. and the employee was getting dearness allowance ₹5,000 p.m. but it was
₹3000 p.m. upto 31.07.2023. 30% of the dearness allowance forms part of salary. Compute
taxable part of gratuity and employee’s Tax Liability under old tax regime.
Solution: ₹
Solution: ₹
2. Rs. 20,00,000
3. 15/26 x 23,000 x 21 = Rs. 2,78,653.85
Received = Rs. 5,75,000
Exempt = (Rs. 2,78,653.85)
Taxable = Rs. 2,96,346.15
Illustration: 13
Solution:
Illustration: 14
Mr. X is retired from ABC Ltd. on 10.11.2023 after serving the employer for 20 years and
10 months. The employer has paid him leave salary of ₹5,00,000. The employee was
entitled for 2 months leave per year of service. During entire service, he has availed 6
months leave and has encashed 7 months leave. The employee was getting basic pay
₹27,000 p.m. but it was increased to ₹33,000 p.m. w.e.f. 01-07-2023. He was getting DA
₹9,000 per month but it was increased to ₹12,000 per month w.e.f. 01-07-2023.
50% of DA forms part of salary. Compute his Tax Liability for the Assessment Year 2024-
25 under old tax regime.
Solution: ₹
Illustration: 15
Mr. X is retired from ABC Ltd. w.e.f. 01.12.2023 after serving the employer for 20 years
and 10 months. The employer has paid him leave salary of ₹3,75,000. The employee was
entitled for 20 days leave per year of service. During entire service, he has availed 35
days of leave and has encashed 10 days of leave. The employee was getting basic pay
₹27,000 p.m. but it was increased to ₹ 30,000 p.m. w.e.f. 01-07-2023. The employer has
allowed him pension of ₹6,000 p.m. and employee was allowed commutation of 1/3rd of his
pension on 01.03.2024 amounting to ₹2,40,000.Compute his Tax Liability for the
Assessment Year 2024-25 under old tax regime.
Solution: ₹
For March
6,000 x 2/3 = 4,000
Total = 18,000 + 4,000 = 22,000
Illustration: 16
Mr. X retired w.e.f. 01.12.2023 after 20 years 10 months of service, receiving leave salary
of ₹ 5,00,000. Other details of his salary income are:
Basic Salary : ₹ 5,000 p.m. (₹1,000 was increased w.e.f. 01.04.2023)
Dearness Allowance : ₹ 3,000 p.m. (60% of which is for retirement benefits)
Commission : ₹ 500 p.m.
Bonus : ₹ 1,000 p.m.
Leave availed during service : 480 days
He was entitled to 30 days leave every year.
You are required to compute his taxable leave salary assuming:
(a) He is a government employee.
(b) He is a non-government employee.
Solution:
Illustration: 17
Mr. X is employed in ABC limited basic pay ₹70,000 per month, D.A. ₹30,000 per month,
Bonus ₹50,000 and commission @ 1% on sales turnover of ₹100 Lakh. The employer has
contributed ₹18,000 per month to unrecognized provident fund and employee has also
contributed an equal amount. During the year, Interest of ₹2,00,000 was credited on
employee plus employer contribution @ 10% per annum. Compute his Income and Tax
Liability under the old tax regime.
Solution:
Illustration: 18
Mr. X retires from service on December 31, 2023, after 25 years of service. Following are
the particulars of his income/investments for the previous year 2023-24:
Particulars
Basic pay @ ₹16,000 per month for 9 months 1,44,000
Dearness pay (50% forms part of the retirement benefits)
₹8,000 per month for 9 months 72,000
Lumpsum payment received from the Unrecognized Provident Fund 6,00,000
Deposits in the PPF account 40,000
Out of the amount received from the provident fund, the employer’s share was ₹2,20,000
and the interest thereon ₹50,000. The employee’s share was ₹2,70,000 and the interest
thereon ₹60,000. What is the taxable portion of the amount received from the
unrecognized provident fund in the hands of Mr. X for the assessment year 2024-25?
Solution:
Taxable portion of the amount received from the URPF in the hands of Mr. X for the A.Y.
2024-25 is computed hereunder:
Amount taxable under the head “Salaries”:
Employer’s share in the payment received from the URPF 2,20,000
Interest on the employer’s share 50,000
Total 2,70,000
Amount taxable under the head “Income from Other Sources”:
Interest on the employee’s share 60,000
Total amount taxable from the amount received from the fund 3,30,000
Illustration: 19
Mrs. X is employed in Central Government since 01.01.2023 and is getting basic pay of
₹1,00,000 p.m. She has contributed ₹15,000 p.m. to the notified pension scheme of Central
Government and employer has also contributed an equal amount. She has paid premium of
Jeevan Suraksha Policy ₹3,000 and invested ₹1,00,000 in NSC. Compute her tax liability
for the assessment year 2024-25 under old tax regime.
Solution: ₹
Illustration: 20
The basic salary of Mr. A is ₹80,000 p.m. Both Mr. A and his employer contribute 10% of
basic salary to the pension scheme referred to in section 80CCD. Explain the tax
treatment in respect of such a contribution in the hands of Mr. A.
Solution:
Illustration: 21
Mr. Srivastava, aged 40 years, a salaried employee of Nirja Ltd. was contributing to
National Pension Scheme ₹50,000 every year since 2018 and was claiming deduction under
section 80CCD. In December 2022, he opted out of the pension scheme and withdrew a
lump sum amount of ₹2,00,000. Is the amount so withdrawn taxable? If yes, how much is
the taxable amount?
Solution:
As per section 80CCD, If an assessee has received any amount from the accumulated
balance under National Pension Scheme, the amount so received is taxable but w.e.f.
assessment year 18-19 some exemption has been granted u/s 10(12A) and is as given below:
Any payment from the National Pension System Trust to an employee on closure of his
account or on his opting out of the pension scheme referred to in section 80CCD shall be
exempt to the extent of 60% of the total amount payable to him at the time of such
closure or his opting out of the scheme.
Further as per section 80CCD, Lumpsum amount received by the nominee on the death of
the assessee shall be fully exempt from Income Tax.
Gross Value 2,00,000
Less: Exempt u/s 10(12A) (60% x 2,00,000) (1,20,000)
Taxable Value 80,000
Illustration: 22
Solution:
Illustration: 23
Mr. X is employed in ABC Ltd. getting basic pay ₹24,000 p.m. and the employee has paid
professional tax ₹200 p.m. and the employer has provided him motor car for official as
well as personal use and its engine capacity is 1.2 litres and it is a chauffeur driven car and
all expenses are met by the employer himself. Compute Tax Liability for the Assessment
Year 2024-25 under old tax regime.
Solution: ₹
Illustration: 24
Mr. X is employed in ABC Ltd. getting basic pay ₹22,000 p.m. Employer has paid
professional tax of ₹75 p.m. on behalf of the employee and employee himself has paid
professional tax of ₹25 p.m. The employer has provided him rent free accommodation
which is owned by the employer himself and it is provided at a place with population of
5,00,000.
The employer has provided him three motor cars for official as well as personal use with
particulars as given below:
I II III
Actual cost 4,00,000 3,00,000 2,50,000
Engine capacity 1.8 liters 1.6 liters 1.4 liters
Petrol expenses 3,000 10,000 15,000
Repairs 5,000 4,000 3,000
Solution: ₹
3,96,100
Income under the head Capital Gains (LTCG) 5,000
Income under the head Other Sources (Gambling activities) 11,000
Gross Total Income 4,12,100
Less: Deduction u/s 80C to 80U Nil
Total Income 4,12,100
H) Leave travel concession [Exemption allowed only under old tax regime]
Illustration: 25
(ii) Compute the amount of LTC Exemption in the following cases with reference to the
provision under Income Tax Act, 1961:
(a) Mr. X went on a holiday on 09.09.2023 to Mysore with his wife and 3 children - one
daughter born on 02.02.2015 and twin sons born on 05.05.2017. The total cost of travel
was ₹80,000. The ticket cost for Mr. X and his wife was ₹50,000 and for all three
children was ₹30,000. The Employer reimbursed total ticket cost ₹80,000.
(b) In the above case (a), if among his 3 children the twin sons born on 02.02.2015 and the
daughter was born on 05.05.2017, what shall be the exemption?
Solution:
As per Section 10(5), Leave Travel concession shall be allowed to employee and his family.
Exemption shall be allowed for the expenditure incurred during the trip.
Family shall include spouse and children of the employee however exemption shall be
allowed for maximum of 2 children but in case of multiple birth after the birth of one
child, exemption is allowed for all the children.
(i) As per the above provision Exemption shall be allowed for all the children as the case is
of multiple birth after the birth of one child. Hence cost of travel of all shall be exempt.
(ii) As per the above provision Exemption shall be allowed for only 2 children.
Hence Exemption shall be ₹80,000 – ₹30,000/3 = ₹ 70,000.
Taxable Amount shall be ₹80,000 - ₹70,000 = ₹ 10,000.
(R) Excess contribution by employer in EPF, NPS or Superannuation fund and Interest
on such contribution
Illustration: 26
Mrs. X, an employee of XYZ Ltd., submits the following information for the assessment
year 2024-25:
Salary: ₹2,56,000; City compensatory allowance: ₹8,000; Bonus: ₹10,200; Education
allowance: ₹4,000 (for her grandchildren); Income tax penalty paid by the employer:
₹2,000; Leave travel concession: ₹1,000 (expenditure incurred by the employee - nil); Free
residential telephone: ₹4,000; Free refreshment during office hours ₹4,000;
reimbursement of electricity bill by the employer: ₹1,060; reimbursement of gas bills:
₹1,000; Professional tax paid by the employer: ₹300 on behalf of Mrs. X; Professional tax
paid by Mrs. X: ₹150.
Determine the Total Income and Tax Liability of Mrs. X for the Assessment Year 2024-
25.
Solution:
₹
Computation of Income under the head Salary
Salary 2,56,000
City Compensatory Allowance 8,000
Bonus 10,200
Education Allowance {Sec 10(14) Rule 2BB} 4,000
Income tax penalty paid by employer {Sec 17(2)(iv)} 2,000
Leave Travel Concession {Sec 10(5) Rule 2B} 1,000
Free Refreshment Nil
Free Residential Telephone Nil
Payment of electricity bills by employer {Sec 17(2)(iv)} 1,060
Reimbursement of gas bills {Sec 17(2)(iv)} 1,000
Professional tax paid by employer {Sec 17(2)(iv)} 300
Gross Salary 2,83,560
Less: Standard Deduction u/s 16(ia) (50,000)
Less: Entertainment Allowance u/s 16(ii) Nil
Less: Professional Tax u/s 16(iii) (450)
Income under the head Sa 2,33,110
Gross Total Income 2,33,110
Less: Deductions u/s 80C to 80U Nil
Total Income 2,33,110
Illustration: 27
Mr. X is employed in Central Government getting basic pay ₹14,000 p.m., dearness
allowance ₹5,000 p.m., House rent allowance ₹4,000 p.m. w.e.f. 01.07.2023. However,
employee is residing in the house of his parents. Employer has paid cash allowance ₹300
p.m., medical allowance ₹250 p.m. and entertainment allowance ₹400 p.m. Employer has paid
professional tax ₹75 p.m. on behalf of the employee. Employee has saved ₹35 p.m. out of
entertainment allowance. Compute employee’s income under the head Salary and Tax
Liability for the Assessment Year 2024-25.
Solution: ₹
Illustration: 28
Mr. X receives the following emoluments during the previous year ending 31.03.2024.
Basic pay ₹ 40,000
Dearness Allowance ₹ 15,000
Commission ₹ 10,000
Entertainment allowance ₹ 4,000
Medical allowance ₹ 10,000
Professional tax paid ₹ 3,000 (₹ 2,000 was paid by his employer)
He has no other income. Determine the income from salary for A.Y. 2024-25, if Mr. X is a
State Government employee.
Solution:
Particulars ₹ ₹
Basic Salary 40,000
Dearness Allowance 15,000
Commission 10,000
Entertainment Allowance received 4,000
Medical allowance 10,000
Professional tax paid by the employer 2,000
Gross Salary 81,000
Standard Deduction u/s 16(ia) (50,000)
Less: Deductions
- under section 16(ii) Entertainment allowance being
lower of:
(a) Allowance received 4,000
(b) One fifth of basic salary [1/5 × 40,000] 8,000
(c) Statutory amount 5,000 (4,000)
- under section 16(iii) Professional tax paid (3,000)
Income from Salary 24,000
Comprehensive Questions
Question: 1
Mrs. Rohini, aged 62 years, was born and brought up in New Delhi. She got married in
Russia in 1996 and settled there since then. Since her marriage, she visits India for 60
days each year during her summer break. The following are the details of her income for
the previous year ended 31.03.2024:
S .No. Particulars ₹
1. Pension received from Russian Government 65,000
2. Long-term capital gain on sale of land at New Delhi (Computed) 3,00,000
3. Short-term capital gain on sale of share of Indian listed companies 60,000
in respect of which STT was paid both at the time acquisition as
well as at the time of sale (computed)
4. Short-term capital gain on sale of share of Indian listed companies 75,000
in respect of which STT was paid both at the time acquisition as
well as at the time of sale (computed)
5. Rent received (equivalent to Annual Value) in respect of house 90,000
property in New Delhi
You are required to ascertain the residential status of Mrs. Rohini and compute her total
income and tax liability in India for Assessment year 2024-25 under old tax regime.
Question: 2
(a) Rajesh was employed in Axis Ltd., Mumbai. He received a salary of ₹45,000 p.m. from
1.04.2023 to 20.09.2023. He resigned and left for Dubai for the first time on 28.09.2023
and got monthly salary of rupee equivalent of ₹90,000 from 01.10.2023 to 31.03.2024.
His salary for October to December was credited in his Mumbai bank account directly and
the salary for January to March 2024 was credited in his Dubai bank account.
The cost of his air tickets to Dubai costing ₹1,50,000 was funded by her sister staying in
London. The cost of his initial stay at Dubai costing ₹40,000 was funded by one of his
friends staying in Delhi.
He further received interest of ₹10,500 on his fixed deposits and ₹7,500 on his savings
a/c with his Mumbai bank. He also paid LIC Premiums of ₹15,000 for self, ₹10,000 for
spouse and ₹25,000 for dependent mother aged 71 years.
Compute taxable income of Mr. Rajesh for the Assessment Year 2024-25.
Question: 3
Mrs. Mitul, a resident individual, aged 63 years, is a qualified medical practitioner. She
runs her own clinic.
Income & Expenditure A/c of Mrs. Mitul for the year ending March 31st 2024 is as under:
Expenditure ₹ Income ₹
To Salary to Staff 1,20,000 By Consultation Fees 12,00,000
To Administrative Exp. 2,90,000 By Salary received from 1,80,000
True Care Hospitals (P) Ltd.
To Conveyance Expenses 24,000 By Rental Income from 78,000
House Property
To Power & Fuel 24,000 By Dividend from Foreign 10,000
Companies
To Interest on Housing Loan 1,00,000
To Interest on Education Loan 26,000
for son
To Amount paid to scientific 25,000
research association approved
& Notified under Section 35
To net profit 8,59,000
Total 14,68,000 Total 14,68,000
Explanatory Information:
(i) She is working part-time with True Care Hospitals (P) Ltd. Her salary details are as
under:
Further, during P.Y. 2023-24, her son had undergone a medical treatment in True Care
Hospitals (P) Ltd. free of cost. The hospital would have charged a sum of ₹60,000 for a
similar treatment to un-related patients.
(ii) She owns a residential house. Ground floor of the house is self-occupied by her while
first floor has been rented out since 01/10/2023. The reconstruction of the house was
started on 01-04-2023 and was completed on 30-09-2023. The monthly rent is ₹10,000.
The tenant also pays ₹3,000 p.m. as power back-up charges. She took a housing loan of ₹12
lakhs on 01-04-2023. Interest on housing loan for the period 01-04-2023 to 30-09-2023
was 60,000 and for the period 01-10-2023 to 31-03-2024 was ₹40,000. During the year,
she also paid municipal taxes for the F.Y. 2022-23 ₹5,000 and for F.Y. 2023-24 ₹5,000.
Mr. Madhvan is a finance manager in Star Private Limited. He gets a salary of ₹30,000 per
month. He owns two houses, one of which has been let out to his employer and which is in-
turn provided to him as rent free accommodation. Following details (annual) are furnished
in respect of two house properties for the Financial Year 2023-24.
House 1 House 2
Fair rent 75,000 1,95,000
Actual rent 65,000 2,85,000
Municipal Valuation 74,000 1,90,000
Municipal taxes paid 18,000 70,000
Repairs 15,000 35,000
Insurance premium on building 12,000 17,000
Ground rent 7,000 9,000
Nature of occupation Let-out to Star Private Let-out to Ms. Puja
₹ 17,000 were paid as Interest on loan taken by mortgaging House 1 for construction of
House 2. During the Previous year 2023-24, Mr. Madhvan purchased a rural agricultural
land for ₹2,50,000. Stamp valuation of such property is ₹3,00,000. Determine the taxable
income of Mr. Madhvan for the assessment year 2024-25. All workings should from part of
your answer.
Ms. Geeta, a resident individual, provides following details of her income/losses for the
year ended 31.03.2024:
Particulars Amount
(i) Income from salary (computed) 41,20,000
(ii) Rent received from house property situated in Delhi 5,00,000
(iii) Interest on loan taken for purchase of above property. Loan was 7,50,000
taken from a friend.
(iv) Rent received from house property situated in Jaipur 3,20,000
(v) Interest on loan taken for house property in Mumbai which is 1,57,000
self-occupied. Loan was taken from PNB on 01.01.1999 for
purchase of this property
(vi) Interest on loan taken for repair of house properties situated in 1,50,000
Mumbai and Delhi. Loan was taken on 01.04.2020 and was utilized
in 50:50 ratio for house properties situated in Mumbai and Delhi,
respectively.
(vii) Long term capital gains on sale of equity shares computed in 8,95,000
accordance with Section 112A
(viii) Interest on fixed deposit 73,000
(ix) Loss from textile business 7,50,000
(x) Speculation profit 2,30,000
(xi) Lottery income 75,000
(xii) Loss incurred by the firm in which she is a partner. 1,60,000
(xiii) Salary received as a partner from partnership firm. The same was 50,000
(xiv) allowed to firm.
(xv) Brought forward short-term capital loss on sale of gold. 2,75,000
Brought forward loss on sale of equity shares of the nature
specified u/s 111A 25,000
(xvi) Life insurance premium paid for her son who is 30 years of age 15,000
and is working in USA
Compute total income of Ms. Geeta for the assessment year 2024-25 and the amount of
loss that can be carried forward.
For the above solution, you may assume principal repayment of loan as under:
(1) Loan taken for purchase of house property in Delhi ₹2,50,000
(2) Loan taken for purchase of house property in Mumbai ₹ 50,000
(3) Loan taken for repair of house properties in Delhi and Mumbai ₹ 75,000
Workings notes should form part of your answer. Wherever necessary, suitable
assumptions may be made by the candidates and disclosed by way of note.
Mr. X working in a private company from last 10 years. His salary details for the financial
year 2023-24 are:
₹
i. Basic Salary 1,50,000 p.m.
ii. Dearness Allowance 55,000 p.m.
iii. Commission 35,000 p.m.
iv. Transport Allowance 5,000 p.m.
v. Medical Reimbursement 20,000 paid during the
year
Mr. X resigned from the services on 30th September, 2023. He was paid gratuity of ₹20
lakhs on his retirement. A lumpsum amount of ₹36 lakhs was also paid from unrecognized
provident fund. The provident fund amount consisted of employer contribution ₹13.20
lakhs and interest thereon ₹3 lakhs. The employee contribution was ₹16.20 lakhs and
interest thereon was ₹3.60 lakhs.
He had taken the possession of house on 28th February, 2024 after making payment of
final installment of housing loan to bank. Loan was taken on 01-04-2022. The accumulated
interest as on 31st march, 2023 was ₹ 1.5 lakh. He made payment of ₹ 2,20,000 during the
year which included interest ₹ 1,10,000 for 11 months.
He started business of hiring of goods vehicle, purchased 3 small goods vehicle on 15th
November, 2023 and3 heavy vehicles having gross weight of 15 MTs each on 1st December,
2023. He did not maintain books of accounts for income and expenditure of hiring of goods
vehicle. One of his friend gifted him ₹6 lakhs to purchase the vehicles.
He was holding 25% equity shares in CMF Ltd., an Indian company. The paid up share
capital of company as on 31st March, 2023 was ₹ 20 lakh divided into 2 lakh shares of ₹ 10
each which were issued at a premium of ₹ 30 each. Company allotted shares to
shareholders on 1st October, 2015. Company bought back 30% of its share on 30th April,
2023 under the provisions of Companies Act, 2013 on making payment of ₹60 per share.
He paid insurance premium of ₹20,000 on his life policy during the financial year 2023-24.
The policy was taken in April 2011 and sum assured was ₹ 1,50,000. He also made payment
of ₹ 25,000 L.I.C pension fund and premium of ₹ 40,000 towards Mediclaim policy for self
and wife.
Compute total income and tax payable thereon for the Assessment year 2024-25 under old
tax regime. There was no change in salary of Mr. X from last two years. Cost inflation
Index is:
Question: 7
Mr. Janakaraj, employed as General Manager in Rajus Refractories Pvt. Ltd., furnishes you
the undermentioned information for the year ended 31-03-2024:
(i) Basic salary upto 30-11-2023 ₹70,000 p.m.
Basic salary from 01-12-2023 ₹80,000 p.m.
Note: Salary is due and paid on the last day of every month.
(ii) Dearness allowance @ 50% of basic salary (not forming part of salary for retirement
benefits).
(iii) Bonus equal to one month salary. This was paid in November, 2023 on basic salary plus
dearness allowance applicable for that month.
(iv) Contribution of employer to recognized provident fund account of the employee @ 18%
of basic salary, employee also contributing an equivalent amount.
(v) Profession tax paid ₹6,000 of which ₹3,000 was paid by the employer.
(vi) Facility of laptop was provided to Janakaraj for both official and personal use. Cost of
laptop ₹65,000 and was purchased by the company on 11-10-2023.
(vii) Leave travel concession given to Janakaraj, his wife and three children (one daughter
aged 6 and twin sons aged 4). Cost of air tickets (economy class) reimbursed by the
employer ₹20,000 for adults and lumpsum of ₹25,000 for three children. Janakaraj is
eligible for availing exemption this year to the extent it is permissible under the Income-
tax Act, 1961.
Compute the taxable salary of Mr. Janakaraj under old tax regime.
Question: 8
Ms. Nandini, a resident individual, aged 48 years, is an assistant manager of Dye Hard Ltd.
She was appointed on 1st June, 2021 at a salary of ₹32,000 per month. During the previous
year 2023-2024, she received the following amounts from her employer.
(i) Dearness allowance (10% of basic pay which forms part of salary for retirement
benefits).
(ii) Bonus for the previous year 2022-2023 amounting to ₹32,000 was received on 01st
October, 2023.
(iii) Fixed Medical allowance of ₹20,000 for meeting medical expenditure.
(iv) She was also reimbursed the medical bill of her father-in-law dependent on her
amounting to ₹3,000.
(v) Ms. Nandini was provided;
• a laptop both for official and personal use. Laptop was acquired by the company on 1st
June, 2021 at ₹15,000.
• a domestic servant at a monthly salary of ₹1,000 which was reimbursed by her employer.
(vi) Dye Hard Ltd. allotted 500 equity shares in the month of December 2023 @ ₹ 150 per
share against the fair market value of ₹250 per share on the date of exercise of option by
Ms. Nandini. The fair market value was computed in accordance with the method
prescribed under the Act.
(vii) Professional tax ₹2,500 (out of which ₹1,800 was paid by the employer).
Compute the total Income of Ms. Nandini for the assessment year 2024-25. (Assume that
Ms. Nandini pays tax on the receipt basis)
Question: 9
Mr. Honey is working with a domestic company having a production unit in the U.S.A. for
last 15 years. He has been regularly visiting India for export promotion of company’s
product. he has been staying in India for at least 184 days every year.
He submits the following information:
Salary received outside India (for 6 months) ₹50,000 p.m.
Salary received in India (for 6 months) ₹50,000 p.m.
He has been given rent free accommodation in U.S.A. for which company pays ₹15,000 per
month as rent, but when he comes in India, he stays in the guest house of the company.
During this period, he is given free lunch facility. During the previous year company
incurred an expenditure of ₹48,000 on this facility.
He has been provided a car of 2000cc capacity in U.S.A. which is used by him for both
office and private purposes. The actual cost of the car is ₹8,00,000. But when he is in
India, the car is used by him and the members of his family only for personal purpose. The
monthly expenditure of car is ₹5,000. His elder son is studying in India for which his
employer spends ₹12,000 per year whereas his younger son is studying is U.S.A. and stays
in a hostel for which Mr. Honey gets ₹3,000 per month as combined allowance.
The company has taken an accident insurance policy and a life insurance policy. During the
previous year the company paid premium of ₹5,000 and ₹10,000 respectively.
Compute Mr. Honey’s taxable income from salary for the Assessment Year 2024-25.
Question: 10
Mrs. Babu, working as journalist with ABC Limited provides the following information for
the year ended 31-03-2024.
Basic salary ₹25,000 p.m.
DA (50% of it is meant for retirement benefits) 50% Basic Pay
Own contribution to Recognized Provident fund (R.P.F.) ₹30,000
Employer’s contribution to R.P.F 20% of Basic Salary
Interest credited in the R.P.F. @ 15% ₹15,000
Arrears of rent received from ABC Limited ₹69,000
Received interest ₹10,000 from Axis Bank Savings account during the year, and interest
of ₹12,040 from the debentures of M/s. Coal India ltd.
She made payment through cheque ₹12,500 for Mediclaim Insurance Policy for her major
daughter.
She had contributed ₹1,196 pm towards Atal Pension Yojana and ₹5,000 pm towards
Sukanya Samridhi account.
M/s. ABC Limited has taken residential house of Mrs. Babu as Company’s guest house and
later purchased from her in the 2018 at market value for ₹75 lakhs. Purchased cost was
only ₹10 lakhs in April, 2006.
During August, 2023 Mrs. Babu had lost her gold chain and a diamond ring which she had
purchased in April, 2005 for ₹1,17,000 and market value of these two items were
₹2,50,000 and she has received insurance compensation of ₹3,50,000 during Feb.,2024
Compute Total Income for the Asst. year 2024-25.
Question: 11
Mrs. Jaya is the marketing manager in XYZ limited. She gives you the following particulars:
Basic Salary ₹65,000 p.m.
Dearness Allowance ₹22,000 p.m. (30% is for retirement benefits)
Bonus ₹17,000 p.m.
Her employer has provided her with an accommodation on 1st April, 2023 at a concessional
rent. The house was taken on lease by XYZ Ltd. for ₹12,000 p.m. Mrs. Jaya occupied the
house from 1st November, 2023. ₹4,800 p.m. is recovered from the salary of Mrs. Jaya.
The employer gave her a gift voucher of ₹8,000 on her birthday. She contributes 18% of
her salary (Basic Pay+ 30% of DA) towards recognized provident fund and the company
contributes the same amount.
The company pays medical insurance premium to effect insurance on the health of Mrs.
Jaya ₹18,000. Motor car owned by the employer (cubic capacity of engine 1.4 liters)
provided to Mrs. Jaya from 1st November, 2023 which is used for both official and
personal purposes. Repair and running expenses of ₹50,000 were fully met by the company.
The motor car was self- driven by the employee.
Compute the income chargeable to tax under the head “Salaries” in the hands of Mrs. Jaya
for the Assessment Year 2024-25. Also compute her tax liability for under old tax regime.
Question: 12
Mr. Nambi, a salaried employee, furnishes the following details for the financial year
2023-24:
Basic salary 6,00,000
Dearness allowance 3,20,000
Commission 50,000
Entertainment allowance 7,500
Profession Tax (of this, 50% paid by employer) 7,000
Health insurance premium paid by employer 9,000
Gift voucher given by employer on his birthday 12,000
Life insurance premium of Nambi Paid by employer 34,000
Laptop provided for use at home. Actual cost of Laptop to employer 30,000
[Children of the assessee are also using the Laptop at home]
Employer owns a Tata Nano car, which was provided to the assessee,
Both for official and personal use. No driver was provided. (Engine cubic capacity less than
1.6 litres)
Annual credit card fees paid by employer [Credit card is not exclusively
used for official purposes; details of usage are not available] 2,000
You are required to compute the income chargeable under the head "Salaries" for the
assessment year 2024-25.
Question: 13
(i) Mr. X, a citizen of India, serving in the Ministry of Finance in India and transferred to
High Commission of Australia on 15th March 2023. He did not come to India during the
financial year 2023-24. His income during the financial year 2023-24 is given here under:
Salary from Govt. of India 7,20,000
Foreign Allowances from Govt. of India 6,00,000
Rent from a house situated at London, received in London 3,60,000
Interest accrued on National Saving Certificate during the year 2023-24 45,000
Compute The Gross Total Income of Mr. X for the Assessment year 2024-25.
Question: 14
Mr. Vinod Kumar, resident, aged 62, furnishes the following information pertaining to the
year ended 31.03.2024: (₹)
(i) Pension receives (Net of TDS) 6,27,000
(ii) Short-term capital gains (from sale of listed shares) 65,000
(iii) Long-term capital gains (from sale of listed shares) 1,24,000
(iv) Interest on fixed deposit from bank 1,60,000
(v) Pertaining to consultancy services provided by him:
Gross receipts 12,60,000
Expenses:
Rent for premises 1,44,000
Salary of P.A. 1,20,000
Stenographer's salary 1,00,000
Business Development expenditure 91,000
Conveyance 3,00,000
(vi) Contribution to PPF 1,10,000
(vii) Premium on life insurance policy taken on 10.01.2024
(sum assured ₹5,00,000) 60,000
(viii) Mediclaim Insurance Premium for self (paid otherwise than by cash) 27,000
Preventive health checkup expenses (in cash) 6,000
(ix) Donation given in cash to a charitable trust registered u/s 12AA 14,000
(eligible for deduction u/s 80G) of the Income-Tax Act, 1961
(x) Interest received from Post Office Savings A/c. 18,000
Additional information:
• TDS from pension 25,000
• 1/4th of conveyance expenses is estimated for personal use.
Compute the total income of the assessee for the assessment year 2024-25, under proper
heads of income. Listed share were sold in recognized stock exchange.
Question: 15
Mrs. X provides the following information for the financial year ending 31.03.2024.
Compute her total income and tax payable thereon for A.Y. 2024-2025 as per Income Tax
Act 1961.
Income / Receipts:
(1) Salary from M/s. XYZ, - ₹60,000 per month (joined from 1st March, 2023).
(2) She is in receipt of HRA, ₹15,000 per month and also educational allowance of ₹1,500
per month for all the three of her children.
(3) She bought a truck on 01.08.2023 and has been letting it on hire. She does not
maintain books of account for this business. But she declares for income tax purpose, that
she is earning net income of ₹11,000 per month from this business.
(4) She received ₹8,500 as interest on Post Office Savings Bank Account.
(5) She received ₹25,000 as interest from Company Deposits.
(6) Amounts withdrawn from National Savings Scheme (Principal ₹10,000 & Interest
₹25,000)
Expenses / Payments:
(1) Interest payable to bank ₹1,000 per month on loan for the purchase of truck.
(2) Total interest paid to the bank for loan borrowed for investing in company deposits is
₹5,000.
(3) Rent paid for residence is ₹18,000 per month.
(4) Tuition fees paid for the year 2023-24 for her three children is ₹50,000, ₹30,000 and
₹20,000 respectively, to educational institution situated in India.
(5) Medical insurance premium for her and for her husband is ₹30,000 (paid by cheque)
and ₹25,000 (paid by cash) respectively.
(6) She has deposited during the year, in 5-year Post Office Recurring Deposit Scheme
₹20,000.
Question: 16
Question: 17
Mr. X an employee of XYZ Co. Ltd. at Mumbai and covered by Payment of Gratuity Act,
retires at the age of 64 years on 31.12.2023 after completing 33 years and 7 months of
service. At the time of retirement, his employer pays ₹20,51,640 as Gratuity and
₹6,00,000 as accumulated balance of Recognized Provident fund.
He is also entitled for monthly pension of ₹8,000. He gets 75% of pension Commuted for
₹4,50,000 on 1st February, 2024.
Determine the salary chargeable to tax for Mr. X for the Assessment Year 2024-25 with
the help of following information: ₹
Basic Salary (₹80,000 x 9) 7,20,000
Bonus 36,000
House Rent Allowance (₹15,000 x 9) 1,35,000
Rent paid by Mr. X (₹10,000 x 12) 1,20,000
Employer contribution towards Recognized Provident Fund 1,10,000
Professional Tax paid by Mr. X 2,000
Note: Salary and Pension falls due on the last day of each month.
Question: 18
From the following details compute the total income of Mr. X, A resident individual aged
54 years for the year ended 31.03.2024. Tax payable need not be calculated.
1. Salary including Dearness Allowance 5,00,000
2. Bonus 15,000
3. Salary to servant provided by Employer 12,000
4. Free gas, electricity and water provided by employer 14,500
5 Cost of Laptop provided by the employer 40,000
(Used both for official and personal purposes)
Following additional information is provided:
(1) Mr. X purchased a flat in a Cooperative Housing Society in Delhi for ₹10,75,000 in
April, 2017 by taking loan from State Bank of India amounting to ₹5,00,000 @ 15% per
annum interest, ₹65,000 from his own savings and a deposit from a Nationalized Bank to
whom this flat was given on lease for 10 years at a monthly lease rental of ₹ 5,500. The
outstanding amount of loan is ₹1,60,000.
(2) Municipal Taxes paid by Mr. X ₹4,500 p.a.
(3) Insurance in respect of the said flat ₹1,275
(4) Mr. X earned a profit of ₹15,000 in shares speculation business and incurred a loss of
₹20,200 in speculation business of cotton.
(5) In the year 2021-22, he had gifted ₹ 50,000 to his wife and ₹30,000 to his son who
was aged 11 years then. These amounts were advanced to Mr. Mohan @ 15% per annum
interest.
(6) Mr. X received a gift of ₹25,000 each from his four friends on the occasion of his
birthday.
(7) He contributed ₹10,500 to Public Provident Fund and ₹6,000 to Unit Linked Insurance
plan.
(8) He deposited ₹60,000 in tax saver deposit with a Nationalized Bank in the name of his
married son.
(9) He has taken a policy on life for his married daughter on 01.04.2023 and paid a
premium of ₹25,000. The sum assured for policy is ₹2,00,000.
Illustration: 1
An individual for previous year 31.03.2024 has business income of ₹30,00,000. For
Previous Year 31.03.2023 he was subject to AMT as he was claiming deduction under
section 35AD. He has an AMT credit of ₹4,00,000. During Previous Year 31.03.2024,
he is not entitled to deductions under Chapter VI-A/ 10AA/ 35AD. What shall be tax
liability?
Solution:
Although AMT is not applicable to the assessee in Previous Year 31.03.2024, yet he can
claim AMT credit as per section 115JEE.
Illustration: 2
Mr. X, an individual set up a unit in Special Economic Zone (SEZ) in the financial year
2019-20 for production of washing machines. The unit fulfills all the conditions of
section 10AA of the Income-tax Act, 1961. During the financial year 2022-23, he has
also set up a warehousing facility in a district of Tamil Nadu for storage of agricultural
produce. It fulfills all the conditions of section 35AD. Capital expenditure in respect of
warehouse amounted to ₹75 lakhs (including cost of land ₹10 lakhs). The warehouse
became operational with effect from 1st April, 2023 and the expenditure of ₹75 lakhs
was capitalized in the books on that date.
Relevant details for the financial year 2023-24 are as follows:
Particulars ₹
Profit of unit located in SEZ 40,00,000
Export sales of above unit 80,00,000
Domestic sales of above unit 20,00,000
Profit from operation of warehousing facility 1,05,00,000
(Before considering deduction under Section 35AD)
Compute income-tax (including AMT under Section 115JC) liability of Mr. X for
Assessment Year 2024-25 both as per regular provisions of the Income-tax Act and as
per section 115BAC for Assessment Year 2024-25. Advise Mr. X whether he should opt
for section 115BAC.
Solutions:
Computation of total income and tax liability of Mr. X for A.Y. 2024-25
(Under the regular provisions of the Income-tax Act, 1961)
₹ ₹
Profits and gains of business or profession
Profit from unit in SEZ 40,00,000
Less: Deduction u/s 10AA [See Note (1) below] 32,00,000
Business income of SEZ unit chargeable to tax 8,00,000
Computation of adjusted total income of Mr. X for levy of Alternate Minimum Tax
Total Income (computed above as per regular provisions of income tax) 48,00,000
Add: Deduction under section 10AA 32,00,000
80,00,000
Add: Deduction under section 35AD 65,00,000
Less: Depreciation under section 32
on building @10% of ₹65 lakhs 6,50,000 58,50,000
Adjusted Total Income 1,38,50,000
Alternate Minimum Tax @ 18.5% 25,62,250
Add: Surcharge@15% (since adjusted total income > ₹1 crore) 3,84,338
29,46,588
Add: Health and Education cess@4% 1,17,863
30,64,451
Tax liability u/s 115JC (rounded off) 30,64,450
Since the regular income-tax payable is less than the alternate minimum tax payable,
the adjusted total income shall be deemed to be the total income and tax is leviable
@18.5% thereof plus surcharge@15% and cess@4%. Therefore, tax liability as per
section 115JC is ₹30,64,450.
Notes:
(1) Deductions u/s 10AA and 35AD are not allowable as per section 115BAC(2).
However, normal depreciation u/s 32 is allowable.
(2) Individuals or HUFs exercising option u/s 115BAC are not liable to alternate
minimum tax u/s 115JC.
Since the tax liability of Mr. X under section 115JC is lower than the tax liability as
computed u/s 115BAC, it would be beneficial for him not to opt for section 115BAC for
A.Y. 2024-25. Moreover, benefit of alternate minimum tax credit is also available to
the extent of tax paid in excess over regular tax.
(2) Deduction @100% of the capital expenditure is available under section 35AD for
A.Y. 2024-25 in respect of specified business of setting up and operating a
warehousing facility for storage of agricultural produce which commences operation on
or after 01.04.2009.
Further, the expenditure incurred, wholly and exclusively, for the purposes of such
specified business, shall be allowed as deduction during the previous year in which he
commences operations of his specified business if the expenditure is incurred prior to
the commencement of its operations and the amount is capitalized in the books of
account of the assessee on the date of commencement of its operations.
Deduction under section 35AD would, however, not be available on expenditure incurred
on acquisition of land.
In this case, since the capital expenditure of ₹65 lakhs (i.e., ₹75 lakhs – ₹10 lakhs,
being expenditure on acquisition of land) has been incurred in the F.Y. 2022-23 and
capitalized in the books of account on 01.04.2023, being the date when the warehouse
became operational, ₹65,00,000, being 100% of ₹65 lakhs would qualify for deduction
under section 35AD.
Comprehensive Questions
Question: 1
From the following particulars furnished by Mr. Ganesh, aged 58 years, a resident
Indian for the previous year ended 31.03.2024, you are requested to compute his total
income and tax liability under normal as well as special provisions (AMT), if any,
applicable to him for the Assessment Year 2024-25.
(i) He occupies ground floor of his residential building and has let out first floor for
residential use at an annual rent of ₹2,28,000. He has paid municipal taxes of ₹60,000
for the current financial year.
(ii) He owns an industrial undertaking established in a SEZ and which has commenced
operation during the financial year 2021-22. Total turnover of the undertaking was
₹200 lakhs, which includes ₹140 lakhs from the export turnover. This industrial
undertaking fulfills all the conditions of section 10AA of the Income-Tax Act,1961.
Profit from this industry is ₹25 lakhs.
(iii) He received royalty of ₹2,88,000 from abroad for a book authored by him on the
nature of artistic. The rate of royalty as 18% of value of books and expenditure made
for earning this royalty was ₹40,000. The amount remitted to India till 30th
September, 2024 is ₹2,30,000
(iv) Received 40,000 as interest on saving bank deposits.
(v) He also sold his vacant land on 10.11.2023 for ₹10 lakhs. The stamp duty value of
land as on 01.04.2001 was ₹4 lakhs. This land was acquired by him on 05.08.1995 for
₹1.80 lakhs. He had incurred registration expenses of ₹10,000 at that time.
The cost of inflation index for the year 2023-24 and 2001-02 are 348 and 100
respectively.
(vi) He paid the following amounts, out of his taxable income:
(a) Insurance premium of ₹39,000 paid on life insurance policy of son, who is not
dependent on him.
(b) Insurance premium of ₹48,000 on policy of his dependent father,
(c) Tuition fees of ₹42,000 for his three children to a school. The fees being ₹14,000
p.a. per child.
Solution:
Working Note:
(i) 15% of value of books 2,40,000
(2,88,000/18% x 15%)
but cannot exceed amount received within 6 months
from the end of the previous year i.e. 2,30,000
Allowed 2,30,000
Notes:
1. As per section 80C, Insurance Premium of son is allowed but for father is not
allowed. It is not specified whether policy of father is life policy or Mediclaim policy. It
is presumed that it is life policy.
2. As per section 80C, Tuition fee shall be allowed for maximum of 2 children.
3. Municipal tax paid for self-occupied portion is not allowed.
4. As per section 10AA, Industrial undertaking was started in 2021-22 and for first
five year assessee can claim 100% of export profit as exemption u/s 10AA. Export
profit means Total Profit/Total Turnover x Export turnover.
Question: 2
S.NO. Particulars ₹ ₹
(i) Interest on capital received from XYZ & Co., at 15% [in 1,50,000
accordance with the partnership deed]
(ii) Share of profit from the firm 35,000
(iii) Salary as working partner (fully allowed in the hands of 1,00,000
the firm)
(iv) Interest from bank on fixed deposit (Net of TDS) 40,500
(v) Interest on saving bank account 12,300
(vi) Income-tax refund received relating to assessment year 34,500
2023-24 including interest of ₹2,300
Net profit from wholesale business 5,60,000
(vii) Amounts debited include the following:
- Depreciation as per books 34,000
- Motor car expenses 40,000
- Municipal taxes for the shop 7,000
(For two half years; payment for one half year made on
12.7.2024 and for the other on 31.12.2024)
- Salary to manager by way of a single cash payment 21,000
(viii) The WDV of the assets (as on 1.4.2023) used in above
wholesale business is as under:
- Computers 2,40,000
- Computer printer 1,50,000
(ix) Motor car acquired on 31.12.2023 (20% used for 6,80,000
personal use)
(x) He owned a house property in Mumbai which was sold in
January 2017. He received arrears of rent in respect of
the said property in October 2023. 1,15,000
(xi) LIP paid for independent son 60,000
(xii) PPF of his wife 70,000
(xiii) Health insurance premium paid towards a policy covering 35,000
her mother aged 75 by way of cheque. She is not
dependents on him.
(xiv) Contribution toward Prime Minister National Relief Fund 50,000
You are required to compute the total income of the Mr. Suraj for the assessment year
2024-25 and the closing WDV of each block of assets.
Solution:
Particulars ₹ ₹
Income from house property
Arrears of rent 1,15,000
(taxable under section 25A even if Mr. Suraj is not the owner of
the house property in the P.Y. 2023-24) 34,500 80,500
Notes:
(1) Depreciation allowable under the Income-tax Rules, 1962
(2) Only to the extent the interest is allowed as deduction in the hands of the firm,
the same is includible as business income in the hands of the partner. Since interest is
paid in accordance with partnership deed, maximum interest allowable as deduction in
the hands of the firm is 12% p.a. Therefore, interest @12% p.a. amounting to ₹1,20,000
would be treated as the business income of Mr. Suraj.
Particulars ₹ ₹
Under section 80C
LIP for independent son 60,000
PPF paid in wife’s name 70,000
1,30,000
Since the maximum deduction under section 80C and 80CCE 1,30,000
is ₹1,50,000, the entire sum of ₹1,30,000 would be allowed
as deduction
Under Section 80D
Health insurance premium taken for mother is fully allowable 35,000
as deduction, even though she is not dependent on him. Since
she is senior citizen whole of amount is allowable as
deduction as it is within overall limit of ₹ 50,000
Under Section 80G
Contribution towards PM National Relief Fund eligible for
100% deduction without any qualifying limit 50,000
Under section 80TTA
Interest on saving bank account, restricted to 10,000
Total deduction 2,25,000